You are on page 1of 250

2015

FINANCIAL ACCOUNTING
AND REPORTING II
QUESTION BANK

CAF-07
Question
P

ICAP
Bank

Financial accounting
and reporting II
Second edition published by
Emile Woolf Limited
Bracknell Enterprise & Innovation Hub
Ocean House, 12th Floor, The Ring
Bracknell, Berkshire, RG12 1AX United Kingdom
Email: info@ewiglobal.com
www.emilewoolf.com

Emile Woolf International, February 2015

All rights reserved. No part of this publication may be reproduced, stored in a retrieval
system, or transmitted, in any form or by any means, electronic, mechanical, photocopying,
recording, scanning or otherwise, without the prior permission in writing of Emile Woolf
Publishing Limited, or as expressly permitted by law, or under the terms agreed with the
appropriate reprographics rights organisation.

You must not circulate this book in any other binding or cover and you must impose the
same condition on any acquirer.

Notice
Emile Woolf International has made every effort to ensure that at the time of writing the
contents of this study text are accurate, but neither Emile Woolf International nor its directors
or employees shall be under any liability whatsoever for any inaccurate or misleading
information this work could contain.

Emile Woolf International ii The Institute of Chartered Accountants of Pakistan


Certificate in Accounting and Finance

C
Financial accounting and reporting II

Contents
Page
Question and Answers Index v
Questions
Section A Questions 1
Answers
Section B Answers 93

Emile Woolf International iii The Institute of Chartered Accountants of Pakistan


Financial accounting and reporting II

Emile Woolf International iv The Institute of Chartered Accountants of Pakistan


Certificate in Accounting and Finance

I
Financial accounting and reporting II

Index to questions and answers


Question Answer
page page

CHAPTER 2 IAS 1: PRESENTATION OF FINANCIAL STATEMENTS

2.1 LARRY 2 94

2.2 MINGORA IMPORTS LIMITED 3 95

2.3 BARRY 4 97

2.4 OSCAR INC 6 99

2.5 CLIFTON PHARMA LIMITED 7 101

2.6 SARHAD SUGAR LIMITED 8 103

2.7 BSZ LIMITED 10 105

2.8 YASIR INDUSTRIES LIMITED 12 108

2.9 SHAHEEN LIMITED 14 112

2.10 MOONLIGHT PAKISTAN LIMITED 15 114

2.11 FIGS PAKISTAN LIMITED 17 115

CHAPTER 3 IAS 7: STATEMENTS OF CASH FLOWS

3.1 KLEA 19 119

3.2 STANDARD INC 21 121

3.3 FALLEN 22 124

3.4 BIN QASIM MOTORS LIMITED 24 126

3.5 ITTEHAD MANUFACTURING LTD 27 129

3.6 WASEEM INDUSTRIES LIMITED 29 131

3.7 JALIB INDUSTRIES LIMITED 31 134

Emile Woolf International v The Institute of Chartered Accountants of Pakistan


Financial accounting and reporting II

Question Answer
page page

3.8 APOLLO INDUSTRY LIMITED 32 136

3.9 MARVEL ENGINEERING LIMITED 34 137

CHAPTER 4 CONSOLIDATED ACCOUNTS: STATEMENTS OF


FINANCIAL POSITION BASIC APPROACH

4.1 HALL 36 139

4.2 HASSLE 37 140

4.3 HYMN 37 141

4.4 HANG 38 143

4.5 HASH 38 144

CHAPTER 5 CONSOLIDATED ACCOUNTS: STATEMENTS OF


FINANCIAL POSITION - COMPLICATIONS

5.1 HAIL 39 146

5.2 HAIRY 40 148

5.3 HARD 41 150

5.4 HALE 42 152

5.5 HELLO 42 153

5.6 HASAN LIMITED 43 155

CHAPTER 6 CONSOLIDATED ACCOUNTS: STATEMENTS OF


COMPREHENSIVE INCOME

6.1 HARRY 45 159

6.2 HORNY 46 161

6.3 HERON 47 163

6.4 HANKS 48 164

CHAPTER 7 PROPERTY, PLANT AND EQUIPMENT

7.1 ROONEY 50 168

7.2 EHTISHAM 51 170

7.3 CARLY 51 172

7.4 ADJUSTMENTS LIMITED 52 173

7.5 FAM 53 175

7.6 IMRAN LIMITED 54 177

7.7 HUMAYUN CHEMICALS LIMITED 55 179

7.8 FARADAY PHARMACEUTICAL LIMITED 55 180

Emile Woolf International vi The Institute of Chartered Accountants of Pakistan


Index to questions and answers

Question Answer
page page

7.9 SPIN INDUSTRIES LIMITED 56 182

7.10 SCIENTIFIC PHARMA LIMITED 56 183

7.11 QURESHI STEEL LIMITED 57 184

7.12 GRANITE CORPORATION 58 185

CHAPTER 8 IAS 38: INTANGIBLE ASSETS

8.1 FAZAL 59 186

8.2 HENRY 59 186

8.3 TOBY 60 187

8.4 BROOKLYN 60 188

8.5 ZOUQ INC 61 189

8.6 STAR-BRIGHT PHARMACEUTICAL


62 190
LIMITED

8.7 RAISIN INTERNATIONAL 62 191

CHAPTER 9 IAS 17: LEASES

9.1 DAWOOD 64 192

9.2 FINLEY 64 192

9.3 FABIAN 64 193

9.4 XYZ INC 65 194

9.5 SNOW INC 65 197

9.6 MIRACLE TEXTILE LIMITED 66 199

9.7 SHOAIB LEASING LIMITED 66 200

9.8 NEPTUNE LIMITED 67 202

9.9 QUARTZ AUTO LIMITED 67 204

9.10 LODHI TEXTILE MILLS LIMITED 68 205

9.11 NOMAN ENGINEERING LIMITED 68 206

CHAPTER 10 IAS 37: PROVISIONS CONTINGENT LIABILITIES AND


CONTINGENT ASSETS AND IAS 10: EVENTS OCCURRING AFTER THE
REPORTING DATE

10.1 BADAR 69 207

10.2 GEORGINA 69 207

10.3 EARLEY INC 70 209

10.4 ACCOUNTING TREATMENT 70 210

Emile Woolf International vii The Institute of Chartered Accountants of Pakistan


Financial accounting and reporting II

Question Answer
page page

10.5 J-MART LIMITED 71 211

10.6 AKBER CHEMICALS LIMITED 72 212

10.7 QALLAT INDUSTRIES LIMITED 72 213

10.8 SKYLINE LIMITED 73 213

10.9 WALNUT LIMITED 74 214

10.10 ATTOCK TECHNOLOGIES LIMITED 75 215

CHAPTER 11 IAS 8: ACCOUNTING POLICIES, CHANGES IN


ACCOUNTING ESTIMATES AND ERRORS

11.1 WONDER LIMITED 76 216

11.2 DUNCAN 77 217

11.3 MOHANI MANUFACTURING LIMITED 78 218

CHAPTER 12 IAS 12: INCOME TAXES

12.1 FRANCESCA 79 219

12.2 SHEP (I) 79 220

12.3 SHEP (II) 80 221

12.4 SHEP (III) 81 222

12.5 SHEP (IV) 82 224

12.6 WAQAR LIMITED 82 225

12.7 SHAKIR INDUSTRIES 83 227

12.8 MARS LIMITED 84 228

12.9 BILAL ENGINEERING LIMITED 85 230

12.10 GALAXY INTERNATIONAL 85 231

12.11 APRICOT LIMITED 86 232

CHAPTER 13 RATIO ANALYSIS

13.1 WASIM 87 233

13.2 AMIR AND MO 88 233

CHAPTER 14 ETHICAL ISSUES IN FINANCIAL REPORTING

14.1 ETHICAL ISSUES 90 235

14.2 SINDH INDUSTRIES LTD 90 236

Emile Woolf International viii The Institute of Chartered Accountants of Pakistan


Certificate in Accounting and Finance

A
Financial accounting and reporting II

SECTION
Questions

Emile Woolf International 1 The Institute of Chartered Accountants of Pakistan


Financial accounting and reporting II

CHAPTER 1 LEGAL BACKGROUND TO THE PREPARATION OF FINANCIAL


STATEMENTS
There are no questions specific to chapter one. This is because the learning outcomes in this
area concern the preparation of financial statements and these questions have given in
relation to chapter 2 in this question bank.

CHAPTER 2 IAS 1: PRESENTATION OF FINANCIAL STATEMENTS

2.1 LARRY
The trial balance of Larry at 31 December 2015 is as follows.
Rupees in million
Dr Cr
Administration charges 342
Bank account 89
Cash 2
Payables ledger 86
Accumulated amortisation on patents at 31 December 2015 5
Accumulated depreciation at 31 December 2015 918
Receivables ledger 189
Distribution expenses 175
Property, plant and equipment at cost 2,830
Interest received 20
Issued share capital 400
Loan 18
Patents at cost 26
Accumulated profits 1,562
Purchases 2,542
Sales 3,304
Inventories at 31 December 2014 118

6,313 6,313

The following information is also relevant.
(1) Inventories on 31 December 2015 amounted to Rs. 127 million.
(2) Current tax of Rs. 75 million is to be provided.
(3) The loan is repayable by equal annual instalments over three years.
Required
Prepare an statement of profit or loss (analysing expenses by function) for the year
ended 31 December 2015 and a statement of financial position as at that date.

Emile Woolf International 2 The Institute of Chartered Accountants of Pakistan


Questions

2.2 MINGORA IMPORTS LIMITED


The trial balance of Mingora Imports Limited at 31 December 2015 is as follows.
Rupees in million
Dr Cr
Patent rights 60
Work-in-progress, 1 January `2015 125
Leasehold buildings at cost 300
Ordinary share capital 600
Sales 1,740
Staff costs 260
Accumulated depreciation on buildings, 1 January 2015 60
Inventories of finished games, 1 January 2015 155
Consultancy fees 44
Directors salaries 360
Computers at cost 50
Accumulated depreciation on computers, 1 January 2015 20
Dividends paid 125
Cash 440
Receivables 420
Trade payables 92
Sundry expenses 294
Accumulated profits, 1 January 2015 121

2,633 2,633

The following information is also relevant.
(1) Closing inventories of finished games are valued at Rs. 180 million. Work in
progress has increased to Rs. 140 million.
(2) The patent rights relate to a computer program with a three year lifespan.
(3) On 1 January 2015 buildings were revalued to Rs. 360 million. This has not
yet been reflected in the accounts. Computers are depreciated over five years.
Buildings are now to be depreciated over 30 years.
(4) An allowance for bad debts (irrecoverable debts) of 5% is to be created.
(5) There is an estimated bill for current tax of Rs. 120 million which has not yet
been recognised.
Required
Prepare an statement of profit or loss (analysing expenses by nature for the year
ended 31 December 2015 and a statement of financial position as at that date.

Emile Woolf International 3 The Institute of Chartered Accountants of Pakistan


Financial accounting and reporting II

2.3 BARRY
Barry has prepared the following draft financial statements for your review

Barry: Statement of profit or loss for year to 31st August 2015


Rs. in
000
Sales revenue 30,000
Raw materials consumed (9,500)
Manufacturing overheads (5,000)
Increase in inventories of work in progress and finished goods 1,400
Staff costs (4,700)
Distribution costs (900)
Depreciation (4,250)
Interest payable (350)

6,700

Statement of financial position as at 31st August 2015
Rs. in Rs. in
000 000
Assets
Non-current
Freehold land and buildings 20,000
Plant and machinery 14,000
Fixtures and fittings 5,600

39,600
Current assets
Prepayments 200
Trade receivables 7,400
Cash at bank 700
Inventories 4,600

12,900

Total assets 52,500

Equity and liabilities
Equity shares of Rs. 1 each 21,000
Accumulated profit 14,000
Share premium 2,000

Total equity 37,000
Revaluation surplus 5,000
Current liabilities 5,300

Non-current liabilities
8% Debentures 2019 5,200

Total equity and liabilities 52,500

Emile Woolf International 4 The Institute of Chartered Accountants of Pakistan


Questions

Additional information
1 Income tax of Rs. 2.1 million has yet to be provided for on profits for the
current year. An unpaid under-provision for the previous years liability of Rs.
400,000 has been identified on 5th September 2015 and has not been
reflected in the draft accounts.
2 There have been no additions to, or disposals of, non-current assets in the
year but the assets under construction have been completed in the year at an
additional cost of Rs. 50,000. These related to plant and machinery.
The cost and accumulated depreciation of non-current assets as at 1st
September 2014 were as follows:
Cost Depreciation
Rs. in 000 Rs. in 000
Freehold land and buildings 19,000 3,000
(land element Rs. 10 million)
Plant and machinery 20,100 4,000
Fixtures and fittings 10,000 3,700
Assets under construction 400 -
3 There was a revaluation of land and buildings during the year, creating the
revaluation surplus of Rs. 5 million (land element Rs. 1 million). The effect on
depreciation has been to increase the buildings charge by Rs. 300,000. Barry
adopts a policy of transferring the revaluation surplus included in equity to
retained earnings as it is realised.
4 Staff costs comprise 70% factory staff, 20% general office staff and 10%
goods delivery staff
5 An analysis of depreciation charge shows the following:
Rs. in
000
Buildings (50% production, 50% administration) 1,000
Plant and machinery 2,550
Fixtures and fittings (30% production, 70% administration) 700

Required
Prepare the following information in a form suitable for publication for Barrys
financial statements for the year ended 31st August 2015.
Statement of profit or loss
Statement of financial position
Reconciliation of opening and closing property, plant and equipment (25)

Emile Woolf International 5 The Institute of Chartered Accountants of Pakistan


Financial accounting and reporting II

2.4 OSCAR INC


The following trial balance has been extracted from the books of accounts of Oscar
Inc as at 31 March 2015.
Rs. in 000
Dr Cr
Administrative expenses 210
Share capital 600
Receivables 470
Bank overdraft 80
Income tax (overprovision in 2014) 25
Provision 180
Distribution costs 420
Non-current investments 560
Investment income 75
Plant and machinery
At cost 750
Accumulated depreciation (at 31 March 2015) 220
Retained earnings (at 1 April 2014) 180
Purchases 960
Inventory (at 1 April 2014) 140
Trade payables 260
Sales revenue 2,010
Interim dividend paid 120
3,630 3,630
Additional information
(1) Inventory at 31 March 2015 was valued at Rs. 150,000.
(2) The income tax charge based on the profits on ordinary activities is estimated
to be Rs. 74,000.
(3) The provision is to be increased by Rs. 16,000.
(4) There were no purchases or disposals of fixed assets during the year.

Required
Prepare the companys statement of profit or loss for the year to 31 March 2015 and
a statement of financial position as at that date in accordance with IAS 1. (18)

Emile Woolf International 6 The Institute of Chartered Accountants of Pakistan


Questions

2.5 CLIFTON PHARMA LIMITED


The following trial balance relates to Clifton Pharma Limited, a public listed
company, at 30 September 2015.
Rs. in 000
Dr Cr
Cost of sales 134,000
Operating expenses 35,000
Loan interest paid (see note (1)) 1,500
Rental of vehicles (see note (2)) 8,600
Revenue 338,300
Investment income 2,000
Leasehold property at cost (see note (4)) 250,000
Plant and equipment at cost 197,000
Accumulated depreciation at 1 October 2014:
- leasehold property 40,000
- plant and equipment 47,000
Investments 92,400
Share capital 280,000
Share premium 20,000
Retained earnings at 1 October 2014 19,300
Loan notes (see note (1)) 50,000
Deferred tax balance at 1 October 2014 (see note (5)) 20,000
Inventory at 30 September 2015 23,700
Trade receivables 76,400
Trade payables 14,100
Bank 12,100
830,700 830,700
The following notes are relevant
(1) The effective interest rate on the loan notes is 6% per year.
(2) There are two separate contracts for rental of vehicles. A recent review by the
finance department of these contracts has reached the conclusion that Rs. 7
million of the total rental cost of vehicles relates to a finance lease rather than
an operating lease or rental arrangement.
The finance lease was entered into on 1 October 2014 which was when the
Rs. 7 million was paid: the lease agreement is for a four-year period in total,
and there will be three more annual payments in advance of Rs. 7 million,
payable on 1 October in each year. The vehicles in the finance lease
agreement had a fair value of Rs. 24 million at 1 October 2014 and they
should be depreciated using the straight line method to a nil residual value.
The interest rate implicit in the lease is 10% per year. The other contract for
vehicle rental is an operating lease and the rental payment should be charged
to operating expenses. (Note: You are not required to calculate the present
value of the minimum lease payments for the finance lease.)
(3) Other plant and equipment is depreciated at 20% per year by the reducing
balance method.
All depreciation of property, plant and equipment should be charged to cost of
sales.

Emile Woolf International 7 The Institute of Chartered Accountants of Pakistan


Financial accounting and reporting II

(4) The leasehold property has a 25-year life and is amortised at a straight-line
rate. On 30 September 2015 the leasehold property was re-valued to Rs. 220
million and the directors wish to incorporate this re-valuation in the financial
statements.
(5) The provision for income tax for the year ended 30 September 2015 has been
estimated at Rs. 18 million. At 30 September 2015 there are taxable
temporary differences of Rs. 92 million. The rate of income tax on profits is
25%.
Required
(a) Prepare an statement of profit or loss for Clifton Pharma Limited for the year to
30 September 2015 (8)
(b) Prepare a statement of financial position (balance sheet) for Clifton Pharma
Limited as at 30 September 2015 (17)
(25)

2.6 SARHAD SUGAR LIMITED


The following trial balance relates to Sarhad Sugar Limited at 30 September 2015:
Rs. in 000
Dr Cr
Leasehold property at valuation 1 October 2014 (note (i)) 50,000
Plant and equipment at cost (note (i)) 76,600
Plant and equipment accumulated depreciation at
1 October 2014 24,600
Capitalised development expenditure at 1 October 2014
(note (ii)) 20,000
Development expenditure accumulated amortisation at 1
October 2014 6,000
Closing inventory at 30 September 2015 20,000
Trade receivables 43,100
Bank 1,300
Trade payables and provisions (note (iii)) 23,800
Revenue (note (i)) 300,000
Cost of sales 204,000
Distribution costs 14,500
Administrative expenses (note (iii)) 22,200
Interest on bank borrowings 1,000
Equity dividend paid 6,000
Research and development costs (note (ii)) 8,600
Share capital 70,000
Retained earnings at 1 October 2014 24,500
Deferred tax (note (v)) 5,800
Revaluation surplus (Leasehold property) 10,000
466,000 466,000
The following notes are relevant:
(i) Non-current assets tangible:
The leasehold property had a remaining life of 20 years at 1 October 2014.
The companys policy is to revalue its property at each year end and at 30
September 2015 it was valued at Rs. 43 million.

Emile Woolf International 8 The Institute of Chartered Accountants of Pakistan


Questions

On 1 October 2014 an item of plant was disposed of for Rs. 25 million cash.
The proceeds have been treated as sales revenue by Sarhad Sugar Limited.
The plant is still included in the above trial balance figures at its cost of Rs. 8
million and accumulated depreciation of Rs. 4 million (to the date of disposal).
All plant is depreciated at 20% per annum using the reducing balance method.
Depreciation and amortisation of all non-current assets is charged to cost of
sales.
(ii) Non-current assets intangible:
In addition to the capitalised development expenditure (of Rs. 20 million),
further research and development costs were incurred on a new project which
commenced on 1 October 2014. The research stage of the new project lasted
until 31 December 2014 and incurred Rs. 14 million of costs. From that date
the project incurred development costs of Rs. 800,000 per month. On 1 April
2015 the directors became confident that the project would be successful and
yield a profit well in excess of its costs. The project is still in development at 30
September 2015.
Capitalised development expenditure is amortised at 20% per annum using
the straight-line method. All expensed research and development is charged
to cost of sales.
(iii) Sarhad Sugar Limited is being sued by a customer for Rs. 2 million for breach
of contract over a cancelled order. Sarhad Sugar Limited has obtained legal
opinion that there is a 20% chance that Sarhad Sugar Limited will lose the
case. Accordingly Sarhad Sugar Limited has provided Rs. 400,000 (Rs. 2
million x 20%) included in administrative expenses in respect of the claim. The
unrecoverable legal costs of defending the action are estimated at Rs.
100,000. These have not been provided for as the legal action will not go to
court until next year.
(iv) The directors have estimated the provision for income tax for the year ended
30 September 2015 at Rs. 114 million. The required deferred tax provision at
30 September 2015 is Rs. 6 million.

Required
(a) Prepare the statement of profit or loss for the year ended 30 September 2015.
(10)
(b) Prepare the statement of financial position as at 30 September 2015. (10)
Note: notes to the financial statements are not required. (20)

Emile Woolf International 9 The Institute of Chartered Accountants of Pakistan


Financial accounting and reporting II

2.7 BSZ LIMITED


The post-closing trial balance of BSZ Limited, a listed company, as at June 30,
2015 is given below:
Debit Credit
Rs. in million
Cash at banks current accounts 7
Cash at banks in saving accounts 22
Stocks in trade closing 90
Accounts receivable 60
Provision for bad debts 3
Advances to suppliers 16
Advances to staff 6
Short term deposits 11
Prepayments 4
Sales tax receivable 12
Freehold land at revalued amount 375
Furniture and fixtures - cost 27
Accumulated depreciation Furniture and fixtures 8
Machines - cost 85
Accumulated depreciation Machines 27
Building on freehold land cost 150
Accumulated depreciation Building 26
Computer software cost 10
Accumulated amortization Computer software 2
Deferred taxation 40
Short term loan 85
Accounts payable 75
Accrued liabilities 7
Provision for taxation 17
Issued, subscribed and paid up capital (Rs. 10 each) 400
Surplus on revaluation of fixed assets 120
Accumulated profits 65
875 875

Additional Information
(i) The first revaluation of freehold land was carried out in 2011 and resulted
in a surplus of Rs. 120 million. The valuation was carried out under market
value basis by an independent valuer, Mr. Dee, Chartered Civil Engineer of
M/s SSS Consultants (Pvt.) Ltd., Islamabad.
(ii) The details relating to additions, disposal and depreciation/amortization of
fixed assets, during the year 2015 are given below:
The company uses the straight line method for charging depreciation
and amortization. The building is depreciated at a rate of 5% whereas
10% is charged on machines, furniture and fixtures and computer
software.
Construction on third floor of the building commenced on March 1,
2015 and is expected to be completed on September 30, 2015. The
cost incurred during the year i.e. Rs. 20 million was capitalised on June
30, 2015.

Emile Woolf International 10 The Institute of Chartered Accountants of Pakistan


Questions

Furniture and fixtures worth Rs. 8 million were purchased on April 1,


2015.
A machine was sold on February 28, 2015 to NJ Enterprise at a price of
Rs. 13 million. At the time of disposal, the cost and written down value
of the machine was Rs. 15 million and Rs. 10 million respectively.
(iii) 50% of the accounts receivable were secured and considered good. 10%
of the unsecured accounts receivable were considered doubtful. Bad debts
expenses for the year amounted to Rs. 1.0 million. An amount of Rs. 1.4
million was written off during the year.
(iv) All advances given to suppliers are considered good and include an
amount of Rs. 4.0 million paid for goods which will be supplied on December
31, 2016.
(v) Cash at banks in saving accounts carry interest / mark-up ranging from 3%
to 7% per annum.
(vi) The authorised share capital of the company is Rs. 500 million.

Required
Prepare the statement of financial position as at June 30, 2015 along with the
relevant notes showing all possible disclosures as required under the International
Accounting Standards and the Companies Ordinance, 1984.
(Comparative figures and the note on accounting policies are not required.)(22)

Emile Woolf International 11 The Institute of Chartered Accountants of Pakistan


Financial accounting and reporting II

2.8 YASIR INDUSTRIES LIMITED


The following trial balance related to Yasir Industries Limited (YIL) for the year
ended June 30, 2015:
Dr Cr
Rs. in million
Ordinary share capital (Rs. 10 each) - 120.00
Retained earnings - 10.20
Sales - 472.40
Purchases 175.70 -
Production labour 61.00
Manufacturing overheads 39.00
Inventories (July 1, 2014) 38.90
Administrative expenses 40.00 -
Distribution expenses 19.80 -
Financial charges 0.30 -
Cash and bank - 13.25
Trade creditors - 30.40
Accrued expenses - 16.20
10% redeemable preference shares - 40.00
Debentures - 80.00
Deferred tax (July 1, 2014) - 6.00
Suspense account 30.00 -
Leasehold property - at cost 230.00 -
Machines at cost 168.60 -
Software at cost 20.00 -
Acc. depreciation Leasehold property (June 30, 2015) - 40.25
Acc. depreciation Machines (June 30, 2015) - 48.60
Acc. amortization Software (June 30, 2015) - 12.00
Trade receivables 66.00 -
889.30 889.30
Additional Information
(i) Sales include an amount of Rs. 27 million, made to a customer under sale or
return agreement. The sale has been made at cost plus 20% and the expiry
date for the return of these goods is July 31, 2015.
(ii) The value of inventories at June 30, 2015 was Rs. 42 million.
(iii) A fraud of Rs. 30 million was discovered in October 2014. A senior employee
of the company who left in June 2014, had embezzled the funds from YILs
bank account. The chances of recovery are remote. The amount is presently
appearing in the suspense account.
(iv) On January 1, 2015 YIL issued debenture certificates which are repayable in
2020. Interest is paid on these at 12% per annum.
(v) Financial charges comprise bank charges and bank commission.
(vi) The provision for current taxation for the year ended June 30, 2015 after
making all the above adjustments is estimated at Rs. 16.5 million.

Emile Woolf International 12 The Institute of Chartered Accountants of Pakistan


Questions

(vii) The carrying value of YILs net assets as on June 30, 2015 exceeds their tax
base by Rs. 30 million. The income tax rate applicable to the company is 30%.
(viii) On July 1, 2014, the leasehold property having a useful life of 40 years was
revalued at Rs. 238 million. No adjustment in this regard has been made in the
books.
(ix) Depreciation of leasehold property is charged using the straight line method.
50% of depreciation is allocated to manufacturing, 30% to administration and
20% to selling and distribution.
Required
In accordance with the requirements of the Companies Ordinance, 1984 and
International Accounting Standards, prepare the:
(a) statement of financial position as of June 30, 2015.
(b) statement of profit or loss for the year ended June 30, 2015. (20)
(Comparative figures and notes to the financial statements are not required.)

Emile Woolf International 13 The Institute of Chartered Accountants of Pakistan


Financial accounting and reporting II

2.9 SHAHEEN LIMITED


Following is the trial balance of Shaheen Limited (SL) as at June 30, 2015:
Rs. in 000
Dr Cr
Sales revenue 200,000
Manufacturing costs 100,000
Selling and distribution costs 35,000
Administrative costs 30,000
Opening inventories 23,000
Interest on borrowings 5,000
Provision for income tax 2,000
Advance income tax paid 6,000
Property, plant and equipment 86,000
Accumulated depreciation on property, plant
and equipment 12,000
Export licence 6,000
Trade receivables 37,800
Cash and bank balances 4,725
Other receivable and prepayments 14,000
Trade payables 12,000
Provisions for litigation 5,000
Long term borrowings 31,525
Deferred tax 5,000
Share capital (Rs. 10 each and fully paid) 60,000
Retained earnings 20,000
347,525 347,525
Additional information
(i) Sales last year (year ended 30 June 2014) included goods invoiced at Rs 10
million which were sent to a customer on June 25, 2014 under a sale or
return agreement, at cost plus 20%. The goods were returned on August 25,
2014. No correction has been made for the return.
(ii) The export licence has been obtained for exporting a new product and is
effective for five years up to December 31, 2019. However, the exports
commenced from July 1, 2015.
(iii) Closing inventories are valued at Rs. 30 million.
(iv) Details of property, plant and equipment are as follows:

Plant and
Land Buildings equipment
Rs in 000
Cost as at June 30, 2014 20,000 36,000 30,000
Fully depreciated amounts included in cost 3,000
Estimated useful life at the date of purchase 20 years 10 years
The company uses straight line method for charging depreciation.
Depreciation is allocated to manufacturing, distribution and administrative
costs at 75%, 15% and 10% respectively.
(v) Rs. 6 million of the long term borrowings is of current maturity (i.e. will be
repaid within 12 months).

Emile Woolf International 14 The Institute of Chartered Accountants of Pakistan


Questions

(vi) During the year Rs. 5 million was paid in full and final settlement of income
tax liability against which a provision of Rs. 7.0 million had been made in the
previous year. Current years taxable income exceeds accounting income by
Rs. 5 million of which 0.8 million are permanent differences. Applicable tax
rate for the company is 35%.
(vii) On July 30, 2015 the board of directors proposed a final dividend at 15% for
the year ended June 30, 2015 (2014: at 20%)

Required
In accordance with the requirements of the Companies Ordinance, 1984 and
International Financial Reporting Standards, prepare:
(a) The statement of financial position as of June 30, 2015
(b) The statement of profit or loss for the year ended June 30, 2015
(c) The statement of changes in equity for the year ended June 30, 2015.
(Comparative figures and notes to the financial statements are not required)
(25)

2.10 MOONLIGHT PAKISTAN LIMITED


Following is the summarised trial balance of Moonlight Pakistan Limited (MPL), a
listed company, for the year ended December 31, 2015:
Rs. in million
Debit Credit
Land and buildings - at cost 2,600 -
Plants at cost 2,104 -
Trade receivables 702 -
Stock in trade at December 31, 2015 758 -
Cash and bank 354 -
Cost of sales 1,784 -
Selling expenses 220 -
Administrative expenses 250 -
Financial charges 210 -
Accumulated depreciation as on January 1, 2015 Buildings - 400
Accumulated depreciation as on January 1, 2015 Plants - 670
Ordinary shares of Rs. 10 each fully paid - 1,200
Retained earnings as at January 1, 2015 - 510
12% Long term loan - 1,600
Provision for gratuity - 8
Deferred tax on January 1, 2015 - 22
Trade payables - 544
Right subscription received - 420
Revenue - 3,608
8,982 8,982

Emile Woolf International 15 The Institute of Chartered Accountants of Pakistan


Financial accounting and reporting II

Additional Information
(i) The land and buildings were acquired on January 1, 2011. The cost of
land was Rs. 600 million. On January 1, 2015 a professional valuation firm
valued the buildings at Rs. 1,840 million with no change in the value of land.
The estimated life at acquisition was 20 years and the remaining life has not
changed as a result of the valuation. 60% of depreciation on buildings is
allocated to manufacturing, 25% to selling and 15% to administration.
(ii) Plant is depreciated at 20% per annum using the reducing balance method.
(iii) On March 31, 2015 MPL made a bonus issue of one share for every six
held. The issue has not been recorded in the books of account.
(iv) Right shares were issued on September 1, 2015 at Rs. 12 per share.
(v) The interest on long term loan is payable on the first day of July and January.
No accrual has been made for the interest payable on January 1, 2013.
(vi) MPL operates an unfunded gratuity scheme for all its eligible employees.
The provision required as on December 31, 2015 is estimated at Rs. 23
million. Rs. 3 million were paid during the year and debited to the provision
for gratuity account. Cost of gratuity is allocated to production, selling and
administration expenses in the ratio of 60% : 20% : 20%.
(vii) The tax charge for the current year after making all related adjustments is
estimated at Rs. 37 million. The timing differences related to taxation are
estimated to increase by Rs. 80 million, over the last year. The applicable
income tax rate is 35%.

Required
In accordance with the requirements of Companies Ordinance, 1984 and
International Financial Reporting Standards, prepare the following:
(a) Statement of Financial Position as of December 31, 2015.
(b) Statement of profit or loss for the year ended December 31, 2015. (22)
(Comparative figures and notes to the financial statements are not required)

Emile Woolf International 16 The Institute of Chartered Accountants of Pakistan


Questions

2.11 FIGS PAKISTAN LIMITED


Figs Pakistan Limited is a listed company engaged in the business of manufacturing
and marketing of personal care and food products. Following is an extract from its
trial balance for the year ended 31 December 2015:
Debit Credit
Rs. in million
Sales - Manufactured goods 56,528
Sales - Imported goods 1,078
Scrap sales 16
Dividend income 12
Return on savings account 2
Sales tax - Imported goods 53
Sales tax - Manufactured goods 10,201
Sales discount 2,594
Raw material stock as on 1 January 2015 1,751
Work in process as on 1 January 2015 73
Finished goods (manufactured) as on 1 January 2015 1,210
Finished goods (imported) as on 1 January 2015 44
Purchases - Raw material 22,603
Purchases - Imported goods 658
Stores and spares consumed 180
Salaries, wages and benefits 2,367
Utilities 734
Depreciation and amortization 1,287
Stationery and office expenses 230
Repairs and maintenance 315
Advertisement and sales promotion 4,040
Outward freight and handling 1,279
Legal and professional charges 71
Auditor's remuneration 13
Donations 34
Workers Profit Participation Fund 257
Worker Welfare Fund 98
Loss on disposal of property, plant and equipment 10
Financial charges on short term borrowings 133
Exchange loss 22
Financial charges on lease 11
Additional information
(i) The position of inventories as at 31 December 2015 was as follows:
Rs. m
Raw material 2,125
Work in process 125
Finished goods (manufactured) 1,153
Finished goods (imported) 66

Emile Woolf International 17 The Institute of Chartered Accountants of Pakistan


Financial accounting and reporting II

(ii) The basis of allocation of various expenses among cost of sales, distribution
costs and administrative expenses are as follows:
Cost of Distribution Administrative
sales costs expenses
% % %
Salaries, wages and benefits 55 30 15
Depreciation and amortization 70 20 10
Stationery and office expenses 25 40 35
Repairs and maintenance / Utilities 85 5 10
(iii) Salaries, wages and benefits include contributions to provident fund (defined
contribution plan) and gratuity fund (defined benefit plan) amounting to Rs. 54
million and Rs. 44 million respectively.
(iv) Auditors remuneration includes taxation services and out-of-pocket expenses
amounting to Rs. 4 million and Rs. 1 million respectively.
(v) Donations include Rs. 5 million given to Dates Cancer Foundation (DCF). One
of the companys directors, Mr. Peanut is a trustee of DCF.
(vi) The tax charge for the current year after making all related adjustments is
estimated at Rs. 1,440 million. Taxable temporary differences of Rs. 3,120
originated in the year million, over the last year. The applicable income tax
rate is 35%.
(vii) 274 million ordinary shares were outstanding as on 31 December 2015.
(viii) There is no other comprehensive income for the year.

Required
Prepare the statement of profit or loss for the year ended 31 December 2015 along
with the relevant notes showing required disclosures as per the Companies
Ordinance, 1984 and International Financial Reporting Standards. Comparatives are
not required. (24)

Emile Woolf International 18 The Institute of Chartered Accountants of Pakistan


Questions

CHAPTER 3 IAS 7: STATEMENTS OF CASH FLOWS

3.1 KLEA
The statement of financial position and statement of profit or loss for Klea for the
year to 31st March 2015 are provided below.
Statement of financial position as at 31st March 2015
2015 2014
Rs. in 000
Assets
Non-current assets
Intangible assets 300 200
Property, plant and equipment 3,450 1,600
Financial assets 400 200

4,150 2,000

Current assets
Inventory 3,200 2,000
Trade receivables 2,400 2,000
Cash and cash equivalents 32 580

5,632 4,580

Total assets 9,782 6,580

Equity and liabilities
Equity
Issued share capital 3,000 2,000
Share premium account 838 560
Retained earnings 910 354

Total equity 4,748 2,914

Revaluation surplus 1,000 -
Non-current liabilities
Interest-bearing loans and liabilities 1,600 2,000
Current liabilities
Bank overdraft 414 -
Trade payables 1,600 1,266
Taxation 420 400

2,434 1,666

Total liabilities 4,034 3,666

Total equity and liabilities 9,782 6,580

Emile Woolf International 19 The Institute of Chartered Accountants of Pakistan


Financial accounting and reporting II

Statement of profit or loss for the year ended 31st March 2015
Rs. in 000
Revenue 10,000
Other income 100
Change in inventory of finished goods and WIP 1,300
Raw materials and consumables used 4,000
Employee benefits costs 3,000
Depreciation and amortisation expense 800
Other expenses 1,724

Total expenses (9,524)

1,876
Finance costs (320)
Finance income 50

Profit before tax 1,606
Income tax expense (650)

Profit for the year 956

Additional information
(i) Non-current assets Rs. in 000
2015 2014
Cost Deprecn Cost Deprecn
Intangible assets 700 400 400 200
Property, plant and equipment 5,000 1,550 3,000 1,400
(ii) At 1 April 2014 land was revalued from Rs. 1million to Rs. 2 million.
(iii) During the year, plant and machinery costing Rs. 600,000 and depreciated by
Rs. 500,000 was sold for Rs. 150,000.
(iv) The interest bearing loans relate to debentures which were issued at their
nominal value. Rs. 400,000 of these debentures were redeemed at par during
the year.
(v) Ordinary shares were issued for cash during the year.
(vi) Rs. 100,000 of current asset investments held as cash equivalents were sold
during the year for Rs. 94,000.
(vii) Dividends paid in the year were Rs. 200,000 relating to the 2014 proposed
dividend and a Rs. 200,000 interim dividend for 2015.
Required
Prepare a statement of cash flows for Klea for the year ended 31 March 2015 in
accordance with IAS 7 using the indirect method. (25)

Emile Woolf International 20 The Institute of Chartered Accountants of Pakistan


Questions

3.2 STANDARD INC


The summarised statements of financial position of Standard Inc at 31 December
2014 and 2015 are as follows.
2015 2014
Rs. in 000 Rs. in 000
Issued share capital 150,000 100,000
Share premium 35,000 15,000
Retained earnings 41,000 14,000
Long-term loans 30,000 70,000
Payables 48,000 34,000
Bank overdraft 14,000
Tax payable 33,000 21,500
Proposed dividends 15,000 7,500
Depreciation
Plant and machinery 54,000 45,000
Fixtures and fittings 15,000 13,000

421,000 334,000

Freehold property at cost 130,000 110,000
Plant and machinery at cost 151,000 120,000
Fixtures and fittings at cost 29,000 24,000
Inventories 51,000 37,000
Trade receivables 44,000 42,800
Long-term investments 4,600
Cash at bank 11,400 200

421,000 334,000

The following information is relevant:
(a) There had been no disposal of freehold property in the year.
(b) A machine tool which had cost Rs. 8,000,000 (in respect of which Rs.
6,000,000 depreciation had been provided) was sold for Rs. 3,000,000, and
fixtures which had cost Rs. 5,000,000 (in respect of which depreciation of Rs.
2,000,000 had been provided) were sold for Rs. 1,00,0000. Profits and losses
on those transactions had been dealt with through the statement of profit or
loss.
(c) The statement of profit or loss charge in respect of tax was Rs. 22,000,000.
(d) The premium paid on redemption of the long-term loan was Rs. 2,000,000,
which has been written off to the statement of profit or loss.
(e) The proposed dividend for 2014 had been paid during the year.
(f) Interest received during the year was Rs. 450,000. Interest charged in the
statement of profit or loss for the year was Rs. 6,400,000. Accrued interest of
Rs. 440,000 is included in payables at 31 December 2014 (nil at 31 December
2015).
(g) The government stock is a long term investment.
Required
Prepare a cash flow statement for the year ended 31 December 2015, together with
notes as required by IAS 7. (20)

Emile Woolf International 21 The Institute of Chartered Accountants of Pakistan


Financial accounting and reporting II

3.3 FALLEN
Fallen has prepared the following rough draft accounts for the year ended 31
December 2015.
Statement of profit or loss
Rs. in 000
Revenue 11,563
Cost of sales (5,502)

Gross profit 6,061
Distribution costs (402)
Administration expenses (882)
Interest payable (152)

Operating profit before tax 4,625
Taxation (35%) including deferred tax (1,531)

Profit after tax 3,094
Dividends (700)

Retained profit 2,394

Statements of financial position


31 December
2015 2014
Rs. in 000 Rs. in 000
Leasehold premises (net) 6,600 5,700
Plant, machinery and equipment (net) 5,040 3,780
Investments at cost 2,406 2,208
Inventories 2,880 1,986
Receivables 2,586 1,992
Bank 576

19,512 16,242

Emile Woolf International 22 The Institute of Chartered Accountants of Pakistan


Questions

31 December
2015 2014
Rs. in 000 Rs. in 000
Share capital 2,280 1,800
Share premium 2,112 1,800
Profit and loss account 9,108 6,714
Deferred taxation 202 138
Long-term loan (10%) 1,240 1,800
Provision for deferred repairs 1,202 1,016
Payables 1,026 702
Overdraft 222
Taxation
Corporation tax 1,730 2,038
Proposed dividends 390 234

19,512 16,242

The following data is relevant.
(1) The 10% long-term loan were redeemed at par.
(2) Plant and equipment with a written down value of Rs. 276,000 was sold for
Rs. 168,000. New plant was purchased for Rs. 2,500,000.
(3) Leasehold premises costing Rs. 1,300,000 were acquired during the year.
(4) The investments are highly liquid securities held for the short term.

Required
Prepare the cash flow statement and supporting notes in accordance with IAS 7 for
Fallen Inc for 2015. (20)

Emile Woolf International 23 The Institute of Chartered Accountants of Pakistan


Financial accounting and reporting II

3.4 BIN QASIM MOTORS LIMITED


The summarised financial statements of Bin Qasim Motors Limited for the year to 30
September 2015, together with a comparative balance sheet, are:
Statement of profit or loss Rs. 000
Sales revenue 7,482
Cost of sales (4,284)
Gross profit 3,198
Operating expenses (1,479)
Interest payable (260)
Investment income 120
Profit before tax 1,579
Income tax (520)
Profit for the period 1,059

Statement of financial position as at 30 September


2015 2014
Rs. in 000 Rs. in 000
Assets
Non-current assets
Property, plant and equipment 2,344 1,908
Investment 690 nil
3,034 1,908
Current assets
Inventory 1,046 785
Trade accounts receivable 935 824
Short term treasury bills 120 50
Bank nil 122
2,101 1,781
Total assets 5,135 3,689

Total equity and liabilities


Equity:
Share capital 1,400 1,000
Reserves:
Share premium 460 60
Retained earnings
At beginning of the year 192 147
Net profit for period 1,059 65
Dividends (180) (20)
At end of the year 1,071 192
2,931 1,252

Emile Woolf International 24 The Institute of Chartered Accountants of Pakistan


Questions

Revaluation surplus 90 40
Non-current liabilities
Deferred tax 439 400
Deferred income 275 200
10% Convertible loan stock nil 400
714 1,000
Current liabilities
Trade accounts payable 644 760
Accrued interest 40 25
Provision for negligence claim nil 120
Provision for income tax 480 367
Deferred income 100 125
Overdraft 136 nil
1,400 1,397
Total equity and liabilities 5,135 3,689
The following information is relevant
(i) Non-current assets
Property, plant and equipment is analysed as follows: Rs in 000
30 September 2015 30 September 2014
Cost/ Cost/
Valuation Depreciation NBV Valuation Depreciation NBV
Land and buildings 2,000 760 1,240 1,800 680 1,120
Plant 1,568 464 1,104 1,220 432 788
3,568 1,224 2,344 3,020 1,112 1,908

On 1 October 2014 Bin Qasim Motors Limited recorded an increase in the


value of its land of Rs. 150,000.
During the year an item of plant that had cost Rs. 500,000 and had
accumulated depreciation of Rs. 244,000 was sold at a loss (included in cost
of sales) of Rs. 86,000 on its carrying value.
(ii) Deferred income
Bin Qasim Motors Limited sells servicing contracts on certain types of
machinery. Payments are received in advance for a service which Bin Qasim
Motors Limited must provide over a number of following years. Income that
relates to these contracts is deferred and recognised in P&L as the period of
service passes.
A credit of Rs. 125,000 for the current years recognition of deferred income
has been included revenue in this period.
(iii) Share capital and loan stocks
The increase in the share capital during the year was due to the following
events:
(1) On 1 January 2015 there was a bonus issue (out of the revaluation
surplus) of one bonus share for every 10 shares held.

Emile Woolf International 25 The Institute of Chartered Accountants of Pakistan


Financial accounting and reporting II

(2) On 1 April 2015 the 10% convertible loan stock holders exercised their
right to convert to ordinary shares. The terms of conversion were 25
ordinary shares of Rs. 1 each for each Rs. 100 of 10% convertible loan
stock.
(3) The remaining increase in the ordinary shares was due to a stock
market placement of shares for cash on 12 August 2015.
(iv) Provision for negligence claim
In June 2015 Bin Qasim Motors Limited made an out of court settlement of a
negligence claim brought about by a former employee. The dispute had been
in progress for two years and Bin Qasim Motors Limited had made provisions
for the potential liability in each of the two previous years. The unprovided
amount of the claim at the time of settlement was Rs. 30,000 and this was
charged to operating expenses.

Required
Prepare a statement of cash flows for Bin Qasim Motors Limited for the year to 30
September 2015 in accordance with IAS 7 Statement of Cash Flows. (25)

Emile Woolf International 26 The Institute of Chartered Accountants of Pakistan


Questions

3.5 ITTEHAD MANUFACTURING LTD


The financial statements of Ittehad Manufacturing Ltd for the year to 30 September
2015, together with the comparative statement of financial position (balance sheet)
for the year to 30 September 2014 are shown below:
Rs. in million
Sales revenue 3,820
Cost of sales (note 1) (2,620)
Gross profit for period 1,200
Operating expenses (note 1) (280)
920
Interest Loan note (30)
Profit before tax 890
Taxation (270)
Net profit for the period 620

Statement of financial position as at 30 September:


2015 2016
Rs. in million Rs. in million
Non-current assets
Property, plant and equipment 1,890 1,830
Intangible assets (note 2) 670 300
2,560 2,130
Current assets
Inventory 1,420 940
Accounts receivable 990 680
Cash 70 nil
2,480 1,620
Total assets 5,040 3,750
Equity and liabilities
Ordinary shares of Rs. 1 each 750 500
Reserves
Share premium 350 100
Revaluation 190 nil
Retained earnings 1,860 1,600
3,150 2,200
Non-current liabilities (note 3) 610 240
Current liabilities (note 4) 1,280 1,310
Total equity and liabilities 5,040 3,750

Extract from statement of changes in equity


2015 2014
Rs. in million
Retained earnings brought forward 1,600 1,000
Profit for the year 620 800
Dividends (320) (200)
Bonus issue (50)
Transfer from revaluation surplus 10
Retained earnings carried forward 1,860 1,600

Emile Woolf International 27 The Institute of Chartered Accountants of Pakistan


Financial accounting and reporting II

Notes to the financial statements:


(1) Cost of sales includes depreciation of property, plant and equipment of Rs. 320
million and a loss on the sale of plant of Rs. 50 million.
2015 2014
Rs. in million
(2) Intangible non-current assets:
Deferred development expenditure 470 100
Goodwill 200 200
670 300
(3) Non-current liabilities:
10% loan note 300 100
Deferred tax 310 140
610 240
(4) Current liabilities:
Accounts payable 875 730
Bank overdraft nil 115
Accrued loan interest 15 5
Deferred income 260 300
Taxation 130 160
1,280 1,310

The following additional information is relevant:


(i) Intangible non-current assets:
The company successfully completed the development of a new product
during the current year, capitalising a further Rs. 500 million before
amortisation charges for the period.
(ii) Property, plant and equipment/revaluation surplus:
The company revalued its buildings by Rs. 200 million on 1 October
2014. The surplus was credited to a revaluation surplus.
New plant was acquired during the year at a cost of Rs. 250.
Rs. 10 million has been transferred from the revaluation surplus to
retained earnings as a year-end adjustment in respect of the additional
depreciation created by the revaluation.
The remaining movement on property, plant and equipment was due to
the disposal of obsolete plant.
(iii) Share issues:
On 1 October 2014 a bonus issue of 1 new share for every 10 held was made
from retained earnings. Ittehad Manufacturing Ltd made a further issue of
ordinary shares for cash during the year.
Required
(a) A statement of cash flows for Ittehad Manufacturing Ltd for the year to 30
September 2015 prepared in accordance with IAS 7 Statement of Cash Flows.
(20)
(b) Comment briefly on the financial position of Ittehad Manufacturing Ltd as
portrayed by the information in your statement of cash flows. (5)
(25)

Emile Woolf International 28 The Institute of Chartered Accountants of Pakistan


Questions

3.6 WASEEM INDUSTRIES LIMITED


The following statements of financial position relate to Waseem Industries Limited for
the years ended December 31:
2015 2014
Rs. in Rs. in
million million
ASSETS
Non-current assets
Fixed assets
Property, plant and equipment 242 182
Capital work-in-progress 20 18
262 200
Long term investments 75 100
Long term deposits 13 13
Total non-current assets 350 313

Current assets
Stocks-in-trade 55 48
Trade debts 51 38
Advances, prepayments and other
receivables 37 40
Cash and bank balances 11 20
Total current assets 154 146
TOTAL ASSETS 504 459

EQUITY AND LIABILITIES


Shareholders' equity
Share capital 150 125
Share premium 55 80
Unappropriated profit 85 50
290 255
Non-current liabilities
Long term finances - Secured 94 118
Deferred liability - Gratuity (unfunded) 16 12
110 130
Current liabilities
Current portion of long term finances 25 22
Short term finances 13 6
Trade and other payables 66 46
104 74
TOTAL EQUITY AND LIABILITIES 504 459

Emile Woolf International 29 The Institute of Chartered Accountants of Pakistan


Financial accounting and reporting II

Other relevant information is as follows:


(i) An interim bonus issue of one for five ordinary shares was made during the
year out of share premium. The company also approved final cash dividend of
10% (2014: 8%), in its annual general meeting.
(ii) During the year, the company provided Rs. 17 million (2014: Rs. 13 million)
on account of depreciation. The details relating to disposal of property, plant
and equipment are as follows:
Carrying amount Sale proceeds
Rs. m Rs. m
Plant and machinery 20 22
Vehicles 3 4

(iii) Advances, prepayments and other receivables include advance tax of Rs. 10
million (2014: Rs. 7 million).
(iv) In 2015, the company paid Rs. 6 million on account of gratuity.
(v) Accrued mark-up on long term finances amounting to Rs. 7 million (2014:
Rs. 9 million) is included in trade and other payables. Financial charges
included in the profit and loss account are Rs. 16 million (2014 : Rs. 14
million).
(vi) Income tax expense for the year 2015 amounted to Rs. 19 million (2014:
Rs. 13 million).

Required
Prepare a cash flow statement in accordance with the requirements of IAS 7 Cash
Flow Statement using the indirect method. (20)

Emile Woolf International 30 The Institute of Chartered Accountants of Pakistan


Questions

3.7 JALIB INDUSTRIES LIMITED


Jalib Industries Limited is a listed company. The relevant information contained
in the financial statements for the year ended December 31, 2015 is as follows:
Statement of Financial Position
2015 2014
Rupees in million
Non-current assets
Property, plant and equipment 129.40 100.60
Capital work in progress 22.50 37.00
151.90 137.60
Current assets
Stock in trade 531.80 451.00
Trade debts 28.50 24.70
Advances and other receivables 37.40 42.00
Cash and bank 12.00 3.00
609.70 520.70
761.60 658.30
Equity
Issued, subscribed and paid-up capital 396.00 300.00
Share premium 45.00 12.00
Unappropriated profit 142.60 163.00
583.60 475.00
Non-current liabilities
Deferred liabilities 40.80 27.50
Long term loans 80.00 100.00
120.80 127.50
Current liabilities
Current portion of long term loans 18.00 20.00
Creditors, accrued and other liabilities 36.20 34.40
Dividend payable 3.00 1.40
57.20 55.80
761.60 658.30

Statement of profit or loss 2015


Rupees in million
Sales 2,535.00
Cost of goods sold (1,774.50)
Gross profit 760.50
Operating expenses (554.00)
Financial charges (10.50)
Loss on sale of fixed assets (4.60)
(569.10)
Profit before tax 191.40
Tax expense - Current (104.60)
- Deferred (2.20)
(106.80)
Profit after tax 84.60

Emile Woolf International 31 The Institute of Chartered Accountants of Pakistan


Financial accounting and reporting II

The following supporting information is available:


(i) During the year, an amount of Rs. 42 million was transferred from capital
work in progress to property, plant and equipment.
(ii) The company sold property, plant and equipment having book value of Rs. 15
million for Rs. 10.4 million.
(iii) Depreciation for the year amounted to Rs. 27.7 million.
(iv) Trade debts written off during the year amounted to Rs. 1 million. It is the
policy of the company to maintain the provision for doubtful debts at 5% of
trade debts.
(v) Advances and other receivables include advance tax of Rs. 3.6 million (2014:
Rs. 2.2 million).
(vi) Deferred liabilities include deferred tax and provision for gratuity. There
was no deferred tax liability at the beginning of the year. Provision for gratuity
made during the year amounted to Rs. 15.5 million.
(vii) Creditors, accrued and other liabilities include accrued financial charges
amounting to Rs. 5 million (2014: Rs. 6 million).
(viii) On January 15, 2016, the company declared final dividend for the year ended
December 31, 2015 comprising 7.5% (2014: 25%) cash dividend and 12.5%
(2014:10%) bonus shares, for its ordinary shareholders.

Required
Prepare a statement of cash flow for the year ended December 31, 2015 in
accordance with the requirements of International Accounting Standards. Show all
necessary workings. (23)

3.8 APOLLO INDUSTRY LIMITED


Following are the relevant extracts from the financial statements of Apollo Industry
Limited, a listed company, for the year ended December 31, 2015.
Statement of financial position as at December 31, 2015

2015 2014
Rs. 000 Rs. 000
Issued, subscribed and paid up capital 25,000 20,000
Unappropriated profit 20,900 22,000
45,900 42,000

Surplus on revaluation of property, plant & equipment 7,000 8,000

Non-current liabilities
Staff gratuity 1,400 1,190
Deferred tax liability- net 590 -
1,990 1,190
Trade and other payables 4,200 6,250
59,090 57,440

Emile Woolf International 32 The Institute of Chartered Accountants of Pakistan


Questions

2015 2014
Rs. 000 Rs. 000

Property, plant and equipment 35,000 25,500


Capital work in progress 5,500 10,000
Intangible assets 1,100 1,140
41,600 36,640
Deferred tax asset- net - 350
Long term deposits and prepayments 400 300
42,000 37,290
Current Assets
Tax refundable 950 800
Other current assets 15,700 12,125
Cash and bank balances 440 7,225
17,090 20,150
59,090 57,440

Statement of comprehensiveiIncome for the year ended December 31, 2015


2015
Rs. 000
Sales 146,700
Cost of sales (127,500)
Gross profit 19,200

Operating expenses (15,000)


Financial charges (500)
Other income 2,800
(12,700)
Profit before tax 6,500
Tax expense - current (4,660)
- deferred (940)
Tax for the year (5,600)
Profit after tax 900
Other relevant information is as under:
(i) During the year, the company has issued 10% bonus shares.
(ii) Depreciation and amortization for the year amounted to Rs. 7 million.
(iii) WDV of assets disposed off during the year amounted to Rs. 1.2 million. (The
assets had not been revalued)
(iv) Other income includes interest earned on short term placements, amounting
to Rs. 1 million. The remaining amount represents gain on disposal of
property, plant and equipment.
(v) Gratuity of Rs. 0.3 million was paid to outgoing employees.
(vi) Intangible assets worth Rs. 50 thousand were acquired during the year.
Required
Prepare the Statement of Cash Flows for the year ended December 31, 2015 in
accordance with the requirements of IAS - 7 (Statement of Cash Flows) using
indirect method. (22)

Emile Woolf International 33 The Institute of Chartered Accountants of Pakistan


Financial accounting and reporting II

3.9 MARVEL ENGINEERING LIMITED


Following are the extracts from the draft financial statements of Marvel
Engineering Limited (MEL), a listed company, for the year ended 30 June 2015:
Statement of Financial Position
Rs. in million
2015 2014 2015 2014
Non current assets Share capital and
reserves
Property, plant and 633 410 Share capital 494 440
equipment (Rs. 10 each)
Long term 130 100 Share premium 8 -
investments
763 510 Retained earnings 133 110
Current assets 635 550
Stock-in-trade 97 68 Non current
liabilities
Trade debts 133 57 Long term loans 330 110
Other receivables 100 120 Gratuity payable 55 50
Cash at bank 31 39 Deferred taxation 15 21
361 284 400 181
Current liabilities
Trade and other 73 56
payables
Tax payable - net 12 5
Dividend payable 4 2
89 63
1,124 794 1,124 794

Statement of profit or loss


2015
Rs. in million
Revenue 654
Cost of sales (458)
Gross profit 196
Operating expenses (68)
Financial charges (75)
Other income 35
(108)
Profit before tax 88
Income tax expense (21)
Profit after tax 67
Additional information:
(i) During the year, the company recognised a provision for impairment in
respect of one of its plant, amounting to Rs. 11 million. Total depreciation for
the year amounted to Rs. 50 million.
(ii) It is the policy of the company to maintain a provision for doubtful debts at
5% of trade debts. During the year, trade debts amounting to Rs. 6 million
(2014: Rs. 2 million) were written off.
(iii) Trade and other payables include accrued financial charges amounting to
Rs. 7 million (2014: Rs. 3 million).

Emile Woolf International 34 The Institute of Chartered Accountants of Pakistan


Questions

(iv) On 15 July 2015, MELs board of directors proposed a final dividend of 10%
for the year ended 30 June 2015 (2014: 5% cash dividend and 5% bonus
declared on 20 July 2014).
(v) Other income comprises of the following:
Rs. m
Dividend income 30
Gain on sale of vehicles (carrying value of Rs. 5 million) 2
Gain on sale of investments (carrying value of Rs. 10 million) 3
35

(vi) Gratuity paid during the year amounted to Rs. 6 million.

Required
Prepare the statement of cash flows for Marvel Engineering Limited for the year
ended 30 June 2015. (24)

Emile Woolf International 35 The Institute of Chartered Accountants of Pakistan


Financial accounting and reporting II

CHAPTER 4 CONSOLIDATED ACCOUNTS: STATEMENTS OF FINANCIAL


POSITION BASIC APPROACH

4.1 HALL
Statements of financial position at 31 December 2015
Hall Stand
Rs. 000 Rs. 000
Assets
Non-current assets
Property, plant and equipment 35,000 20,000
Investment in Stand 12,000

Current assets 16,000 14,000



63,000 34,000

Equity and liabilities
Capital and reserves
Share capital 10,000 4,000
Retained earnings 13,000 12,000

23,000 16,000
Non-current liabilities
8% Debenture loans 20,000 9,000

Current liabilities 20,000 9,000



63,000 34,000

On 1 January 2013 Hall acquired 75% of Stand for Rs. 12,000,000. At that date the
balance on Stands retained earnings was Rs. 8,000,000.

Required
Prepare the consolidated statement of financial position of Hall as at 31 December
2015. (6)

Emile Woolf International 36 The Institute of Chartered Accountants of Pakistan


Questions

4.2 HASSLE
Statements of financial position at 31 December 2015
Hassle Strife
Rs. Rs.
Investment in Strife 60,000
Sundry assets 247,500 226,600

307,500 226,600

Share capital 120,000 50,000
Retained earnings 87,500 70,000
Liabilities 100,000 106,600

307,500 226,600

Hassle bought 80% of Strife when the balance on Strifes retained profit was Rs.
50,000.
Required
Prepare the consolidated statement of financial position at 31 December 2015. (8)

4.3 HYMN
The following are the summarised statements of financial position of a group of
companies as at 31 December 2015.
Hymn Psalm
Rs. Rs.
Assets
Non-current assets
Property, plant and equipment 105,000 65,000
Investment 85,000
Current assets 220,000 55,000

410,000 120,000

Equity and liabilities
Equity
Share capital 100,000 50,000
Retained earnings 155,000 49,000

255,000 99,000
Current liabilities 155,000 21,000

410,000 120,000

Hymn purchased 80% of Psalms shares on 1 January 2015 when there was a
credit balance on that companys retained earnings of Rs. 20,000.

Required
Prepare the Hymn group consolidated statement of financial position as at 31
December 2015. (6)

Emile Woolf International 37 The Institute of Chartered Accountants of Pakistan


Financial accounting and reporting II

4.4 HANG
On 31 December 2012, Hang acquired 60% of Swing for Rs. 140,000. At that date
Swing had a retained earnings balance of Rs. 50,000 and a share premium account
balance of Rs. 49,000.
The following statements of financial position have been prepared as at 31
December 2015.
Hang Swing
Rs. Rs.
Assets
Non-current assets
Property, plant and equipment 240,000 180,000
Investment in Swing 140,000
Current assets 250,000 196,000

630,000 376,000

Equity and liabilities
Equity
Share capital 200,000 90,000
Share premium 25,000 49,000
Retained earnings 180,000 80,000

405,000 219,000
Current liabilities 225,000 157,000

630,000 376,000

Required
Prepare the consolidated statement of financial position of Hang and its subsidiary
as at 31 December 2015. (6)

4.5 HASH
Statements of financial position at 31 December 2015
Hash Stash
Rs. 000 Rs. 000
Investment in Stash (80%) 100,000
Sundry assets 207,500 226,600

307,500 226,600

Share capital 120,000 50,000


Retained earnings 87,500 70,000
Liabilities 100,000 106,600

307,500 226,600

Hash purchased the shares in Stash on 30th September 2015.
Stashs retained profit for the year ended 31st December 2015 was Rs. 24,000,000.

Required
Prepare the consolidated statement of financial position at 31 December 2015. (8)

Emile Woolf International 38 The Institute of Chartered Accountants of Pakistan


Questions

CHAPTER 5 CONSOLIDATED ACCOUNTS: STATEMENTS OF FINANCIAL


POSITION COMPLICATIONS

5.1 HAIL
The following are the draft statements of financial position of Hail and its subsidiary
Snow as at 31 December 2015.
Hash Stash
Rs. 000 Rs. 000
Assets
Non-current assets
Property, plant and equipment 161,000 85,000
Investments 68,000
Current assets
Cash 7,700 25,200
Trade receivables 92,500 45,800
Snow current account 15,000 -
Inventory 56,200 36,200

400,400 192,200

Equity and liabilities
Shareholders equity
Share capital 100,000 50,000
Retained earnings 185,400 41,200
Share premium - 5,000
Capital reserve - 20,000

285,400 116,200
Current liabilities 115,000 68,000
Hail current account - 8,000

400,400 192,200

Notes
(1) Snow has 50,000 shares in issues. Hail acquired 45,000 of these on 1
January 2012 for a cost of Rs. 65,000,000 when the balances on Snows
reserves were
Rs. 000
Share premium account 5,000
Capital reserve
Retained earnings 10,000

(2) Hail declared a dividend of Rs. 3,000,000 before the year end and Snow
declared one of Rs. 2,000,000. These transactions have not been accounted
for.
(3) The current account difference is due to cash in transit.

Required
Prepare the consolidated statement of financial position as at 31 December 2015 of
Hail. (12)

Emile Woolf International 39 The Institute of Chartered Accountants of Pakistan


Financial accounting and reporting II

5.2 HAIRY
The summarised statements of financial position of Hairy and Spider as at 31
December 2015 were as follows.

Hairy Spider
Rs. 000 Rs. 000
Assets
Non-current assets
Property, plant and equipment 120,000 60,000
Investments 55,000

Current assets
Cash 11,000 4,000
Investments 3,000
Trade receivables 72,600 19,100
Current account Hairy 3,200
Inventory 17,000 11,000

275,600 100,300

Equity and liabilities


Share capital 100,000 60,000
Share premium 20,000
Capital reserve 23,000 16,000
Retained earnings 91,900 7,300
Trade payables 38,000 17,000
Current account Spider 2,700

275,600 100,300

The following information is relevant.


(1) On 31 December 2012, Hairy acquired 48,000 shares in Spider for Rs.
55,000,000 cash. Spider has 60,000 shares in total.
(2) The inventory of Hairy includes Rs. 4,000,000 goods from Spider invoiced to
Hairy at cost plus 25%.
(3) The difference on the current account balances is due to cash in transit.
(4) The balance on Spiders retained earnings was Rs. 2,300,000 at the date of
acquisition. There has been no movement in the balance on Spiders capital
reserve since the date of acquisition.

Required
Prepare the consolidated statement of financial position of Hairy and its subsidiary
Spider as at 31 December 2015. (12)

Emile Woolf International 40 The Institute of Chartered Accountants of Pakistan


Questions

5.3 HARD
On 31 December 2011, Hard acquired 60% of the ordinary share capital of Soft for
Rs. 110 million. At that date Soft had a retained earnings balance of Rs. 50 million
and a share premium account balance of Rs. 10 million.
The following statements of financial position have been prepared as at 31
December 2015.
Hard Soft
Rs. 000 Rs. 000
Assets
Non-current assets
Property, plant and equipment 225,000 175,000
Investments in Soft 110,000

Current assets 271,000 157,000



606,000 332,000

Equity and liabilities
Capital and reserves
Share capital 100,000 100,000
Share premium 15,000 10,000
Retained earnings 260,000 80,000

375,000 190,000
Current liabilities 231,000 142,000

606,000 332,000

During the year to 31 December 2015 Hard sold a tangible asset to Soft for Rs. 50
million. The asset was originally purchased in the year to 31 December 2012 at a
cost of Rs. 100 million and had a useful economic life of five years.
Softs depreciation policy is 25% per annum based on cost. Both companies charge
a full years depreciation in the year of acquisition and none in the year of disposal.

Required
Prepare the consolidated statement of financial position of Hard and its subsidiary
as at 31 December 2015. (12)

Emile Woolf International 41 The Institute of Chartered Accountants of Pakistan


Financial accounting and reporting II

5.4 HALE
On 1 July 2012 Hale acquired 128,000 of Sowens 160,000 shares. The following
statements of financial position have been prepared as at 31 December 2015.
Hale Sowen
Rs. 000 Rs. 000
Property, plant and equipment 152,000 129,600
Investment in Sowen 203,000
Inventory at cost 112,000 74,400
Receivables 104,000 84,000
Bank balance 41,000 8,000

612,000 296,000

Hale Sowen
Rs. 000 Rs. 000
Share capital 100,000 160,000
Retained earnings 460,000 112,000
Payables 52,000 24,000

612,000 296,000

The following information is available.
(1) At 1 July 2012 Sowen had a debit balance of Rs. 11 million on retained
earnings.
(2) Property, plant and equipment of Sowen included land at a cost of Rs. 72
million. This land had a fair value of Rs. 100,000 at the date of acquisition.
(3) The inventory of Sowen includes goods purchased from Hale for Rs. 16
million. Hale invoiced those goods at cost plus 25%.

Required
Prepare the consolidated statement of financial position of Hale as at 31 December
2015. (12)

5.5 HELLO
On 1 January 2012, Hello acquired 60% of the ordinary share capital of Solong for
Rs. 110,000. At that date Solong had a retained earnings balance of Rs. 60,000.
The following statements of financial position have been prepared as at 31
December 2015.
Hello Solong
Rs. Rs.
Assets
Non-current assets
Property, plant and equipment 225,000 175,000
Investments in Solong 110,000

Current assets 271,000 157,000



606,000 332,000

Emile Woolf International 42 The Institute of Chartered Accountants of Pakistan


Questions

Equity and liabilities


Capital and reserves
Share capital 100,000 100,000
Retained earnings 275,000 90,000

375,000 190,000
Current liabilities 231,000 142,000

606,000 332,000

The fair value of Solongs net assets at the date of acquisition was determined to be
Rs. 170,000.
The difference between the book value and the fair value of the new assets at the
date of acquisition was due to an item of plant which had a useful life of 10 years
from the date of acquisition.

Required
Prepare the consolidated statement of financial position of Hello and its subsidiary
as at 31 December 2015. (12)

5.6 HASAN LIMITED


On 1 April 2014, Hasan Limited acquired 90% of the equity shares in Shakeel
Limited. On the same day Hasan Limited accepted a 10% loan note from Shakeel
Limited for Rs. 200,000 which was repayable at Rs. 40,000 per annum (on 31 March
each year) over the next five years. Shakeel Limiteds retained earnings at the date
of acquisition were Rs. 2,200,000.

Statements of financial position as at 31 March 2015


Hasan Shakeel
Limited Limited
Rs. 000 Rs. 000
Non-current assets
Property, plant and equipment 2,120 1,990
Intangible software 1,800
Investments equity in Shakeel Limited 4,110
Investments 10% loan note Shakeel 200
Limited
Investments others 65 210
6,495 4,000
Current assets
Inventories 719 560
Trade receivables 524 328
Shakeel Limited current account 75
Cash 20
1,338 888
Total assets 7,833 4,888
Equity and liabilities:
Capital and reserves
Equity shares of Rs. 1 each 2,000 1,500
Share premium 2,000 500

Emile Woolf International 43 The Institute of Chartered Accountants of Pakistan


Financial accounting and reporting II

Retained earnings 2,900 1,955


6,900 3,955
Non-current liabilities
10% Loan note from Hasan Limited 160
Government grant 230 40
230 200
Current liabilities
Trade payables 475 472
Hasan Limited current account 60
Income taxes payable 228 174
Operating overdraft 27
703 733
Total equity and liabilities 7,833 4,888
The following information is relevant:
(i) Included in Shakeel Limiteds property at the date of acquisition was a
leasehold property recorded at its depreciated historical cost of Rs. 400,000.
The leasehold had been sub-let for its remaining life of only four years at an
annual rental of Rs. 80,000 payable in advance on 1 April each year. The
directors of Hasan Limited are of the opinion that the fair value of this
leasehold is best reflected by the present value of its future cash flows. An
appropriate cost of capital for the group is 10% per annum.
The present value of a Rs. 1 annuity received at the end of each year where
interest rates are 10% can be taken as:
3 year annuity Rs. 2.50
4 year annuity Rs. 3.20
(ii) The software of Shakeel Limited represents the depreciated cost of the
development of an integrated business accounting package. It was completed
at a capitalised cost of Rs. 2,400,000 and went on sale on 1 April 2013.
Shakeel Limiteds directors are depreciating the software on a straight-line
basis over an eight-year life (i.e. Rs. 300,000 per annum). However, the
directors of Hasan Limited are of the opinion that a five-year life would be
more appropriate as sales of business software rarely exceed this period.
(iii) The inventory of Hasan Limited on 31 March 2015 contains goods at a
transfer price of Rs. 25,000 that were supplied by Shakeel Limited who had
marked them up with a profit of 25% on cost. Unrealised profits are adjusted
for against the profit of the company that made them.
(iv) On 31 March 2015 Shakeel Limited remitted to Hasan Limited a cash payment
of Rs. 55,000. This was not received by Hasan Limited until early April. It was
made up of an annual repayment of the 10% loan note of Rs. 40,000 (the
interest had already been paid) and Rs. 15,000 of the current account
balance.
(v) The accounting policy of Hasan Limited for non-controlling interests (NCI) in a
subsidiary is to value NCI at a proportionate share of the net assets.
(v) An impairment test at 31 March 2015 on the consolidated goodwill concluded
that it should be written down by Rs. 120,000. No other assets were impaired.
Required:
Prepare the consolidated statement of financial position of Hasan Limited as at
31 March 2015.
(Total: 25 marks)

Emile Woolf International 44 The Institute of Chartered Accountants of Pakistan


Questions

CHAPTER 6 CONSOLIDATED ACCOUNTS: STATEMENTS OF


COMPREHENSIVE INCOME

6.1 HARRY
The following are the statements of profit or loss for the year ended 31 December
2015 of Harry and its subsidiary Sally.
Harry Sally
Rs. 000 Rs. 000
Revenue 1,120 390
Cost of sales (610) (220)
Gross profit 510 170
Distribution costs (50) (40)
Administration costs (55) (45)
Operating profit 405 85
Investment income 20 4
Finance costs (18) (4)
Profit before tax 407 85
Income tax expense (140) (25)
Profit for the year 267 60

Rs. 000 Rs. 000


Retained profit brought forward 100 45
Profit for year 267 60
Dividends paid and proposed (50) (20)
Retained profit carried forward 317 85
The following information is relevant.
(1) Harry acquired 75% of Sally six years ago when Sallys retained earnings
were Rs. 9,000.
(2) Harry made sales to Sally totalling Rs. 100,000 in the year. At the year end the
statement of financial position of Sally included inventory purchased from
Harry. Harry had taken a profit of Rs. 3,000 on this inventory.
(3) Harrys investment income includes Rs. 15,000 being its share of Sallys
dividends.

Required
Prepare a consolidated statement of profit or loss and a working showing the
movement on consolidated retained profit for the year ended 31 December 2015.(10)

Emile Woolf International 45 The Institute of Chartered Accountants of Pakistan


Financial accounting and reporting II

6.2 HORNY
Statements of profit or loss for the year ended 31 December 2015.
Horny Smooth
Rs. 000 Rs. 000
Revenue 304,900 195,300
Cost of sales (144,200) (98,550)
Gross profit 160,700 96,750
Operating costs (76,450) (52,100)
Operating profit 84,250 44,650
Investment income 10,500 2,600
Profit before tax 94,750 47,250
Income tax expense(42,900) (16,500)
Profit for the year 51,850 30,750
Statement of changes in equity (extracts) for the year ended 31 December 2015.
Horny Smooth
Rs. 000 Rs. 000
Retained earnings brought forward 80,200 31,000
Profit for the year 51,850 30,750
Proposed ordinary dividend (20,000) -
112,050 61,750
The following information is also available.
(1) Horny acquired 75% of the share capital of Smooth on 31 August 2015.
(2) Negative goodwill of Rs. 3.8 million arose on the acquisition.
(3) Profits of both companies are deemed to accrue evenly over the year except
for the investment income of Smooth all of which was received in November
2015.
(4) Horny has bought goods from Smooth throughout the year at Rs. 2 million per
month. At the year-end Horny does not hold any inventory purchased from
Smooth.

Required
Prepare the consolidated statement of profit or loss and a working showing the
movement on consolidated retained profit for the year ended 31 December 2015.(10)

Emile Woolf International 46 The Institute of Chartered Accountants of Pakistan


Questions

6.3 HERON
Statements of financial position as at 30 June 2015
Heron Stork
Assets Rs. 000 Rs. 000
Non-current assets
Property, plant and equipment 31,000 15,000
Investment in Stork ( 1,000 ordinary shares) 1,000
32,000 15,000
Current assets 23,000 11,000
55,000 26,000
Shareholders equity and liabilities
Share capital (Rs. 0001 ordinary shares) 10,000 1,500
Share premium 5,000
Retained earnings 20,000 18,500
35,000 20,000
Non-current liabilities 15,000
Current liabilities 5,000 6,000
55,000 26,000
Heron acquired its shares in Stork when the balance on the retained earnings was
Rs. 000nil.
Statements of profit or loss for the year ended 30 June 2015
Heron Stork
Rs. 000 Rs. 000
Revenue 30,000 25,000
Cost of sales (9,000) (10,000)

Gross profit 21,000 15,000


Distribution costs (3,000) (1,200)
Administrative expenses (1,000) (2,800)
Finance costs (2,000)

Profit before tax 15,000 11,000


Income tax expense (3,000) (3,000)

Profit for the period 12,000 8,000



Statement of changes in equity for the year ended 30 June 2015 (extract)
Retained earnings brought forward 8,000 10,500
Profit for the financial year 12,000 8,000

Retained earnings carried forward 20,000 18,500

Required
Prepare Herons consolidated statement of profit or loss, consolidated statement of
financial position and a working showing the movement on consolidated retained
profit for Heron for the year ended 30 June 2015. (12)

Emile Woolf International 47 The Institute of Chartered Accountants of Pakistan


Financial accounting and reporting II

6.4 HANKS
Statements of financial position as at 31 December 2015
Hanks Streep Scott
Rs. 000 Rs. 000 Rs. 000
Assets
Non-current assets
Property, plant and equipment 32,000 25,000 20,000
Investments 33,500

65,500 25,000 20,000
Current assets
Cash at bank and in hand 9,500 2,000 4,000
Trade receivables 20,000 8,000 17,000
Inventory 30,000 18,000 18,000

125,000 53,000 59,000

Equity and liabilities
Share capital 40,000 10,000 15,000
Share premium account 6,500
Retained earnings 55,000 37,000 27,000

101,500 47,000 42,000
Current liabilities 23,500 6,000 17,000

125,000 53,000 59,000

Statements of profit or loss for the year ended 31 December 2015
Hanks Streep Scott
Rs. 000 Rs. 000 Rs. 000
Revenue 125,000 117,000 82,000
Cost of sales (65,000) (64,000) (42,000)

Gross profit 60,000 53,000 40,000
Distribution costs (21,000) (14,000) (16,000)
Administrative expenses (14,000) (8,000) (7,000)

Profit before taxation 25,000 31,000 17,000
Income tax expense (10,000) (9,000) (5,000)

Profit after tax 15,000 22,000 12,000

Statement of changes in equity (extract) for the year ending
31 December 2015
Hanks Streep Scott
Rs. 000 Rs. 000 Rs. 000
Retained earnings brought forward 40,000 15,000 15,000
Retained profit for the financial year 15,000 22,000 12,000
Dividends

Retained earnings carried forward 55,000 37,000 27,000

Emile Woolf International 48 The Institute of Chartered Accountants of Pakistan


Questions

You are given the following additional information


(1) Hanks owns 80% of Streeps shares. These were purchased in 2012 for Rs.
20.5 million cash, when the balance on Streeps retained earnings stood at
Rs. 7million.
(2) In 2010 Hanks purchased 60% of the shares of Scott by the issue of shares
with a nominal value of Rs. 6.5 million. These shares were issued at a
premium of Rs. 6.5 million. At that date the retained earnings of Scott stood at
Rs. 3 million and the fair value of the net assets of Scott was Rs. 24 million. It
was agreed that any undervaluation of the net assets should be attributed to
land. This land was still held at 31 December 2015.
(3) Included in the inventory of Scott and Streep at 31 December 2015 are goods
purchased from Hanks for Rs. 5.2 million and Rs. 3.9 million respectively.
Hanks aims to earn a profit of 30% on cost. Total sales from Hanks to Scott
and to Streep were Rs. 8 million and Rs. 6 million respectively.
(4) Hanks and Streep each proposed a dividend before the year end of Rs. 2
million and Rs. 2.5 million respectively. No accounting entries have yet been
made for these.
(5) Hanks has carried out annual impairment tests on goodwill in accordance with
IFRS 3 and IAS 36. The estimated recoverable amount of goodwill at 31
December 2012 was Rs. 5 million and at 31 December 2015 was Rs. 4.5
million.

Required
Prepare the consolidated statement of profit or loss and consolidated statement of
changes in equity for the year ended 31 December 2015 and the consolidated
statement of financial position at that date. (20)

Emile Woolf International 49 The Institute of Chartered Accountants of Pakistan


Financial accounting and reporting II

CHAPTER 7 TANGIBLE NON-CURRENT ASSETS (IAS 16: PROPERTY,


PLANT AND EQUIPMENT AND IAS 23: BORROWING COSTS)

7.1 ROONEY
(a) Rooney has recently finished building a new item of plant for its own use. The
item is a press for use in the manufacture of industrial diamonds. Rooney
commenced construction of the asset on 1st April 2013 and completed it on 1st
April 2015.

1st January 2013, Rooney took out a loan to finance the construction of the
asset. Interest is charged on the loan at the rate of 5% per annum. The annual
interest must be paid in four equal instalments at the end of each quarter.
Rooney capitalises interest on manufactured assets in accordance with the
rules in IAS 23 Borrowing costs.

The costs (excluding finance costs) of manufacturing the asset were Rs. 28
million.
Required
State the IAS 23 rules on the capitalisation of borrowing costs, calculate the
cost of the asset on initial recognition and explain the amount of borrowing
cost capitalised. (6)

(b) The press comprises two significant parts, the hydraulic system and the
frame. The hydraulic system has a three year life and the frame has an eight
year life. Rooney depreciates plant on a straight line basis. The cost of the
hydraulic system is 30% of the total cost of manufacture.

Rooney uses the IAS 16 revaluation model in accounting for diamond presses
and revalues these assets on an annual basis.

Revaluation surpluses or deficits are apportioned between the hydraulic


system and the frame on the basis of their year end book values before the
revaluation.
Required
Explain the IAS 16 rules on accounting for significant parts of property, plant
and equipment and show the accounting treatment of the diamond press in
the financial statements for the financial years ending:
(i) 31st March 2016 (assume that the press has a fair value of Rs. 21
million)
(ii) 31st March 2017 (assume that the press has a fair value of Rs. 19.6
million). (13)
(25)

Emile Woolf International 50 The Institute of Chartered Accountants of Pakistan


Questions

7.2 EHTISHAM
The following information relates to the financial statements of Ehtisham for the year
to 31 March 2015.
The head office of Ehtisham was acquired on 1 April 2012 for Rs. 1million. Ehtisham
intend to occupy the building for 25 years. On 31 March 2014 it was revalued to Rs.
1.15 million. On 31 March 2015, a surplus of vacant commercial property in the area
had led to a fall in property prices and the fair value was now only Rs. 0.8 million.
Required
Explain the correct accounting treatment for the above (with calculations if
appropriate). (10)

7.3 CARLY
The following is an extract from the financial statements of Carly on 31 December
2014.
Property, plant and equipment
Land and Plant and
buildings equipment Computers Total
Rs. Rs. Rs. Rs.
Cost
On 31 December 2014 1,500,000 340,500 617,800 2,458,300

Accumulated depreciation
On 31 December 2014 600,000 125,900 505,800 1,231,700

Carrying amount
On 31 December 2014 900,000 214,600 112,000 1,226,600

Accounting policies
Depreciation
Depreciation is provided at the following rates.

On land and buildings 2% per annum straight line on buildings only

On plant and equipment 25% reducing balance

On computers 33.33% per annum straight line

During 2015 the following transactions took place.


(1) On 31 December the land and buildings were revalued to Rs. 1,750,000. Of
this amount, Rs. 650,000 related to the land (which had originally cost Rs.
500,000). The remaining useful life of the buildings was assessed as 40 years.
(2) A machine which had cost Rs. 80,000 and had accumulated depreciation of
Rs. 57,000 at the start of the year was sold for Rs. 25,000 in the first week of
the year.

Emile Woolf International 51 The Institute of Chartered Accountants of Pakistan


Financial accounting and reporting II

(3) A new machine was purchased on 31 March 2015. The following costs were
incurred:
Rs.
Purchase price, before discount, inclusive of reclaimable
sales tax of Rs. 3,000 20,000
Discount 1,000
Delivery costs 500
Installation costs 750
Interest on loan taken out to finance the purchase 300

(4) On 1 January it was decided to change the method of providing depreciation


on computer equipment from the existing method to 40% reducing balance.
Required
Produce the analysis of property, plant and equipment as it would appear in the
financial statements of Carly for the year ended 31 December 2015.

7.4 ADJUSTMENTS LIMITED


Adjustments Limited has carried out a review of its non-current assets.
(a) A lathe was purchased on 1 January 2009 for Rs. 150,000. The plant had an
estimated useful life of twelve years, residual value of nil. Depreciation is
charged on the straight line basis. On 1 January 2015, when the assets net
book value is Rs. 75,000, the directors decide that the assets total useful life
is only ten years.
(b) A grinder was purchased on 1 January 2012 for Rs. 100,000. The plant had
an estimated useful life of ten years and a residual value of nil. Depreciation is
charged on the straight line basis. On 1 January 2015, when the assets net
book value is Rs. 70,000, the directors decide that it would be more
appropriate to depreciate this asset using the sum of digits approach. The
remaining useful life is unchanged.
(c) The company purchased a fifty year lease some years ago for Rs. 1,000,000.
This was being depreciated over its life on a straight line basis. On 1 January
2015, when the net book value is Rs. 480,000 and twenty-four years of the
lease are remaining, the asset is revalued to Rs. 1,500,000. This revised value
is being incorporated into the accounts.

Required
Explain the effects of these changes on the depreciation for the year to 31
December 2015. (15)

Emile Woolf International 52 The Institute of Chartered Accountants of Pakistan


Questions

7.5 FAM
Fam had the following tangible fixed assets at 31 December 2014.
Cost Depreciation NBV
Rs. 000 Rs. 000 Rs. 000
Land 500 500
Buildings 400 80 320
Plant and machinery 1,613 458 1,155
Fixtures and fittings 390 140 250
Assets under construction 91 91

2,994 678 2,316

In the year ended 31 December 2015 the following transactions occur.
(1) Further costs of Rs. 53,000 are incurred on buildings being constructed by the
company. A building costing Rs. 100,000 is completed during the year.
(2) A deposit of Rs. 20,000 is paid for a new computer system which is
undelivered at the year end.
(3) Additions to plant are Rs. 154,000.
(4) Additions to fixtures, excluding the deposit on the new computer system, are
Rs. 40,000.
(5) The following assets are sold.
Cost Depreciation Proceeds
brought forward
Rs. 000 Rs. 000 Rs. 000
Plant 277 195 86
Fixtures 41 31 2
(6) Land and buildings were revalued at 1 January 2015 to Rs. 1,500,000, of
which land is worth Rs. 900,000. The revaluation was performed by Messrs
Jackson & Co, Chartered Surveyors, on the basis of existing use value on the
open market.
(7) The useful economic life of the buildings is unchanged. The buildings were
purchased ten years before the revaluation.
(8) Depreciation is provided on all assets in use at the year end at the following
rates.
Buildings 2% per annum straight line
Plant 20% per annum straight line
Fixtures 25% per annum reducing balance
Required
Show the disclosure under IAS 16 in relation to fixed assets in the notes to the
published accounts for the year ended 31 December 2015. (14)

Emile Woolf International 53 The Institute of Chartered Accountants of Pakistan


Financial accounting and reporting II

7.6 IMRAN LIMITED


On January 1, 2015, Imran Limited started the construction of its new factory.
The construction period is approximately 15 months and the cost is estimated
at Rs. 80 million. The work has been divided into 5 phases and payment to
contractor shall be made on completion of each phase.
In the year the company had the following sources of finance available.
(i) Rights i s s u e o f shares amounting to Rs. 15 million on January 1, 2015.
The company usually pays a dividend of 10% each year.
(ii) Bank loan of Rs. 32 million carrying a mark-up of 13% was raised on March
1, 2015. (This loan was outstanding for 306 days in the year).
(iii) On August 1, 2015, Rs. 10 million were borrowed from the bank. Interest
thereon, is payable at the rate of 11%. (This loan was outstanding for 153 days
in the year).
Investment income on temporary investment of the borrowings amounted to Rs. 0.5
million.
The details of bills submitted by the contractor, during the year are as follows:
Particulars Date of payment Rupees
On completion of 1st phase March 1, 2015 20,000,000
nd
On completion of 2 phase April 1, 2015 18,000,000
rd
On completion of 3 phase October 1, 2015 16,000,000
On completion of 4th phase Payment not yet made 17,000,000
On June 1, 2015, the Building Control Authority issued instructions for stoppage of
work on account of certain discrepancies in the completion plan. The company filed
a petition in the Court and the matter was decided in the companys favour on July
31, 2015. Work recommenced after a delay of 61 days.
The following periods may be relevant:
Period Days
March 1 to December 31 306
April 1 to December 31 275
August 1 to December 31 153
October 1 to December 31 92
Required
a) Assuming that the loans were taken specifically for the project, calculate the
amount of borrowing costs that s h o u l d be capitalised i n t h e p e r i o d
e n d i n g December 31, 2015 in accordance with the requirements of IAS 23
Borrowing Costs.
b) Assuming that the loans constituted general finance, calculate the amount of
borrowing costs that s h o u l d be capitalised i n t h e p e r i o d e n d i n g
December 31, 2015 in accordance with the requirements of IAS 23 Borrowing
Costs.

Emile Woolf International 54 The Institute of Chartered Accountants of Pakistan


Questions

7.7 HUMAYUN CHEMICALS LIMITED


(a) On July 1, 2013, Humayun Chemicals Limited acquired a machine at a
cost of Rs. 10 million. The useful life of the machine and its salvage value was
estimated at 5 years and Rs. 3.0 million respectively. The cost of machine is
being depreciated under the straight line method.
Based on the practice followed by similar type of companies, the company
has determined that the remaining useful economic life of the machine is six
years. It has also been established that the residual value at the end of the
useful life will be equal to 10% of the cost of machine.

Required
Compute the depreciation expenses and other adjustments (if any) required
to be made in the financial statements of the company for the year ended
June 30, 2015 under each of the following assumptions:
(i) the review of useful life and residual value was carried out on June 30,
2015;
(ii) the review of useful life and residual value was carried out on June 30,
2014 but in the financial statements for the year then ended the
depreciation expense was erroneously recorded on the previous basis.
(11)
(b) Discuss the requirements of International Accounting Standard(s) in
respect of estimation and revision of useful life of an item of property, plant
and equipment. (04)

7.8 FARADAY PHARMACEUTICAL LIMITED


Faraday Pharmaceutical Limited (FPL) acquired a building for Rs. 200 million on
July 1, 2011. The following information relating to the building is available:
(i) It is being depreciated on the straight line basis, over 20 years.
(ii) FPL uses the revaluation model for subsequent measurement of its
property, plant and equipment and accounts for revaluations on the net
replacement value method. The details of revaluation carried out by the
independent valuers during the past years are as follows:
Fair value
Revaluation date
Rupees in million
July 1, 2012 230
July 1, 2013 170
July 1, 2014 180
(iii) FPL transfers the maximum possible amount from the revaluation surplus
to retained earnings on an annual basis.
(iv) There is no change in the useful life of the building.

Required
Prepare the journal entries to record the above transactions from the date of
acquisition of the building to the year ended June 30, 2015.
(Ignore deferred tax) (16)

Emile Woolf International 55 The Institute of Chartered Accountants of Pakistan


Financial accounting and reporting II

7.9 SPIN INDUSTRIES LIMITED


On September 1, 2014, Spin Industries Limited (SIL) started construction of its
new office building and completed it on May 31, 2015. The payments made to the
contractor were as follows:
Date of Payment Rupees
September 1, 2014 10,000,000
December 1, 2014 15,000,000
February 1, 2015 12,000,000
June 1, 2015 9,000,000
In addition to the above payments, SIL paid a fee of Rs. 8 million on September 1,
2014 for obtaining a permit allowing the construction of the building.

The project was financed through the following sources:


(i) On August 1, 2014 a medium term loan of Rs. 25 million was obtained
specifically for the construction of the building. The loan carried mark up of
12% per annum payable semi-annually. A commitment fee @ 0.5% of the
amount of loan was charged by the bank.
Surplus funds were invested in savings account @ 8% per annum. On
February 1, 2015 SIL paid the six monthly interest plus Rs. 5 million towards
the principal.
(ii) Existing running finance facilities of SIL
Running finance facility of Rs. 28 million from Bank A carrying mark
up of 13% payable annually. The average outstanding balance during
the period of construction was Rs. 25 million.
Running finance facility of Rs. 25 million from Bank B. The mark up
accrued during the period of construction was Rs. 3 million and the
average running finance balance during that period was Rs. 20 million.

Required
Calculate the amount of borrowing costs to be capitalised on June 30, 2015 in
accordance with the requirements of International Accounting Standards.
(Borrowing cost calculations should be based on number of months). (18)

7.10 SCIENTIFIC PHARMA LIMITED


Scientific Pharma Limited (SPL) is a manufacturer of pharmaceutical products. In
January 2015, one of its plants suffered a major break down. It was repaired at a
cost of Rs. 1.5 million but the production capacity was reduced significantly. The
plant was ready for production on June 30, 2015. At that time the companys
engineers advised that the plant could be used at a reduced level for 3 years only.
The factory was estimated to have a recoverable amount of Rs. 19,277,000 at June 30,
2015

Other related information is as under:


(i) The plant was imported at FOB price of US$ 800,000. The payment was
made at the time of shipment on July 1, 2005 at Rs. 52 per US$. Other
charges including installation cost amounted to Rs. 7 million. Installation of the
plant was completed on December 31, 2005 and commercial production
commenced from April 1, 2006.

Emile Woolf International 56 The Institute of Chartered Accountants of Pakistan


Questions

(ii) The company uses straight line method of deprecation. Depreciation is


charged from the month the asset is available for use upto the month prior to
disposal. At the time of purchase, the estimated useful life of the plant was
estimated at 15 years whereas the salvage value was estimated at Rs. 2.0
million.
(iii) Based on the report of a professional independent valuer, the plant was
revalued on July 1, 2010 at Rs. 45 million. There was however, no change in
estimated useful life of the plant.
(iv) The factory remained closed from April 1, to June 30, 2012 due to law and
order situation.
(v) The salvage value has not changed since it was first estimated at the time of
purchase.

Required
Prepare accounting entries for the year ended June 30, 2015. Give all the necessary
calculations.
(Ignore taxation) (20)

7.11 QURESHI STEEL LIMITED


On July 1, 2014, Qureshi Steel Limited (QSL) signed an agreement with Pak
Construction Limited for construction of a factory building at a cost of Rs. 100
million. It was agreed that the factory would be ready for use from January 1,
2016. The terms of payments were agreed as under:
(i) 10% advance payment would be made on signing of the agreement. The
advance paid would be adjusted at 10% of the quarterly progress bills.
(ii) 5% retention money would also be deducted from the progress bills.
Retention money will be refunded one year after completion of the factory
building.
(iii) Progress bills will be raised on last day of each quarter and settled on 15th of
the next month.
The under mentioned progress bills were received and settled by QSL as per the
agreement:
Invoice date Amount (Rs. )
September 30, 2014 30 million
December 31, 2014 20 million
March 31, 2015 10 million
June 30, 2015 15 million
On April 30, 2015 an invoice of Rs. 1.5 million was raised by the contractor for
damages sustained at the site, on account of rains. After negotiations, QSL finally
agreed to make additional payment of Rs. 1.0 million to compensate the contractor.
The amount was paid on May 15, 2015. It is expected that 75% of the payment
would be recovered from the insurance company.

The cost of the project has been financed through the following sources:
(i) Issue of right shares amounting to Rs. 15 million, on September 1, 2014.
The company has been following a policy of paying dividend of 20% for the
past many years.

Emile Woolf International 57 The Institute of Chartered Accountants of Pakistan


Financial accounting and reporting II

(ii) Bank loan of Rs. 25 million obtained on December 1, 2014. The loan carries a
markup of 13% per annum. The principal is repayable in 5 half yearly equal
instalments of Rs. 5 million each along with the interest, commencing from
May 31, 2015. Loan processing charges of Rs. 0.5 million were deducted by
the bank at the time of disbursement of loan. Surplus funds, when available,
were invested in short term deposits at 8% per annum.
(iii) Cash withdrawals from the existing running finance facility provided by a
bank. Average running finance balance for the year was Rs. 60 million.
Markup charged by the bank for the year was Rs. 9 million.

Required
Compute cost of capital work in progress for the factory building as of June 30,
2015 in accordance with the requirements of relevant IFRSs.
(Borrowing costs calculations should be based on number of months) (18)

7.12 GRANITE CORPORATION


On 1 March 2014, Granite Corporation (GC) started the construction of a new plant
to meet the growing demand for its products. The new plant was completed at a
cost of Rs. 100 million on 31 May 2015.

GC financed the cost of the project from the following sources:


(i) On 1 March 2014, a 7-year loan of Rs. 70 million was obtained specifically for
the construction of the plant. The loan carried mark-up @ 13% per annum
payable semi-annually. An arrangement fee @ 1% of the loan amount was
paid to the bank.
Two instalments, each comprising of repayment of principal of Rs. 5 million
with interest, were paid on 31 August 2014 and 28 February 2015.
(ii) GC also has a running finance facility of Rs. 100 million carrying mark-up @
14% per annum. Average utilization of this facility, prior to commencement of
construction was Rs. 10 million. Any additional amount required for the
project was provided through this facility.
(iii) Surplus funds were used to reduce the running finance utilization or invested
in savings account @ 8% per annum.
Payments made to the contractor were as follows:
Payment date Rs. m
01 March 2014 25
31 January 2015 65
30 September 2015 10
The construction work was suspended from 1 February 2015 to 28 February 2015.
The suspension was caused due to delay in shipment of essential components for
the installation of the plant.

Required
Calculate the amount of borrowing costs that may be capitalised during the years
ended 30 June 2014 and 2015 in accordance with the requirements of International
Financial Reporting Standards. (20)

Emile Woolf International 58 The Institute of Chartered Accountants of Pakistan


Questions

CHAPTER 8 IAS 38: INTANGIBLE ASSETS

8.1 FAZAL
The following information relates to the financial statements of Fazal for the year to
31 March 2015.
The IT division has begun a training course for all managers in a new programming
language at a cost of Rs. 200,000. The consultants running the training course have
quantified the present value of the training benefits over the next two years to be Rs.
400,000. The project cost has been included in the statement of financial position as
a current asset. The accounting policy note identifies that the costs will be written off
over the next two years to match the benefits.
Required
Explain the correct accounting treatment for the above (with calculations if
appropriate). (3)

8.2 HENRY
During 2015 Henry has the following research and development projects in
progress.
Project A was completed at the end of 2014. Development expenditure brought
forward at the beginning of 2015 was Rs. 412,500 on this project. Savings in
production costs arising from this project are first expected to arise in 2015. In 2015
savings are expected to be Rs. 100,000, followed by savings of Rs. 300,000 in 2016
and Rs. 200,000 in 2017.
Project B commenced on 1 April 2015. Costs incurred during the year were Rs.
56,000. In addition to these costs a machine was purchased on 1 April 2015 for Rs.
30,000 for use on the project. This machine has a useful life of five years. At the end
of 2015 there were still some uncertainties surrounding the completion of the
project.
Project C had been started in 2014. In 2014 the costs relating to this project of Rs.
36,700 had been written off, as at the end of 2014 there were still some
uncertainties surrounding the completion of the project. Those uncertainties have
now been resolved and a further Rs. 45,000 costs incurred during the year.
Required
Show how the above would appear in the financial statements (including notes to
the financial statements) of Henry as of 31 December 2015.

Emile Woolf International 59 The Institute of Chartered Accountants of Pakistan


Financial accounting and reporting II

8.3 TOBY
Toby entered into the following transactions during the year ended 31 December
2015. The directors of Toby wish to capitalise all assets wherever possible.
(1) On 1 January Toby acquired the net assets of George for Rs. 105,000. The
assets acquired had the following book and fair values.
Book value Fair value
Rs. Rs.
Goodwill 5,000 5,000
Patents 15,000 20,000
Non-current assets 40,000 50,000
Other sundry net assets 30,000 25,000

90,000 100,000

The patent expires at the end of 2022. The goodwill arising from the above
had a recoverable value at the end of 2015 of Rs. 7,000.
(2) On 1 April Toby acquired a brand from a competitor for Rs. 50,000. The
directors of Toby have assessed the useful life of the brand as five years.
(3) During the year Toby spent Rs. 40,000 on developing a new brand name. The
development was completed on 30 June. The useful life of this brand has
been assessed as eight years.
(4) The directors of Toby believe that there is total goodwill of Rs. 2 million within
Toby and that this has an indefinite useful life.
Required
Prepare the note to the financial statements for intangible assets as at 31 December
2015.

8.4 BROOKLYN
Brooklyn is a bio-technology company performing research for pharmaceutical
companies. The finance director has contacted your financial consulting company to
arrange a meeting to discuss issues relevant to the preparation of the financial
statements for the year to 30th June 2015. Your initial telephone conversation has
provided the necessary background information.
1 On 1st August 2014 Brooklyn began investigating a new bio-process. On 1st
September 2015, the new process was widely supported by the scientific
community and the feasibility project was approved. A grant was then
obtained relating to future work. Several pharmaceutical companies have
expressed an interest in buying the know how when the project completes in
June 2016. The nominal ledger account set up for the project shows that the
expenditure incurred between 1st August 2014 and 30th June 2015 was Rs.
300,000 per month.
2 In August 2015, an employee lodged a legal claim against the company for
damage to his health as a result of working for the company for the two years
through to 31st March 2014 when he had to retire due to ill health. He has
argued that his health deteriorated as a result of the stress from his position in
the organisation. Brooklyn has denied the claim and has appointed an
employment lawyer to assist with contesting the case. The lawyer has advised
that there is a 25% chance that the claim will be rejected, 50% chance that the
damages will be Rs. 600,000 and 25% chance of Rs. 1 million. The company
has an insurance policy that will pay 10% of any damages to the company.

Emile Woolf International 60 The Institute of Chartered Accountants of Pakistan


Questions

The lawyer has said that the case could take until 30th June 2018 to resolve.
The present value of the estimated damages discounted at 8% is Rs. 476,280
and Rs. 793,800 respectively.
3 Brooklyn owns several buildings, which include an administrative office in the
centre of London. The company has revalued these on a regular basis every
five years and the next valuation is due on 30th June 2017. Property prices
have increased since the last review and particularly for the London premises.
The cost of engaging a professionally qualified valuer is very expensive and
so to reduce costs the finance director is proposing that the property manager,
who is a professionally qualified valuer, should value the London property and
that the increase in value should be included in the financial statements. The
finance director is of the opinion that the property prices may fall next year.

Required
Prepare notes for your meeting with the finance director which explain and justify the
accounting treatment of these issues, preparing calculations where appropriate and
identifying matters on which your require further information. (25)

8.5 ZOUQ INC


Zouq Inc. is a multinational company. As part of its vision to expand its business
in South Asia, it purchased a 90% share of a locally incorporated company, Momin
Limited. Following are the brief details of the acquisition:

Date of acquisition January 1, 2014


Total paid up capital of Momin Limited (Rs. 10 each) 500,000,000
Purchase price per share Rs. 30
Net assets of Momin Limited (as per 2013 audited financial
statements) 650,000,000
Fair value of net assets (other than intangible assets) of Momin
Limited 1,100,000,000
Momin Limited has an established line of products under the brand name of
Badar. On behalf of Zouq Inc., a firm of specialists has valued the brand name at
Rs. 100 million with an estimated useful life of 10 years at January 1, 2014. It is
expected that the benefits will be spread equally over the brands useful life.
An impairment test of goodwill and brand was carried out on December 31,
2014 which indicated an impairment of Rs. 50 million in the value of goodwill.
An impairment test carried out on December 31, 2015 indicated a decrease of Rs.
13.5 million in the carrying value of the brand.

Required:
(a) What are the requirements of International Accounting Standards relating to
amortization of intangible assets having finite life?
(b) Prepare the ledger accounts for goodwill and the brand, showing initial
recognition and all subsequent adjustments. (15)

Emile Woolf International 61 The Institute of Chartered Accountants of Pakistan


Financial accounting and reporting II

8.6 STAR-BRIGHT PHARMACEUTICAL LIMITED


Star-Bright Pharmaceutical Limited (SPL), a listed company, purchased a brand
on January 1, 2010 at a cost of Rs. 382 million. It has incurred a substantial
amount on further development of the brand, in subsequent years.
It is the policy of SPL to amortise the development expenditures which meet
the recognition criteria as given in IAS-38 Intangible Assets, over a period of ten
years. The amortization commences when the development expenditures first
meet the recognition criteria. However, it was discovered during the year 2015 that
the development expenditure incurred after acquisition had erroneously been
written-off to the profit and loss account, details of which are as follows:
Year ended Rs. m

December 31, 2012 24

December 31, 2013 54

December 31, 2014 38

December 31, 2015 43

The draft financial statements (before correction of error) show that retained
earnings as at December 31, 2015 was Rs. 1,950 million (2014: Rs. 1,785 million).

Required
In accordance with the requirements of International Financial Reporting Standards,
prepare relevant extracts of the Statement of Financial Position along with the
note on intangible assets after incorporating the required corrections.
(Ignore tax) (16)

8.7 RAISIN INTERNATIONAL


(a) Discuss the criteria that should be used while recognizing intangible assets
arising from research and development work. (05 marks)
(b) Raisin International (RI) is planning to expand its line of products. The related
information for the year ended 31 December 2015 is as follows:
(i) Research and development of a new product commenced on 1 January
2015. On 1 October 2015, the recognition criteria for capitalization of an
internally generated intangible asset were met. It is estimated that the
product would have a useful life of 7 years. Details of expenditures
incurred are as follows:
Rs. m
Research work 4.50
Development work 9.00
Training of production staff 0.50
Cost of trial run 0.80

Total costs 14.80

Emile Woolf International 62 The Institute of Chartered Accountants of Pakistan


Questions

(ii) The right to manufacture a well-established product under a patent for a


period of five years was purchased on 1 March 2015 for Rs. 17 million.
The patent has an expected remaining useful life of 10 years. RI has the
option to renew the patent for a further period of five years for a sum of
Rs. 12 million.
(iii) RI has acquired a brand at a cost of Rs. 2 million. The cost was incurred
in the month of June 2015. The life of the brand is expected to be 10
years. Currently, there is no active market for this brand. However, RI is
planning to launch an aggressive marketing campaign in February 2016.
(iv) In September 2014, RI developed a new production process and
capitalised it as an intangible asset at Rs. 7 million. The new process is
expected to have an indefinite useful life. During 2015, RI incurred
further development expenditure of Rs. 3 million on the new process
which meets the recognition criteria for capitalization of an intangible
asset.

Required
In the light of International Financial Reporting Standards, explain how each of the
above transaction should be accounted for in the financial statements of Raisin
International for the year ended 31 December 2015. (11)

Emile Woolf International 63 The Institute of Chartered Accountants of Pakistan


Financial accounting and reporting II

CHAPTER 9 IAS 17: LEASES

9.1 DAWOOD
The following information relates to the financial statements of Dawood for the year
to 31 March 2015.
On 1 October 2014, Dawood entered into a 5 year lease for a machine from
Narbonne, agreeing to make payments every 6 months of Rs. 29,500 beginning on
the 1 October. The cash price of the machine is Rs. 250,000 and the machine is
believed to have a useful life of 5 years. Dawood has treated the arrangement as a
finance lease. Any finance costs are to be treated using the sum-of-digits method.
Required
Explain the correct accounting treatment for the above (with calculations if
appropriate). (7)

9.2 FINLEY
On 1 January 2015, Finley entered into an agreement to lease a boat. The fair value
of the boat was Rs. 36,000 and the term of the lease was four years. Annual lease
payments of Rs. 10,000 are payable in advance. The interest rate implicit in the
lease is 7.5%. Finley is responsible for insuring and maintaining the boat throughout
the term of the lease.
Required
Show how this lease would be presented in the statement of profit or loss of Finley
for the year ended 31 December 2015 and the statement of financial position as at
that date. Detailed disclosure notes are not required.

9.3 FABIAN
In the year ended 31 December 2015, Fabian leased two assets.
(1) A car was leased on 1 July 2015 via a three year lease agreement. Fabian
paid a deposit of Rs. 7,500 followed by 36 monthly payments of Rs. 700 each
on the 1st of each month. At the end of the three years Fabian will return the
car. The car has a useful life of eight years.
(2) A machine was leased on 1 January 2015 via a four year lease. The machine
has a fair value of Rs. 130,000 and Fabian is responsible for its upkeep.
Lease payments of Rs. 40,000 are payable in arrears annually. The interest
rate implicit in the lease is 10% and the present value of the minimum lease
payments is Rs. 126,760.
Required
Show how the two lease agreements would be presented in the statement of profit
or loss for 2015 and the statement of financial position at 31 December 2015. Notes
to the financial statements are not required.

Emile Woolf International 64 The Institute of Chartered Accountants of Pakistan


Questions

9.4 XYZ INC


A lessor, ABC Inc, leases an asset, which it purchased for Rs. 4,400,000, to XYZ Inc
under a finance lease. It estimates that its residual value after five years will be Rs.
400,000 and after seven years will be zero.
The lease is for five years at a rental of Rs. 600,000 per half year in advance, with
an option of two more years at nominal rental. The lease commences on 1 January
2015. The directors of XYZ Inc consider that the asset has a useful life of seven
years. The finance charge is to be allocated using the sum of digits (rule of 78)
method. Title to the asset will pass to XYZ at the end of seven years if the option is
exercised. It is likely that it will be.

Required
(a) Show the relevant extracts from the accounts of XYZ Inc at 31 December
2015. (5)
(b) Show the allocation of the finance charge for XYZ Inc using the actuarial
before tax method (using the interest rate implicit in the lease). Compare this
with the sum of the digits allocation in (a) above. (14)
The rate of interest implicit in the lease is 7.68% per half year.

9.5 SNOW INC


On 1 January 2015, Snow Inc entered into the following finance lease agreements.
(a) Snowplough
To lease a snowplough for 3 years from Ice Inc. The machine had cost Ice Inc
Rs. 35,000,000.
A deposit of Rs. 2,000,000 was payable on 1 January 2015 followed by 6 half
yearly instalments of Rs. 6,500,000 payable in arrears, commencing on 30
June 2015. Finance charges are to be allocated on a sum of digits basis.
(b) Snow machine
To lease a snow machine for 5 years from Slush Inc. The snow machine cost
Slush Inc Rs. 150,000 and is estimated to have a useful life of 5 years.
Snow Inc has agreed to make 5 annual instalments of Rs. 35,000,000,
payable in advance, commencing on 1 January 2015.
The interest rate implicit in the lease is 8.36%.

Required
Show the relevant extracts from the accounts of Snow Inc for year ended 31
December 2015. (15)

Emile Woolf International 65 The Institute of Chartered Accountants of Pakistan


Financial accounting and reporting II

9.6 MIRACLE TEXTILE LIMITED


On 1 July 2013, Miracle Textile Limited (MTL) acquired a machine on lease,
from a bank.
Details of the lease are as follows:
(i) Cost of machine is Rs. 20 million.
(ii) The lease term and useful life is 4 years and 10 years respectively.
(iii) Instalment of Rs. 5.80 million is to be paid annually in advance on 1 July.
(iv) The interest rate implicit in the lease is 15.725879%.
(v) At the end of lease term, MTL has an option to purchase the machine on
payment of Rs. 2 million. The fair value of the machine at the end of lease
term is expected to be Rs. 3 million.
MTL depreciates the machine on the straight line method to a nil residual value.

Required
Prepare relevant extracts of the statement of financial position and related notes to
the financial statements for the year ended 30 June 2015 along with comparative
figures. Ignore taxation (16)

9.7 SHOAIB LEASING LIMITED


Shoaib Leasing Limited (the lessor) has entered into a three year agreement with
Sarfaraz Limited (the lessee) to lease a machine with an expected useful life of 4
years. The cost of machine is Rs. 2,100,000.
The following information relating to lease transaction is available:
(i) Date of commencement of lease is July 1, 2015.
(ii) The lease contains a purchase bargain option at Rs. 100,000. At the end of the
lease term, the value of the machine will be Rs. 300,000.
(iii) Lease instalments of Rs. 860,000 are payable annually, in arrears, on June 30.
(iv) The implicit interest rate is 12.9972%.

Required
(a) Prepare the journal entries for the years ending June 30, 2016, 2017 and
2018 in the books of lessor. Ignore tax.
(b) Produce extracts from the statement of financial position including relevant
notes as at June 30, 2016 to show how the transactions carried out in 2016
would be reflected in the financial statements of the lessor.
(Disclosure of accounting policy is not required.) (20)

Emile Woolf International 66 The Institute of Chartered Accountants of Pakistan


Questions

9.8 NEPTUNE LIMITED


Neptune Limited (NL) had established its business in December 2014 as a supplier
of plant and machinery. During the year ended December 31, 2015 the company
sold two machines under lease arrangements. The details are as under:
A B
Date of commencement of lease January 1, 2015 January 1, 2015
Lease period 6 years 3 years
Lease instalments payable annually Rs. 2,000,000 Rs. 4,000,000
in advance (to be reduced
annually by 5%)
Cost of machine Rs. 6,963,448 Rs. 15,000,000
Economic life 6 years 6 years
NL sells machines on cash at cost plus 25%. It depreciates its assets under
straight line method with no residual value. Fair market annual interest rate is 15%.
Required
(a) Prepare journal entries to record the above transactions.
(b) Prepare notes to the financial statements for the year ended December
31, 2015 in accordance with the requirements of IAS - 17 (Leases). (19)
(Ignore taxation and comparative figures)

9.9 QUARTZ AUTO LIMITED


Quartz Auto Limited (QAL) is engaged in the business of manufacturing of
trucks. Since a number of the prospective customers do not have adequate
funds to purchase the vehicles against full payment, QAL provides lease financing
facility to its customers. It expects to receive a return at the rate of 15% per annum
on the amount of lease finance.
On 1 July 2014, QAL sold seven trucks to Emerald Goods Transport Company
(EGTC) on lease. The terms of the lease and related information are as follows:
(i) The lease period is 4 years, extendable up to the expected useful life of
the trucks i.e. 5 years.
(ii) EGTC has guaranteed a residual value of Rs. 360,000 for each truck, till
the end of the fourth year. However, the guarantee would lapse if the lease
term is extended to the fifth year. EGTC will return the truck at the end of the
lease term.
(iii) Lease rentals amount to Rs. 2,715,224 per annum and are payable in
arrears i.e. on 30 June.
(iv) The cost of each truck is Rs. 900,000. Price in case of outright sale is Rs.
1,350,000 per truck.
(v) The expected residual value of each truck at the end of the 4th and 5th year is
Rs. 150,000 and Rs. 100,000 respectively.
Required
Assuming that QAL and EGTC intend to extend the lease for a period of five years,
prepare:
(a) Journal entries to record the transactions for the year ended 30 June 2015. (08)
(b) A note for inclusion in the financial statements, for the year ended 30
June 2015, in accordance with the requirements of IAS 17 Leases. (07)

Emile Woolf International 67 The Institute of Chartered Accountants of Pakistan


Financial accounting and reporting II

9.10 LODHI TEXTILE MILLS LIMITED


Lodhi Textile Mills Limited is facing severe financial difficulties. To improve the
cash flows, the management has decided to sell and lease back three power
generators of the company under three different sale and lease back arrangements
which were signed on August 15, 2015. The company has assessed that all the
leases shall qualify as finance leases.
The related information as on August 15, 2015 is given below:
Cost Book Fair Value Value in Amount of
Value Use Financing
Rs. 000 Rs. 000 Rs. 000 Rs. 000 Rs. 000
Generator A 10,000 7,500 6,000 6,500 6,000
Generator B 12,000 6,000 4,000 5,000 6,000
Generator C 10,000 7,000 10,000 12,000 8,000
Required
Prepare the accounting entries that should be recorded by the company on
August 15, 2015 in respect of the above transactions. (13)
Note: Ignore tax and deferred tax implications, if any.

9.11 NOMAN ENGINEERING LIMITED


Noman Engineering Limited (NEL) manufactures auto parts. On July 1, 2014 it
finalised a lease agreement with a bank for sale and leaseback of one of its plants
costing Rs. 18.75 million.
Relevant information is as under:
(i) Proceeds from the bank amounting to Rs. 20 million which represent the
prevailing market value of such type and age of plant, were received on July 1,
2014.
(ii) The plant had a book value of Rs. 15 million at the time of commencement of
the lease.
(iii) The remaining life of the plant on July 1, 2014 was estimated at 8 years.
(iv) The lease period is 6 years. Lease instalments of Rs. 2.5 million each are
payable semi-annually in arrears from December 31, 2014.
(v) NEL has the option to purchase the plant at market value at the end of the
lease term. No final decision has yet been made by NEL, in this regard.
(vi) The rate of interest implicit in the lease is 13.731% per annum.

Required
Pass journal entries in respect of the lease, for the year ended June 30, 2015. (12)

Emile Woolf International 68 The Institute of Chartered Accountants of Pakistan


Questions

CHAPTER 10 IAS 37: PROVISIONS CONTINGENT LIABILITIES AND


CONTINGENT ASSETS AND IAS 10: EVENTS OCCURRING AFTER THE
REPORTING DATE
10.1 BADAR
The following information relates to the financial statements of Badar for the year to
31 March 2015.
The mining division of Badar has a 3 year operating licence from an overseas
government. This allows it to mine and extract copper from a particular site. When
the licence began on 1 April 2014, Badar started to build on the site. The cost of the
construction was Rs. 500,000.
The overseas country has no particular environmental decommissioning laws. In its
past financial statements Badar has given information about the companys
environmental policy and has provided examples to demonstrate that it is a
responsible company that believes in restoring mining sites at the end of the
extraction period. The cost of removing the construction at the end of the three
years is estimated to be Rs. 100,000.
The cost of the site currently shown in the trial balance is Rs. 500,000. The
company has a cost of borrowing of 10%.
Required
Explain the correct accounting treatment for the above (with calculations if
appropriate). (6)

10.2 GEORGINA
Georgina Company is preparing its financial statements for the year ended 30
September 2015. The following matters are all outstanding at the year end.
(1) Georgina is facing litigation for damages from a customer for the supply of
faulty goods on 1 September 2015. The claim, which is for Rs. 500,000, was
received on 15 October 2015. Georginas legal advisors consider that
Georgina is liable and that it is likely that this claim will succeed. On 25
October 2015 Georgina sent a counter-claim to its suppliers for Rs. 400,000.
Georginas legal advisors are unsure whether or not this claim will succeed.
(2) Georginas sales director, who was dismissed on 15 September, has lodged a
claim for Rs. 100,000 for unfair dismissal. Georginas legal advisors believe
that there is no case to answer and therefore think it is unlikely that this claim
will succeed.
(3) Although Georgina has no legal obligation to do so, it has habitually operated
a policy of allowing customers to return goods within 28 days, even where
those goods are not faulty. Georgina estimates that such returns usually
amount to 1% of sales. Sales in September 2015 were Rs. 400,000. By the
end of October 2015, prior to the drafting of the financial statements, goods
sold in September for Rs. 3,500 had been returned.
(4) On 15 September 2015 Georgina announced in the press that it is to close
one of its divisions in January 2016. A detailed closure plan is in place and the
costs of closure are reliably estimated at Rs. 300,000, including Rs. 50,000 for
staff relocation.
Required
State, with reasons, how the above should be treated in Georginas financial
statements for the year ended 30 September 2015.

Emile Woolf International 69 The Institute of Chartered Accountants of Pakistan


Financial accounting and reporting II

10.3 EARLEY INC


Earley Inc is finalising its accounts for the year ended 31 December 2014. The
following events have arisen since the year end and the financial director has asked
you to comment on the final accounts.
(a) At 31 December 2014 trade receivables included a figure of Rs. 250,000 in
respect of Nedengy Inc. On 8 March 2015, when the current debt was Rs.
200,000, Nedengy Inc went into receivership. Recent correspondence with the
receiver indicates that no dividend will be paid to unsecured creditors.
(b) On 15 March 2015 Earley Inc sold its former head office building, Whitley
Wood, for Rs. 2.7 million. At the year end the building was unoccupied and
carried at a value of Rs. 3.1 million.
(c) Inventories at the year-end included Rs. 650,000 of a new electric tricycle, the
Opasney. In January 2015 the European Union declared the tricycle to be
unsafe and prohibited it from sale. An alternative market, in Bongolia, is being
investigated, although the current price is expected to be cost less 30%.
(d) Stingy Inc, a subsidiary in Outer Sonning, was nationalised in February 2015.
The Outer Sonning authorities have refused to pay any compensation. The net
assets of Stingy Inc have been valued at Rs. 200,000 at the year end.
(e) Freak floods caused Rs. 150,000 damage to the Southcote branch of Earley
Inc in January 2015. The branch was fully insured.
(f) On 1 April 2015 Earley Inc announced a 1 for 1 rights issue aiming to raise Rs.
15 million.

Required
Explain how you would respond to the matters listed above. (13)

10.4 ACCOUNTING TREATMENT


You have been asked to advise on the appropriate accounting treatment for the
following situations arising in the books of various companies. The year end in each
case can be taken as 31 December 2015 and you should assume that the amounts
involved are material in each case.
(a) At the year end there was a debit balance in the books of a company for Rs.
15,000, representing an estimate of the amount receivable from an insurance
company for an accident claim. In February 2016, before the directors had
agreed the final draft of the published accounts, correspondence with lawyers
indicated that Rs. 18,600 might be payable on certain conditions.
(b) A company has an item of equipment which cost Rs. 400,000 in 2012 and was
expected to last for ten years. At the beginning of the 2015 financial year the
book value was Rs. 280,000. It is now thought that the company will soon
cease to make the product for which the equipment was specifically
purchased. Its recoverable amount is only Rs. 80,000 at 31 December 2015.
(c) On 30 November a company entered into a legal action defending a claim for
supplying faulty machinery. The companys solicitors advise that there is a
20% probability that the claim will succeed. The amount of the claim is Rs.
500,000.

Emile Woolf International 70 The Institute of Chartered Accountants of Pakistan


Questions

(d) An item has been produced at a manufacturing cost of Rs. 1,800 against a
customers order at an agreed price of Rs. 2,300. The item was in inventory at
the year-end awaiting delivery instructions. In January 2016 the customer was
declared bankrupt and the most reasonable course of action seems to be to
make a modification to the unit, costing approximately Rs. 300, which is
expected to make it marketable with other customers at a price of about Rs.
1,900.
(e) At 31 December a company has a total potential liability of Rs. 1,000,400 for
warranty work on contracts. Past experience shows that 10% of these costs
are likely to be incurred, that 30% may be incurred but that the remaining 60%
is highly unlikely to be incurred.

Required
For each of the above situations outline the accounting treatment you would
recommend and give the reasoning of principles involved. The accounting treatment
should refer to entries in the books and/or the year-end financial statements as
appropriate. (12)

10.5 J-MART LIMITED


(a) Explain the terms adjusting events and non-adjusting events and give three
examples of each. (05)
(b) J-Mart Limited, a chain of departmental stores has distributed its operations
into four Divisions i.e. Food, Furniture, Clothing and Household Appliances.
The following information has been extracted from the records:
(i) The company allows the dissatisfied customers to return the goods
within 30 days. It is estimated that 5% of the sales made in June 2015
will be refunded in July 2015.
(ii) On June 2, 2015, three employees were seriously injured as a result of
a fire at the companys warehouse. They have lodged claims seeking
damages of Rs. 2.0 million from the company. The companys lawyers
have advised that it is probable that the court may award compensation
of Rs. 400,000.
(iii) Under a new legislation, the company is required to fit smoke detectors
at all the stores by December 31, 2015. The company has not yet
installed the smoke detectors.
(iv) On June 20, 2015, the board of directors decided to close down the
Household Appliances Division. However, the decision was made
public after June 30, 2015.
(v) The company has a large warehouse in Lahore which was acquired
under a three-year rent agreement signed on April 1, 2014. The
agreement is non- cancellable and the company cannot sub-let the
warehouse. However, due to operational difficulties, the company shifted
the warehouse to a new location.
(vi) A 15% cash dividend was declared on July 5, 2015.
Required
Describe how each of the above issue should be dealt with in the financial
statements for the year ended June 30, 2010. Support your point of view in
the light of relevant International Accounting Standards. (15)

Emile Woolf International 71 The Institute of Chartered Accountants of Pakistan


Financial accounting and reporting II

10.6 AKBER CHEMICALS LIMITED


Akber Chemicals Limited is engaged in the business of manufacture and sale
of different type of chemicals. The following transactions have not yet been
incorporated in the financial statements for the year ended June 30, 2015:
(a) On June 15, 2015, one of its tankers carrying chemicals fell into a canal, thus
polluting the water. The company has never faced such a situation before.
The company has neither any legal obligation to clean the canal nor does it
have any published environmental policy. In a meeting held on July 26,
2015 the Board of Directors decided to clean the canal, which is estimated to
cost Rs. 5.5 million.
(b) During the second week of July 2015, a significant decline in the demand for
companys products was observed which also led to a decrease in net
realizable value of finished goods. It was estimated that goods costing Rs.
25 million as at June 30, 2015 would only fetch Rs. 23 million.
(c) On June 21, 2015, a customer lodged a claim of Rs. 2 million with the
company as a consignment dispatched on June 1, 2015 was not according to
the agreed specifications. The companys inspection team found that this
defect arose because of inferior quality of raw materials supplied by the
vendor. On June 28, 2015, the company lodged a claim for damages of Rs.
5.0 million, with its vendor, which include reimbursement of the cost of raw
materials. The company anticipates that it will have to pay compensation to its
customer and would be able to recover 50% of the amount claimed from the
vendor.

Required
Discuss how Akber Chemicals (Pvt.) Limited would deal with the above
situations in its financial statements for the year ended June 30, 2015. Explain
your point of view with reference to the guidance contained in the International
Financial Reporting Standards. (13)

10.7 QALLAT INDUSTRIES LIMITED


The following information pertains to Qallat Industries Limited (QIL) for its financial
year ended June 30, 2015:
(i) QIL sells all its products on one-year warranty which covers all types of
defects. Previous history indicates that 2% of the products contain major
defects whereas 10% have minor defects. It is estimated that if major defects
were detected in all the products sold, repair cost of Rs. 150 million would
result. If minor defects were detected in all products sold, repair cost of Rs.
70 million would result. Total sales for the year are amounted to Rs. 830
million.
(ii) QIL has two large warehouses, A and B. These were acquired under non-
cancellable lease agreements. Details are as follows:
Warehouse A Warehouse B
Effective date of agreement July 1, 2010 January 1, 2013
Lease period 10 years 8 years
Rental amount per month Rs. 450,000 Rs. 300,000

Emile Woolf International 72 The Institute of Chartered Accountants of Pakistan


Questions

On account of serious operating difficulties, QIL vacated both the


warehouses on January 1, 2015 and moved to a warehouse situated close
to its factory. On the same day QIL sub-let Warehouse A at Rs. 250,000
per month for the remaining lease period. Warehouse B was sub-let on
March 1, 2015 for Rs. 350,000 per month for the remaining lease period.
(iii) On July 18, 2015, QIL was sued by an employee claiming damages for Rs. 6
million on account of an injury caused to him due to alleged violation of safety
regulations on the part of the company, while he was working on the machine
on June 15, 2015. Before filing the suit, he contacted the management on
June 29, 2015 and asked for compensation of Rs. 4 million which was turned
down by the management. The lawyer of the company anticipates that the
court may award compensation ranging between Rs. 1.5 million to Rs. 3
million. However, in his view the most probable amount is Rs. 2 million.
(iv) On November 1, 2014 a new law was introduced requiring all factories to
install specialised safety equipment within four months. The Equipment
costing Rs. 5.0 million was ordered on December 15, 2014 against 100%
advance payment but the supplier delayed installation to July 31, 2015. On
August 5, 2015 the company received a notice from the authorities levying a
penalty of Rs. 0.4 million i.e. Rs. 0.1 million for each month during which the
violation continued. QIL has lodged a claim for recovery of the penalty from
the supplier of the equipment.
Required
Describe how each of the above issues should be dealt with in the financial
statements for the year ended June 30, 2015. Support your answer in the light of
relevant International Accounting Standards and quantify the effect where possible.
(14)
10.8 SKYLINE LIMITED
The following information pertains to Skyline Limited (SL) for the financial year
ended December 31, 2015:
(i) A customer who owed Rs. 1 million was declared bankrupt after his
warehouse was destroyed by fire on February 10, 2016. It is expected that the
customer would be able to recover 50% of the loss from the insurance
company.
(ii) An employee of SL forged the signatures of directors and made cash
withdrawals of Rs. 7.5 million from the bank. Of these, Rs. 1.5 million were
withdrawn before December 31, 2015. Investigations revealed that an
employee of the bank was also involved and therefore, under a settlement
arrangement, the bank paid 60% of the amount to SL on January 27, 2016.
(iii) SL has filed a claim against one of its vendors for supplying defective goods.
SLs legal consultant is confident that damages of Rs. 1 million would be paid
to SL. The supplier has already reimbursed the actual cost of the defective
goods.
(iv) A suit for infringement of patents, seeking damages of Rs. 2 million, was filed
by a third party. SLs legal consultant is of the opinion that an unfavourable
outcome is most likely. On the basis of past experience he has advised that
there is 60% probability that the amount of damages would be Rs. 1 million
and 40% likelihood that the amount would be Rs. 1.5 million.
Required
Advise SL about the amount of provision that should be incorporated and the
disclosures that are required to be made in the financial statements for the year
ended December 31, 2015. (16)

Emile Woolf International 73 The Institute of Chartered Accountants of Pakistan


Financial accounting and reporting II

10.9 WALNUT LIMITED


Walnut Limited (WL) is engaged in the business of import and distribution of
electronic appliances.
The following events took place subsequent to the reporting period i.e. 31 December
2015:
(i) On 15 January 2016, one of WLs competitors announced launching of an
upgraded version of DVD players. WLs inventories include a large stock of
existing version of DVD players which are valued at Rs. 15 million. Because of
the introduction of the upgraded version, the net realizable value of the
existing version in WLs inventory at 31 December 2015 has reduced to Rs.
12.5 million.
(ii) On 20 December 2015, the board of directors decided to close down the
division which imports and sells mobile sets. This decision was made public
on 29 December 2015. However, the business was actually closed on 29
February 2016.
Net costs incurred in connection with the closure of this division were as
follows:
Rs. m
Redundancy costs 1.50
Staff training 0.15
Operating loss from 1 July 2015 to closure of division 0.80
Less: Profit on sale of remaining mobile sets (0.50)
1.95
(iii) On 16 January 2016, LED TV sets valuing Rs. 3 million were stolen from a
warehouse. These sets were included in WLs inventory as at 31 December
2015.
(iv) WL owns 9,000 shares of a listed company whose price as on 31 December
2015 was Rs. 22 per share. During February 2016, the share price declined
significantly after the government announced a new legislation which would
adversely affect the companys operations. No provision in this regard has
been made in the draft financial statements.
(v) On 31 January 2016, a customer announced voluntary liquidation. On 31
December 2015, this customer owed Rs. 1.5 million.
(vi) On 15 February 2016, WL announced final dividend for the year ended 31
December 2015 comprising 20% cash dividend and 10% bonus shares, for its
ordinary shareholders.

Required
Describe how each of the above transactions should be accounted for in the
financial statements of Walnut Limited for the year ended 31 December 2015.
Support your answer in the light of relevant International Financial Reporting
Standards. (16)

Emile Woolf International 74 The Institute of Chartered Accountants of Pakistan


Questions

10.10 ATTOCK TECHNOLOGIES LIMITED


Attock Technologies Limited (ATL) manufactures five hi-tech products, each on a
different plant. It is in the process of preparing its financial statements for the year
ended June 30, 2015. As the CFO of the company, the following matters are under
your consideration:
(i) Inventory carried at Rs. 25 million on June 30, 2015 was sold for Rs. 15
million after it had been damaged in a flood, in July 2015.
(ii) On July 5, 2015 one of ATLs corporate customers declared bankruptcy.
The liquidator announced on August 25, 2015 that 20% of the debt would be
paid on liquidation.
(iii) A new product introduced by a competitor on August 1, 2015 had caused
a significant decline in the market demand of one of ATLs major products. As
a result, ATL is considering a reduction in price and a cut in production.
(iv) On August 18, 2015 the government announced a retrospective increase
in the tax rate applicable to the company.
(v) The directors of ATL declared a dividend of Rs. 3 per share on August 28,
2015.

Required
State how the above events should be treated in ATLs financial statements for the
year ended June 30, 2015. You may assume that all the above events are material
to the company. (11)

Emile Woolf International 75 The Institute of Chartered Accountants of Pakistan


Financial accounting and reporting II

CHAPTER 11 IAS 8: ACCOUNTING POLICIES, CHANGES IN ACCOUNTING


ESTIMATES AND ERRORS

11.1 WONDER LIMITED


Wonder Limited (WL) is engaged in the manufacturing and sale of textile machinery.
Following are the draft extracts of the statement of financial position and the
statement of profit or loss for the year ended 30 June 2015:
Statement of Financial Position
2015 2014
Rs. m Rs. m
Property, plant and equipment 189 130
Retained earnings 166 108
Deferred tax liability 45 27

Statement of profit or loss


2015 2014
Rs. m Rs. m
Profit before taxation 90 120
Taxation 32 42
Profit after taxation 58 78

Following additional information has not been taken into account in the
preparation of the above financial statements:
(i) Cost of repairs amounting to Rs. 20 million was erroneously debited to the
machinery account on 1 October 2013. The estimated useful life of the
machine is 10 years.
(ii) On 1 July 2014, WL reviewed the estimated useful life of its plant and
revised it from 5 years to 8 years. The plant was purchased on 1 July 2013 at
a cost of Rs. 70 million.
Depreciation is provided under the straight line method. Applicable tax rate is 30%.

Required
Prepare relevant extracts (including comparative figures) for the year ended 30 June
2015 related to the following:
(a) Statement of financial position
(b) Statement of profit or loss
(c) Statement of changes in equity
(d) Correction of error note (20)

Emile Woolf International 76 The Institute of Chartered Accountants of Pakistan


Questions

11.2 DUNCAN
Duncan Company has previously written off any expenditure on borrowing costs in
the period in which it was incurred.
The company has appointed new auditors this year. They have expressed the view
that the previous recognition of borrowing costs in the statement of profit or loss was
in error. The company has decided to correct the error retrospectively in accordance
with IAS 8.
The financial statements for 2014 and the 2015 draft financial statements, both
reflecting the old policy, show the following.
Statement of changes in equity (extract)
2014 2015
Retained Retained
earnings earnings
Rs. 000 Rs. 000
Opening balance 22,500 23,950
Profit after tax for the period 3,200 4,712
Dividends paid (1,750) (2,500)

Closing balance 23,950 26,162

Borrowing costs written off were Rs. 500,000 in 2014 and Rs. 600,000 in 2015.
The directors have calculated that borrowing costs, net of depreciation which should
have been included in property, plant and equipment had the correct policy been
applied, are as follows.
Rs. 000
At 30 December 2013 400
At 31 December 2014 450
At 31 December 2015 180
Had the correct policy been in force depreciation of Rs. 450,000 would have been
charged in 2014 and Rs. 870,000 in 2015.
Required
Show how the change in accounting policy must be reflected in the statement of
changes in equity for the year ended 31 December 2015. Work to the nearest Rs.
000.

Emile Woolf International 77 The Institute of Chartered Accountants of Pakistan


Financial accounting and reporting II

11.3 MOHANI MANUFACTURING LIMITED


Mohani Manufacturing Limited is engaged in manufacturing of spare parts for
motor car assemblers. The audited financial statements for the year ended
December 31, 2014 disclosed that the profit and retained earnings were Rs. 21
million and Rs. 89 million respectively. The draft financial statements for the year
show a profit of Rs. 15 million. However, following adjustments are required to be
made:
(i) The management of the company has decided to change the method for
valuation of raw materials from FIFO to weighted average. The value of
inventory under each method is as follows:
FIFO Weighted Average
Rs. m Rs. m
December 31, 2013 37.0 35.5
December 31, 2014 42.3 44.5
December 31, 2015 58.4 54.4
(ii) In 2014, the company purchased a plant for Rs. 100 million. Depreciation on
plant was recorded at Rs. 25 million instead of Rs. 10 million. This error was
discovered after the publication of financial statements for the year ended
December 31, 2014. The error is considered to be material.

Required
Produce an extract showing the movement in retained earnings, as would
appear in the statement of changes in equity for the year ended December 31,
2015. (11)

Emile Woolf International 78 The Institute of Chartered Accountants of Pakistan


Questions

CHAPTER 12 IAS 12: INCOME TAXES


12.1 FRANCESCA
On 30 June 2014 Francesca Company had a credit balance on its deferred tax
account of Rs. 1,340,600 all in respect of the difference between depreciation and
capital allowances.

During the year ended 30 June 2015 the following transactions took place.
(1) Rs. 45 million was charged against profit in respect of depreciation. The tax
computation showed capital allowances of Rs. 50 million.
(2) Interest receivable of Rs. 50,000 was reflected in profit for the period.
However, only Rs. 45,000 of interest was actually received during the year.
Interest is not taxed until it is received.
(3) Interest payable of Rs. 32,000 was treated as an expense for the period.
However, only Rs. 28,000 of interest was actually paid during the year.
Interest is not an allowable expense for tax purposes until it is paid.
(4) During the year Francesca incurred development costs of Rs. 500,600, which
it has capitalised. Development costs are an allowable expense for tax
purposes in the period in which they are paid.
(5) Land and buildings with a net book value of Rs. 4,900,500 were revalued to
Rs. 6 million.
The tax rate is 30%. Francesca has a right of offset between its deferred tax
liabilities and its deferred tax assets.
Required
Calculate the deferred tax liability on 30 June 2015. Show where the increase or
decrease in the liability in the year would be charged or credited.

12.2 SHEP (I)


Shep was incorporated on 1 January 2015. In the year ended 31 December 2015
the company made a profit before taxation of Rs. 121,000
During the period Shep made the following capital additions.
Rs.
Plant 48,000
Motor vehicles 12,000

During the period:


Accounting depreciation 11,000
Tax depreciation 15,000
Tax is chargeable at a rate of 30%.
Required
(a) Calculate the corporate income tax liability for the year ended 31st December
2015.
(b) Calculate the deferred tax balance that is required in the statement of financial
position as at 31st December 2015.
(c) Prepare a note showing the movement on the deferred tax account and thus
calculate the deferred tax charge for the year ended 31st December 2015
(d) Prepare the statement of profit or loss note which shows the compilation of the
tax expense for the year ended 31st December 2015.

Emile Woolf International 79 The Institute of Chartered Accountants of Pakistan


Financial accounting and reporting II

12.3 SHEP (II)


Continuing from the previous year. The following information is relevant for the year
ended 31st December 2016.
(a) Capital transactions
Rs.
Depreciation charged 14,000
Tax allowances 16,000
(b) Interest payable
On 1st April 2016 the company issued Rs. 25,000 of 8% convertible loan stock.
Interest is paid in arrears on 30th September and 30th March. Assume that tax
relief on interest expense is only given when the interest is paid.
(c) Interest receivable
On 1st April Shep purchased debentures having a nominal value of Rs. 4,000.
Interest at 15% pa is receivable on 30th September and 30th March. Assume
that interest income is not taxed until the cash is actually received.
(d) Provision for warranty
In preparing the financial statements for the year to 31st December 2016, Shep
has recognised a provision for warranty payments in the amount of Rs. 1,200.
This has been correctly recognised in accordance with IAS 37 and the amount
has been expensed. Assume that tax relief on the warranty cost is only given
when the expense is paid.
(e) Fine
During the period Shep has paid a fine of Rs. 6,000. The fine is not tax
deductible.
(f) Further information
The accounting profit before tax for the year was Rs. 125,000.

Tax is chargeable at a rate of 30%.

Required
(a) Calculate the corporate income tax liability for the year ended 31st December
2016.
(b) Calculate the deferred tax balance that is required in the statement of financial
position as at 31st December 2016.
(c) Prepare a note showing the movement on the deferred tax account and thus
calculate the deferred tax charge for the year ended 31st December 2016
(d) Prepare the statement of profit or loss note which shows the compilation of the
tax expense for the year ended 31st December 2016.
(e) Prepare a note to reconcile the product of the accounting profit and the tax
rate to the tax expense for year ended 31st December 2016.

Emile Woolf International 80 The Institute of Chartered Accountants of Pakistan


Questions

12.4 SHEP (III)


Continuing from the previous year. The following information is relevant for the year
ended 31st December 2017.
(a) Interest payable/Interest receivable
Shep still has Rs. 25,000 of 8% convertible loan stack in issue and still retains
its holding in the debentures purchased in 2004.
(b) Provision for warranty
During the year Shep had paid out Rs. 500 in warranty claims and provided for
a further Rs. 2,000.
(d) Development costs
During 2017 Shep has capitalised development expenditure of Rs. 17,800 in
accordance with the provisions of IAS 38. Assume that tax relief on this
expenditure is taken in full in the period in which it is incurred.
(e) Further information
Rs.
Profit before taxation 175,000
Depreciation charged 18,500
Tax allowable depreciation 24,700

(f) Entertainment
Shep paid for a large office party during 2017 to celebrate a successful first
two years of the business. This cost Rs. 20,000. Assume that this expenditure
is not tax deductible.

Tax is chargeable at a rate of 30%.

Required
(a) Calculate the corporate income tax liability for the year ended 31st December
2017.
(b) Calculate the deferred tax balance that is required in the statement of financial
position as at 31st December 2017.
(c) Prepare a note showing the movement on the deferred tax account and thus
calculate the deferred tax charge for the year ended 31st December 2017
(d) Prepare the statement of profit or loss note which shows the compilation of the
tax expense for the year ended 31st December 2017.
(e) Prepare a note to reconcile the product of the accounting profit and the tax
rate to the tax expense for year ended 31st December 2017.

Emile Woolf International 81 The Institute of Chartered Accountants of Pakistan


Financial accounting and reporting II

12.5 SHEP (IV)


Using the information provided in Shep III and assume that Shep is subject to a
higher tax rate of 34% in 2017.

Required
(a) Calculate the corporate income tax liability for the year ended 31st December
2017.
(b) Calculate the deferred tax balance that is required in the statement of financial
position as at 31st December 2017.
(c) Prepare a note showing the movement on the deferred tax account and thus
calculate the deferred tax charge for the year ended 31st December 2017
(d) Prepare the statement of profit or loss note which shows the compilation of the
tax expense for the year ended 31st December 2017.
(e) Prepare a note to reconcile the product of the accounting profit and the tax
rate to the tax expense for year ended 31st December 2017.

12.6 WAQAR LIMITED


Waqar Limited has provided you the following information for determining its tax
and deferred tax expense for the year 2014 and 2015:
(i) During the year ended December 31, 2015, the companys accounting profit
before tax amounted to Rs. 40 million (2014: Rs. 30 million). The profit
includes capital gains amounting to Rs. 10 million (2014: Rs. 8 million) which
are exempt from tax.
(ii) The accounting written down values of the fixed assets, as at December 31,
2013 were as follows:

Accumulated Written down


Cost value
Depreciation

Rs. m Rs. m Rs. m

Machinery 200 25 175

Furniture and fittings 50 10 40

No additions or disposals of fixed assets were made in the years 2014 and
2015.
(iii) Machinery was acquired on January 1, 2013 and is being depreciated on
straight- line basis over its estimated useful life of 8 years. The tax base of
machinery as at December 31, 2013 was Rs. 90 million.
(iv) Furniture and fittings are also depreciated on the straight line basis at the
rate of 10% per annum. The tax base of furniture and fittings as at December
31, 2013 was Rs. 40.5 million.
(v) Normal rate of tax depreciation on both types of assets is 10% on written
down value.
(vi) The tax rates for 2013, 2014 and 2015 were 35%, 35% and 30% respectively.

Emile Woolf International 82 The Institute of Chartered Accountants of Pakistan


Questions

Required
For each year:
(a) Calculate the corporate income tax liability for the year.
(b) Calculate the deferred tax balance that is required in the statement of financial
position as at the year end.
(c) Prepare a note showing the movement on the deferred tax account and thus
calculate the deferred tax charge for the year.
(d) Prepare the statement of profit or loss note which shows the compilation of the
tax expense.
(e) Prepare a note to reconcile the product of the accounting profit and the tax
rate to the tax expense. (25)

12.7 SHAKIR INDUSTRIES


Given below is the statement of profit or loss of Shakir Industries for the year
ended December 31, 2015:
2015
Rs. m
Sales 143.00
Cost of goods sold (96.60)
Gross profit 46.40
Operating expenses (28.70)
Operating profit Other income 17.70
Profit before interest and tax 3.40
Financial charges 21.10
Profit before tax (5.30)
15.80
Following information is available:
(i) Operating expenses include an amount of Rs. 0.7 million paid as penalty to
SECP on non-compliance of certain requirements of the Companies
Ordinance, 1984.
(ii) During the year, the company made a provision of Rs. 2.4 million for
gratuity. The actual payment on account of gratuity to outgoing members was
Rs. 1.6 million.
(iii) Lease payments made during the year amounted to Rs. 0.65 million which
include financial charges of Rs. 0.15 million. As at December 31, 2015,
obligations against assets subject to finance lease stood at Rs. 1.2 million.
The movement in assets held under finance lease is as follows:
Rs. m
Opening balance 01/01/2015 2.50
Depreciation for the year (0.7)
Closing balance 31/12/2015 1.80

Emile Woolf International 83 The Institute of Chartered Accountants of Pakistan


Financial accounting and reporting II

(iv) The details of owned fixed assets are as follows:


Accounting Tax
Rs. m Rs. m
Opening balance 01/01/2015 12.50 10.20
Purchased during the year 5.30
5.3 5.30
5.3
Depreciation for the year (1.10)
(1.1) (1.65)
Closing balance 31/12/2015 5.30
16.70 5.30
13.85
(1.10) (1.65)
(v) Capital work-in-progress as on December 31, 2015 include financial charges
of Rs. 2.3 million which have been capitalised in accordance with IAS-23
Borrowing Costs. However, the entire financial charges are admissible,
under the Income Tax Ordinance, 2001.
(vi) Deferred tax liability and provision for gratuity as at January 1, 2015 was
Rs. 0.55 million and Rs. 0.7 million respectively.
(vii) Applicable income tax rate is 35%.

Required
Based on the available information, compute the current and deferred tax
expenses for the year ended December 31, 2015. (15)

12.8 MARS LIMITED


Mars Limited (ML) is engaged in the manufacturing of chemicals. On July 1, 2014
it obtained a motor vehicle on lease from a bank. Details of the lease agreement
are as follows:
(i) Cost of motor vehicle is Rs. 1,600,000.
(ii) Instalments of Rs. 480,000 are to be paid annually in advance.
(iii) The lease term and useful life is 4 years and 5 years respectively.
(iv) The interest rate implicit in the lease is 13.701%.
ML follows a policy of depreciating the motor vehicles over their useful life, on
the straight-line method. However, the tax department allows only the lease
payments as a deduction from taxable profits.
The tax rate applicable to the company is 30%. MLs accounting profit before tax
for the year ended June 30, 2015 is Rs. 4,900,000.
There are no temporary differences other than those evident from the
information provided above.

Required
(a) Prepare journal entries in the books of Mars Limited for the year ended June
30, 2015 to record the above transactions including tax and deferred tax.
(b) Prepare a note to the financial statements related to disclosure of finance
lease liability, in accordance with the requirements of IFRS. (18)
(Ignore comparative figures.)

Emile Woolf International 84 The Institute of Chartered Accountants of Pakistan


Questions

12.9 BILAL ENGINEERING LIMITED


Bilal Engineering Limited earned profit before tax amounting to Rs. 50 million
during the year ended December 31, 2015. The accountant of the company has
submitted draft accounts to the Finance Manager along with the following
information which he believes could be useful in determining the amount of taxation:
(i) Accounting deprecation for the year is Rs. 10 million which includes Rs. 1
million charged on the difference between cost and revalued amount.
(ii) A motor vehicle costing Rs. 1 million was taken on lease in 2014. Related
clauses of the lease agreement are as under:
Annual instalment of Rs. 0.3 million is payable annually in advance.
The lease term and useful life is 4 years and 5 years respectively.
The interest rate implicit in the lease is 13.701% per annum.
Accounting depreciation on the leased vehicle is included in the
depreciation referred to in para (i) above.
(iii) Tax depreciation on the assets owned by the company is Rs. 7 million.
(iv) Research and development expenses of Rs. 15 million were incurred in 2013
and are being amortised over a period of 15 years. For tax purposes research
and development expenses are allowed to be written off in 10 years. However,
10% of these expenses were not verifiable and have not been claimed.
(v) Expenses amounting to Rs. 0.25 million were disallowed in 2012. Out of
these Rs. 0.15 million were allowed in appeal, during the current year. The
company had initially expected that the full amount would be allowed but has
decided not to file a further appeal.
(vi) The applicable tax rate is 35%.

Required
(a) Prepare journal entries in respect of taxation, for the year ended December 31,
2015.
(b) Prepare a reconciliation to explain the relationship between tax expense and
accounting profit as is required to be disclosed under IAS 12 Income Taxes.
(18)

12.10 GALAXY INTERNATIONAL


The following information relates to Galaxy International (GI), a listed company,
which was incorporated on January 1, 2014.
(i) The (loss) / profit before taxation for the years ended December 31, 2014 and
2015 amounted to (Rs. 1.75 million) and Rs. 23.5 million respectively.
(ii) The details of accounting and tax depreciation on fixed assets is as follows:
2015 2014
Rs. m Rs. m
Accounting depreciation 15 15
Tax depreciation 6 45
(iii) In 2014, GI accrued certain expenses amounting to Rs. 2 million which were
disallowed by the tax authorities. However, these expenses are expected to
be allowed on the basis of payment in 2015.

Emile Woolf International 85 The Institute of Chartered Accountants of Pakistan


Financial accounting and reporting II

(iv) GI earned interest on Special Investment Bonds amounting to Rs. 1.0 million
and Rs. 1.25 million in the years 2014 and 2015 respectively. This income is
exempt from tax.
(v) GI operates an unfunded gratuity scheme. The provision during the years
2014 and 2015 amounted to Rs. 1.7 million and Rs. 2.2 million respectively.
No payment has so far been made on account of gratuity.
(vi) The applicable tax rate is 35%.

Required
Prepare a note on taxation for inclusion in the companys financial statements for
the year ended December 31, 2015 giving appropriate disclosures relating to
current and deferred tax expenses including a reconciliation to explain the
relationship between tax expense and accounting profit. (20)

12.11 APRICOT LIMITED


The following information relates to Apricot Limited (AL), a listed company, for the
financial year ended 31 December 2015:
(i) The profit before tax for the year amounted to Rs. 60 million (2014: Rs. 45
million).
(ii) The accounting and tax written down value of fixed assets as on 31 December
2014 was Rs. 95 million and Rs. 90 million respectively. Accounting
depreciation for the year is Rs. 10 million (2014: Rs. 9 million) whereas tax
depreciation for the year is Rs. 8 million (2014: Rs. 7 million).
(iii) During the year, AL sold a machine for Rs. 3 million and recognised a profit of
Rs. 0.5 million. The tax written down value of the machine as on 31
December 2014 was Rs. 2 million. There were no other additions/disposals of
fixed assets in 2014 and 2015.
(iv) AL earned capital gain of Rs. 6 million (2014:Nil) on sale of shares of a listed
company. This income is exempt from tax.
(v) Bad debt expenses recognised during the year was Rs. 5 million (2014: Rs. 7
million).
(vi) Bad debts written off during the year amounted to Rs. 3 million (2014: Rs. 4
million).
(vii) Deferred tax liability and provision for bad debts as on 31 December
2011 was Rs. 18.90 million and Rs. 9 million respectively.
(viii) The companys assessed brought forward losses up to 31 December 2011
amounted to Rs. 19.25 million.
(ix) Applicable tax rate is 35%.

Required
Prepare a note on taxation for inclusion in ALs financial statements for the year
ended 31 December 2015 giving appropriate disclosures relating to current and
deferred tax expenses including comparative figures for 2014 and a reconciliation to
explain the relationship between 2015 tax expense and 2015 accounting profit.
(21)

Emile Woolf International 86 The Institute of Chartered Accountants of Pakistan


Questions

CHAPTER 13 RATIO ANALYSIS

13.1 WASIM
Wasim is an importer and retailer of vegetable oils. Extracts from the financial
statements for this year and last are set out below.
Income statements for the years ended 30 September
Year 7 Year 6
Rs.000 Rs.000
Revenue 1,806
2,160
Cost of sales (1,755) (1,444)

Gross profit 405 362
Distribution costs (130) (108)
Administrative expenses (260) (198)

Profit before tax 15 56
Income tax expense (6) (3)

Profit for the period 9 53

Statements of financial position as of 30 September


Year 7 Year 6
Rs.000 Rs.000
Assets
Non-current assets
Property, plant and equipment 78 72

Current assets
Inventories 106 61
Trade receivables 316 198
Cash - 6

422 265

Total assets 500 337

Equity and liabilities
Equity
Ordinaryshares 110 85
Preference shares 23 11
Share premium 15 -
Revaluation reserve 20 20
Retained earnings 78 74
246 190
Current liabilities
Bank overdraft 49 -
Trade payables 198 142
Current tax payable 7 5

254 147

Total equity and liabilities 500 337

Emile Woolf International 87 The Institute of Chartered Accountants of Pakistan


Financial accounting and reporting II

Required
Define and calculate the following ratios:
a) Gross profit percentage.
b) Net profit percentage
c) Return on capital employed
d) Asset turnover
e) Current ratio
f) Quick ratio
g) Average receivables collection period
h) Average payables period
i) Inventory turnover

13.2 AMIR AND MO


The income statements and statements of financial position of two manufacturing
companies in the same sector are set out below.
Amir Mo
Rs. Rs.
Revenue 150,000 700,000
Cost of sales (60,000) (210,000)

Gross profit 90,000 490,000
Interest payable (500) (12,000)
Distribution costs (13,000) (72,000)
Administrative expenses (15,000) (35,000)

Profit before tax 61,500 371,000
Income tax expense (16,605) (100,170)

Profit for the period 44,895 270,830

Assets
Non-current assets
Property - 500,000
Plant and equipment 190,000 280,000


190,000 780,000
Current assets
Inventories 12,000 26,250
Trade receivables 37,500 105,000
Cash at bank 500 22,000

50,000 153,250

Total assets 240,000 933,250

Emile Woolf International 88 The Institute of Chartered Accountants of Pakistan


Questions

Equity and liabilities


Equity
Share capital 156,000 174,750
Retained earnings 51,395 390,830

207,395 565,580
Non-current liabilities
Long-term debt 10,000 250,000

Current liabilities
Trade payables 22,605 117,670

Total equity and liabilities 240,000 933,250

Required
Define and calculate the following ratios for each company:
a) Gross profit percentage.
b) Net profit percentage
c) Return on capital employed
d) Asset turnover
e) Current ratio
f) Quick ratio
g) Average receivables collection period
h) Average payables period
i) Inventory turnover

Emile Woolf International 89 The Institute of Chartered Accountants of Pakistan


Financial accounting and reporting II

CHAPTER 14 ETHICAL ISSUES IN FINANCIAL REPORTING

14.1 ETHICAL ISSUES


Waheed is a chartered accountant, recently employed by AA plc as deputy to the
finance director, Arif (also a chartered accountant). AA plc is listed on the Lahore
stock exchange.
On Waheeds first day on the job he met with Arif who said Look, keep it to yourself
but Im having a second interview next week for a new job. The first thing that I need
you to do is to review the financial statements before the auditors arrive. I qualified a
few years ago and am not up to date on all of the little technicalities in IFRS. You
should now these better than me and youll know more about what the auditors
might focus on. We must do our best to present the financial statements in the most
favourable light as the bonus paid to employees (including me) depends on profit
being more than 10% bigger than last years and remember that you qualify for this
too. Keep this in mind when you carry out the review as we do not really want to find
anything. Do well at this and I might put in a good word for you when I leave as Im
sure youll be a great replacement for me.

Required
Explain the ethical issues inherent in the above conversation and what Waheed
should do about them.

14.2 SINDH INDUSTRIES LTD


Jafar has recently been appointed as financial controller to Sindh Industries Ltd.
Until a month ago, Sindh Industries had a finance director, who resigned suddenly,
due to ill health. Since Jafar joined the company, he has learned that his resignation
was related to stress caused by a series of disagreements with the managing
director about the performance of the business.. The directors have not yet
appointed a replacement.
It is now March 2016 and you have been asked to finalise the financial statements
for the year ended 31 December 2015. The draft statement of profit or loss extract
and statement of financial position are shown below:
Draft statement of profit or loss for the year ended 31 December 2015
Rs. 000
Profit before tax 2,500

Draft statement of financial position at 31 December 2015


Rs. 000
Property, plant and equipment 12,000
Current assets 3,500
Total assets 15,500

Share capital 2,000


Retained earnings 6,000
Equity 8,000
Non-current liabilities 5,000
Current liabilities 2,500
Total equity and liabilities 15,500

Emile Woolf International 90 The Institute of Chartered Accountants of Pakistan


Questions

During the year ended 31 December 2015 Sindh Industries entered into the
following transactions.

(1) Just before the year end Sindh Industries signed a contract to deliver
consultancy services for a period of 2 years at a fee of Rs. 500,000 per
annum. The full amount of this fee has been paid in advance and is non-
refundable.

(2) Sindh Industries has constructed a new factory. The construction has been
financed from the pool of existing borrowings. Land at a cost of Rs. 1.8 million
was acquired on 1 February 2015 and construction began on 1 June 2015.
Construction was completed on 30 September 2015 at an additional cost of
Rs. 2.7 million. Although the factory was usable from that date, full production
did not commence until 1 December 2015. Throughout the year the
companys average borrowings were as follows:
Annual
interest
Amount rate
Rs. %
Bank overdraft 1,000,000 9.75
Bank loan 1,750,000 10
Debenture 2,500,000 8
An amount of Rs. 450,000 has been included in property, plant and equipment
in respect of borrowing costs relating to the construction of the factory. The
useful life of the factory has been estimated at 20 years. No depreciation has
been charged for the year. The reason for this is that the factory has only been
in use for one month and that the depreciation charge would be immaterial.

(3) A blast furnace with a carrying amount at 1 January 2015 of Rs. 3.5 million
has been depreciated in the draft financial statements on the basis of a
remaining life of 20 years. In December 2015 the directors carried out a review
of the useful lives of various significant items of plant and machinery, including
the blast furnace and came to the conclusion that the useful life of the furnace
was 20 years at 31 December 2015. The reasoning behind this judgement
was that the lining of the furnace had been replaced in the last week of
December 20X6 at a cost of Rs. 1.4 million. Provided that the lining is
replaced every five years, the life of the furnace can be extended accordingly.
You have found a report, commissioned by the previous finance director and
prepared by a firm of asset valuation specialists, which assesses the
remaining useful life of the main structure of the furnace at 1 January 2015 at
15 years and the lining of the furnace at 5 years. You have also found
evidence that the managing director has seen this report.

Jafar has had a conversation with the managing director who told him, We
need to make the figures look as good as possible so I hope youre not going
to start being difficult. The consultancy fee is non-refundable so theres no
reason why we cant include it in full. I think we should look at our depreciation
policies. Were writing off our assets over far too short a period. As you know,
were planning to go for a stock market listing in the near future and being
prudent and playing safe wont help us do that. It wont help your future with
this company either.

Emile Woolf International 91 The Institute of Chartered Accountants of Pakistan


Financial accounting and reporting II

Required

(a) Explain the required IFRS accounting treatment of these issues, preparing
relevant calculations where appropriate. (17)

(b) Prepare a revised draft of the statement of profit or loss extract for the year
ended 31 December 2015 and the statement of financial position at that date.
(6)

(c) Discuss the ethical issues arising from your review of the draft financial
statements and the actions that you should consider. (5)

Emile Woolf International 92 The Institute of Chartered Accountants of Pakistan


Certificate in Accounting and Finance

B
Financial accounting and reporting II

SECTION
Answers

Emile Woolf International 93 The Institute of Chartered Accountants of Pakistan


Financial accounting and reporting II

CHAPTER 2 IAS 1: PRESENTATION OF FINANCIAL STATEMENTS

2.1 LARRY
Statement of profit or loss
For the year ended 31 December 2015
Rs. in
million
Revenue 3,304
Cost of sales (2,542 + 118 127) (2,533)
Gross profit 771
Other income 20
Distribution costs (175)
Administrative expenses (342)
Profit before tax 274
Income tax expense (75)
Profit for the period 199

Statement of financial position


As at 31 December 2015
Assets Rs. in
million
Non-current assets
Property, plant and equipment (2,830 918) 1,912
Intangible assets (26 5) 21
1,933
Current assets
Inventories 127
Trade and other receivables 189
Cash (89 +2) 91
407
Total assets 2,340
Equity and liabilities
Equity
Share capital 400
Retained earnings (1,562 + 199) 1,761
2,161
Non-current liabilities
Long-term borrowings (18 x 2/3) 12
Current liabilities
Trade and other payables 86
Current portion of long-term borrowing (18 3) 6
Current tax payable 75
167
Total equity and liabilities 2,340

Emile Woolf International 94 The Institute of Chartered Accountants of Pakistan


Answers

2.2 MINGORA IMPORTS LIMITED


Statement of profit or loss for the year ended 31 December 2015
Rs. in
million
Revenue 1,740
Change in inventories of finished goods and work-in-progress (W3) 40
Staff costs (W3) (620)
Depreciation and other amortisation expense (W3) (42)
Other expenses (W3) (359)
Profit before tax 759
Income tax expense (120)
Profit for the period 639
Statement of financial position as at 31 December 2015
Assets Rs. in
million
Non-current assets
Property, plant and equipment (W1) 368
Intangible assets (W2) 40
408
Current assets
Inventories (180 + 140) 320
Trade and other receivables (420 x 95%) 399
Cash 440
1,159
Total assets 1,567
Equity and liabilities
Equity
Share capital 600
Other reserves 120
Retaind earnings 635
1,355
Current liabilities
Trade and other payables 92
Current tax payable 120
212
Total equity and liabilities 1,567
Statement of changes in equity for the year ended 31 December 2015
Amounts in Rs. million
Share Revaluati Retained
capital on earnings Total
reserve
Balance at 31 December 2014 620 121 721
Dividends paid (125) (125)
Net revaluation surplus in the 120 - 120
year (360 (300 60))
Profit after tax for the period - - 639 639
Balance at 31 December 2015 620 120 635 1,355

Emile Woolf International 95 The Institute of Chartered Accountants of Pakistan


Financial accounting and reporting II

Workings
(1) Property, plant and equipment
Rs. in
million
Cost brought forward
Leasehold 300
Computers 50
Revaluation 60
Cost carried forward 410
Accumulated depreciation brought forward (60 + 20) 80
Revaluation (60)
Charge for the year
Leasehold (360 30) 12
Computers (50 5) 10
Accumulated depreciation carried forward 42
Carrying amount carried forward 368
(2) Intangible assets
Rs. in
million
Cost 60
Amortisation (60 3) (20)
Carried forward 40
(3) Allocation of costs
Amounts in Rs. million
Change
in Staff Depreciat Other
inventori costs ion etc expenses
es
Work-in-progress (140 (15)
125)
Staff costs 260
Finished goods (180 (25)
155)
Consultancy fees 44
Directors salaries 360
Doubtful receivables (420 21
5%)
Sundry 294
Amortisation of patent 20
(W2)
Depreciation (12 + 10) 22
(W1)
(40) 620 42 359

Emile Woolf International 96 The Institute of Chartered Accountants of Pakistan


Answers

2.3 BARRY
Barry
Statement of profit or loss
For the year ended 31st August 2015
Rs. in
million
Revenue 30,000
Cost of sales (W1) (19,650)
Gross profit 10,350
Distribution costs (W1) (1,370)
Administrative expenses (W1) (1,930)
Profit from operations 7,050
Finance costs (350)
Profit before tax 6,700
Tax (W2) (2,500)
Profit after tax 4,200
Barry
Statement of financial position
As at 31st August 2015
Rs. in
million
ASSETS
Non-current assets
Property, plant and equipment 39,600
Current assets
Inventory 4,600
Trade and other receivables (7,400 + 200) 7,600
Cash and cash equivalents 700
12,900
Total assets 52,500
EQUITY AND LIABILITIES
Capital and reserves
Equity shares 21,000
Share premium 2,000
Accumulated profits (W3) 11,800
Total equity 34,800
Revaluation reserve (W4) 4,700
Non current liabilities
Borrowings 5,200
Current liabilities
Trade and other payables 5,300
Taxation (2,100 + 400) 2,500
7,800
Total equity and liabilities 52,500

Emile Woolf International 97 The Institute of Chartered Accountants of Pakistan


Financial accounting and reporting II

Reconciliation of opening and closing property, plant and equipment


Rs. in 000
Assets
Plant & Fixtures
under
Land Buildings machine & Total
construct
ry fittings
ion
Cost/ Valuation
At 1 Sept 2014 10,000 9,000 20,100 10,000 400 49,500
Additions - - - - 50 50
Reclassification - - 450 - (450) -
Revaluation 1,000 1,000 - - - 2,000

At 31 Aug 2015 11,000 10,000 20,550 10,000 - 51,550

Depreciation
At 1 Sept 2014 - 3,000 4,000 3,700 - 10,700
Revaluation - (3,000) - - - (3,000)
Charge for year - 1,000 2,550 700 - 4,250

At 31 Aug 2015 - 1,000 6,550 4,400 - 11,950

Net book value


At 31 Aug 2015 11,000 9,000 14,000 5,600 - 39,600

At 1 Sept 2014 10,000 6,000 16,100 6,300 400 38,800

Workings

1 Allocation of expenses Rs.in 000


Cost of Admin Distrib
sales
Raw materials consumed 9,500
Manufacturing overheads 5,000
Increase in inventories (1,400)
Staff costs (70%/20%/10%) 3,290 940 470
Distribution costs 900
Depreciation
Building (50%/50%) 500 500
Plant and machinery 2,550
Fixtures and fittings (30%/70%) 210 490
19,650 1,930 1,370
2 Tax charge
Rs. in
000
Current year 2,100
Under provision from previous year 400
2,500

Emile Woolf International 98 The Institute of Chartered Accountants of Pakistan


Answers

3 Accumulated profits carried forward


Rs. in 000
Accumulated profits carried forward per question 14,000
Less tax charge
- Current year estimate 2,100
- Under-provision in previous year 400
(2,500)
Add transfer of excess depreciation on revalued building 300
11,800
4 Revaluation reserve carried forward
Revaluation reserve per question 5,000
Add transfer of excess depreciation on revalued building (300)
4,700
2.4 OSCAR INC
(a) Statement of profit or loss
For the year ended 31 March 2015
Rs. in 000
Sales 2,010
Operating costs (140 + 960 150 + 420 + 210 + 16) (1,596)

Operating profit before interest 414
Income from investments 75

Profit before taxation 489
Income tax (49)

440

Statement of financial position
As at 31 March 2015
Assets
Non-current assets
Tangible assets 530
Investments 560

1,090
Current assets
Inventory 150
Receivables 470

620

1,710

Equity and liabilities
Capital and reserves
Share capital 600
Retained earnings 500

1,100
Current liabilities 414
Provisions for liabilities and charges 196

1,710

Emile Woolf International 99 The Institute of Chartered Accountants of Pakistan


Financial accounting and reporting II

Workings

(1) Income tax


Rs. in 000
Income tax (current year) 74
Over provision for tax in the previous year (25)

49

(2) Tangible assets plant and machinery
Rs. in 000
Cost at 1 April 2014 and 31 March 2015 750

Accumulated depreciation
At 31 March 2014 188
Provided during the year (27 + 5) 32

At 31 March 2015 220

Net book value at 31 March 2015 530

(3) Current liabilities


Rs. in 000
Trade payables 260
Mainstream corporation tax 74
Bank overdraft 80

414

(4) Provisions for liabilities and charges
Rs. in 000
At 1 April 2014 180
Provided in the year 16

At 31 March 2015 196

(5) Retained earnings


Rs. in 000
Retained earnings 440
Opening retained earnings 180
Dividends (120)

Closing retained earnings 500

Emile Woolf International 100 The Institute of Chartered Accountants of Pakistan


Answers

2.5 CLIFTON PHARMA LIMITED


(a) Clifton Pharma Limited
Statement of profit or loss for the year ended 30 September 2015
Rs. in
000
Revenue 338,300
Cost of sales: see working (1) (180,000)
Gross profit 158,300
Operating expenses: see working (2) (36,600)
Investment income 2,000
Finance costs: Loan notes see working (3) (3,000)
Finance lease see working (2) (1,700)
(4,700)
Profit before tax 119,000
Income tax expense: see working (4) (21,000)
Profit for the period 98,000

(b) Clifton Pharma Limited


Statement of financial position as at 30 September 2015
Non-current assets
Property, plant and equipment: see working (5) 358,000
Investments 92,400
450,400
Current assets
Inventory 23,700
Trade receivables 76,400
Bank 12,100
112,200
Total assets 562,600
Equity and liabilities
Capital and reserves
Share capital 280,000
Share premium 20,000
Retained earnings: see working (6) 117,300
417,300
Revaluation surplus 20,000
Non-current liabilities
3% loan notes: see working (3) 51,500
Deferred tax: see working (4) 23,000
Finance lease obligation: see working (2) 11,700
86,200
Current liabilities
Trade payables 14,100
Accrued lease finance costs: see working (2) 1,700
Finance lease obligation: see working (2) 5,300
Income tax payable 18,000
39,100
Total equity and liabilities 562,600

Emile Woolf International 101 The Institute of Chartered Accountants of Pakistan


Financial accounting and reporting II

Workings
(1) Cost of sales Rs. in 000
As given in the trial balance 134,000
Depreciation of plant and equipment: 20% (197,000 30,000
47,000)
Depreciation of leased vehicles: 24,000/4 years 6,000
Amortisation of leasehold property: 250,000/25 years 10,000
180,000

(2) Vehicle rentals and finance lease. Operating expenses


Rental costs given in the trial balance 8,600
Relating to finance lease (7,000)
Balance: relating to operating lease operating expense 1,600
Other operating expenses (trial balance in question) 35,000
Total operating expenses 36,600

Finance lease
Fair value of leased assets 24,000
Less: First rental payment, paid in advance (7,000)
1 October 2014
Remaining obligation, 1 October 2014 17,000
Interest at 10% to 30 September 2015 (current liability) 1,700
Lease payment due 1 October 2015 7,000
Capital repayment due (= balance, current liability) (5,300)
Remaining lease obligation = non-current liability 11,700
(3) Loan notes
The effective interest rate is 6%. Actual interest paid was Rs.1,500,000
(in trial balance); therefore the balancing Rs.1,500,000 should be added
to the loan notes obligation, to make the total loan notes liability Rs.50
million + Rs.1,500,000 = Rs.51.5 million.
(4) Taxation
Deferred tax liability b/f 20,000
Deferred tax: credit in the statement of profit or loss 2,000
Deferred tax liability c/f (92,000 25%) 23,000
Tax expense
Income tax on profits for the year 18,000
Deferred tax movement 3,000
Tax charge in the statement of profit or loss 21,000

Emile Woolf International 102 The Institute of Chartered Accountants of Pakistan


Answers

(5) Non-current assets and depreciation


Leasehold property Rs. in 000
Carrying value in the trial balance (250,000 40,000) 210,000
Amortisation charge for the year to 30 September 2015 (10,000)
200,000
Re-valued amount 220,000
Transfer to revaluation reserve 20,000

The annual depreciation charges for plant and equipment and the leased
vehicles are shown in workings (1) Rs. in 000
Cost or Accumulated Carrying
valuation depreciation amount
Leasehold property 220,000 0 220,000
Plant and equipment 197,000 77,000 120,000
(non-leased)
Leased vehicles 24,000 6,000 18,000
441,000 83,000 358,000

(6) Retained profits


At 1 October 2014 (trial balance) 19,300
Profit for the year 98,000
Retained profits at 30 September 2015 117,300

2.6 SARHAD SUGAR LIMITED


(a) Sarhad Sugar Limited
Statement of profit or loss for the year ended 30 September 2015
Revenue (300,000 2,500) 297,500
Cost of sales (w (i)) (225,400)
Gross profit 72,100
Distribution costs (14,500)
Administrative expenses (22,200 400 + 100 see note below) (21,900)
Finance costs (1,000)
Profit before tax 34,700
(Income tax expense (11,400 + (6,000 5,800 deferred tax)) (11,600)
Profit for the year 23,100

(b) Sarhad Sugar Limited


Statement of financial position as at 30 September 2015
Assets
Non-current assets (w (ii))
Property, plant and equipment (43,000 + 38,400) 81,400
Development costs 14,800
96,200
Current assets
Inventory 20,000
Trade receivables 43,100

Emile Woolf International 103 The Institute of Chartered Accountants of Pakistan


Financial accounting and reporting II

63,100
Total assets 159,300
Equity and liabilities:
Equity
Share capital 70,000
Retained earnings (w (iii)) 41,600
117,100
Revaluation reserve (w (iii)) 5,500
Non-current liabilities
Deferred tax 6,000
Current liabilities
Trade payables (23,800 400 + 100 re legal action) 23,500
Bank overdraft 1,300
Current tax payable 11,400
36,200
Total equity and liabilities 159,300
Note: As it is considered that the outcome of the legal action against Sarhad
Sugar Limited is unlikely to succeed (only a 20% chance) it is inappropriate to
provide for any damages. The potential damages are an example of a
contingent liability which should be disclosed (at Rs.2 million) as a note to the
financial statements. The unrecoverable legal costs are a liability (the start of
the legal action is a past event) and should be provided for in full.
Workings (figures in brackets in Rs.000)
(i) Cost of sales: Rs. in 000
Per trial balance 204,000
Depreciation (w (iii)) leasehold property 2,500
plant and equipment 9,600
Loss on disposal of plant (4,000 2,500) 1,500
Amortisation of development costs (w (iii)) 4,000
Research and development expensed (1,400 + 2,400 (w (iii)) 3,800

225,400

(ii) Non-current assets:
Leasehold property
Valuation at 1 October 2014 50,000
Depreciation for year (20 year life) (2,500)

Carrying amount at date of revaluation 47,500
Valuation at 30 September 2015 (43,000)

Revaluation deficit 4,500

Plant and equipment per trial balance (76,600 24,600) 52,000


Disposal (8,000 4,000) (4,000)

48,000
Depreciation for year (20%) (9,600)

Carrying amount at 30 September 2015 38,400

Emile Woolf International 104 The Institute of Chartered Accountants of Pakistan


Answers

Capitalised/deferred development costs Rs. in 000


Carrying amount at 1 October 2014 (20,000 6,000) 14,000
Amortised for year (20,000 x 20%) (4,000)
Capitalised during year (800 x 6 months) 4,800

Carrying amount at 30 September 2015 14,800

Note: development costs can only be treated as an asset from the point
where they meet the recognition criteria in IAS 38 Intangible assets.
Thus development costs from 1 April to 30 September 2015 of Rs.48
million (800 x 6 months) can be capitalised. These will not be amortised
as the project is still in development.
The research costs of Rs.14 million plus three months development
costs of Rs.24 million (800 x 3 months) (i.e. those incurred before 1
April 2015) are treated as an expense.
(iii) Movements on reserves
Revaluation Retained
surplus earnings
Rs. in 000
Balances at 1 October 2014 10,000 24,500
Dividend (6,000)
Comprehensive income 23,100
Revaluation loss (4,500)
Balances at 30 September 2015 5,500 41,600

2.7 BSZ LIMITED


BSZ Limited
Statement of financial position as at June 30, 2015
Note Rs. in
million
ASSETS
Fixed Assets
Property, plant & equipment 1 576
Intangible assets 2 8
584
Long term advances considered good 4
Current assets
Stocks in trade 90
Accounts receivable 3 57
Advances, deposits, prepayments and other
receivables 4 45
Cash at banks 5 29
221
809

Emile Woolf International 105 The Institute of Chartered Accountants of Pakistan


Financial accounting and reporting II

Rs. in
million
EQUITY AND LIABILITIES
Share capital and reserves
Authorized share capital
50,000,000 shares of Rs. 10 each 500

Issued, subscribed and paid up capital


40,000,000 shares of Rs. 10 each 400
Unappropriated profit 65
465
Surplus on revaluation of fixed assets 120
Non-current liabilities
Deferred taxation 40
Current liabilities
Short term loan 85
Account and other payables 6 82
Provision for taxation 17
184
809

Rs. in
Notes million
1. Property, plant and equipment
Operating assets 556
Capital work in progress building 20
576

1.1 Operating assets Rs. in million


Freehold
Cost/revalued amount land Building Machines Fixtures Total
As of July 01 2014 375.0 130.0 100.0 19.0 624.0
Additions - - - 8.0 8.0
Disposals - - (15.0) - (15.0)
As at June 30 2015 375.0 130.0 85.0 27.0 617.0

Accumulated
depreciation
As of July 01 2014 - 19.5 22.5 5.9 47.9
For the year - 6.5 18.1
(105 85) + 10% 15 9.5
8
/12)
(105 19) + 10% 8 3/12) 2.1
Disposals - - (5.0) - (5.0)
As at June 30 2015 - 26.0 27.0 8.0 61.0
Carrying amount 375.0 104.0 58.0 19.0 556.0
Depreciation rate - 5% 10% 10%

Emile Woolf International 106 The Institute of Chartered Accountants of Pakistan


Answers

1.2 Revaluation
During the year 2011, the first revaluation of freehold land was carried out.
The valuation was carried out under market value basis by an independent
valuer, Mr. Dee, Chartered Civil Engineer of M/s SSS Consultants (Pvt.)
Ltd., Islamabad. It resulted in a surplus of Rs. 120 million over book values
which was credited to surplus on revaluation of fixed assets. Had there
been no revaluation, the value of freehold land would be Rs. 255 million.

1.3 Disposal of machine


Rs. in
million
Proceeds 13.0
Cost 15.0
Accumulated depreciation (5.0)
Carrying amount (10.0)
Profit on disposal 3.0

Note 2015
Rs. in
million
2. Intangible Assets
Cost of computer software/license 10.0
Accumulated Amortization as of July 1, 2014 1.0
Amortization for the year 1.0
Accumulated Amortization as of June 30, 2015 2.0
Carrying value as at June 30, 2015 8.0
Amortization rate 10%

3. Accounts Receivable
Considered good
- Secured 30
- Unsecured 27
57
Considered doubtful 3
60
Less: Provision for bad debts 3.1 3
57

3.1 Provision for bad debts


Balance as at July 1, 2014 3.4
Provision made during the year 1.0
Amount written off during the year (1.4)
Balance as at June 30, 2015 (Rs. 30 million x 10%) 3.0

4 Advances, Deposits, Prepayments and Other Receivables


Advances
- suppliers - considered good 12
- staffs 6
18
Deposits 11
Prepayments 4
Sales tax receivable 12
45

Emile Woolf International 107 The Institute of Chartered Accountants of Pakistan


Financial accounting and reporting II

5 Cash at banks
Cash at banks - current accounts 7
saving accounts 5.1 22
29
5.1: It carries interest / mark up ranging from 3% to 7% per annum.

6 Accounts and other payables


Accounts payable 75
Accrued liabilities 7
82

2.8 YASIR INDUSTRIES LIMITED


Yasir Industries Limited
Statement of Financial Position as at June 30, 2015
Assets Rs. in
million
Non-current assets
Property, plant and equipment (W2) 351.00
Intangible assets (20 12) 8.00
359.00
Current assets
Inventories (W6) 64.50
Trade receivables (W5) 39.00
103.50
462.50
Equity and Liabilities
Equity
Issued, subscribed and paid up capital 120.00
Retained earnings (W4) 87.10
207.10

Revaluation surplus 41.25

Non-current liabilities
Redeemable preference shares 40.00
Debentures 80.00
Deferred taxation (W 10) 9.00
129.00
Current liabilities
Trade payables 30.40
Accrued expenses (W3) 25.00
Taxation 16.50
Bank overdraft 13.25
85.15
Total equity and liabilities 462.50

Emile Woolf International 108 The Institute of Chartered Accountants of Pakistan


Answers

Yasir Industries Limited


Statement of profit or loss for the year ended June 30, 2015
Rs. in
million
Sales revenue (W5) 445.40
Cost of sales (W7) (250.72)
Gross profit 194.68
Distribution costs (W8) (20.05)
Administrative expenses (W8) (40.38)
Financial charges (W9) (9.10)
125.15
Loss due to fraud (30.00)
Profit before tax 95.15
Income tax expense (W10) (19.50)
Profit for the year 75.65
Workings
(W1) Leasehold property
Annual depreciation before the revaluation (230 40 years) = Rs. 5.75 million per
annum.
Depreciation this year has been charged incorrectly on cost (whereas it should
have been on the revalued amount).
This years charge must be added back
Dr Cr
Accumulated depreciation 5.75
Cost of sales (50%) 2.88
Administrative expenses (30%) 1.72
Distribution costs (20%) 1.15

Rs. in
million
Carrying amount at the 30 June (as per trial balance)(230.00 40.25) 189.75
Add back depreciation incorrectly charged (see above) 5.75
Carrying amount of property at the start of the year 195.5

Revaluation surplus Rs. in


million
Revalued amount of leasehold property 238.00
Less: WDV of leasehold property at revaluation 195.50
Revaluation surplus arising in the year 42.50
Transfer to retained earnings in respect of incremental depreciation
(Rs. 7 million Rs. 5.75 million) (1.25)
41.25

Depreciation of revalued property


Number of years depreciation by the year end: (40.25 5.75) = 7 years.
Therefore, remaining useful life as at the year-end = 33 years
Revaluation was at the start of the year
Remaining useful life at the start of the year = 34 years

Depreciation charge based on the revalued amount (238/34 years) = Rs. 7 million

Emile Woolf International 109 The Institute of Chartered Accountants of Pakistan


Financial accounting and reporting II

Dr Cr
Cost of sales (50%) 3.5
Administrative expenses (30%) 2.1
Distribution costs (20%) 1.4
Accumulated depreciation 7.00

(W2) Property, plant and equipment


Rs. in
million
Leasehold property (Rs. 238m 7) 231
Machines (Rs. 168.6 Rs. 48.6m) 120
351
(W3) Accrued Expenses
Rs. in
million
As per trial balance 15.00
Accrued interest on debentures (Rs. 80m 12% 6/12) 4.80
Dividend on preference shares (Rs. 40m 10%) 4.00
23.80
(W4) Retained earnings
Rs. in
million
Balance as per trial balance 10.20
Profit for the year 75.65
Transfer from revalution surplus 1.25
87.10
(W5) Sales and receivables
Sales. Rec.
Rs. in Rs. m
million
Given in the trial balance 478.40 66.00
Deduct revenue incorrectly recognised (sale or return) (27.00) (27.00)
Cost of sales 451.40 39.00
(W6) Closing inventory
Rs. in
million
Given in the question 42.00
Add back inventory held by customer on sale or return (100/120 27) 22.50
Cost of sales 64.50
(W7) Cost of sales
Rs. in
million
Opening inventory as of July 1, 2014 38.90
Purchases 175.70
Direct labour 61.00
Manufacturing overheads excluding incremental depreciation 39.00
Less: Closing inventory (64.50)
Deduct depreciation incorrectly charged on cost (2.88)
Add depreciation charged on revalued amount 3.50
Cost of sales 250.10

Emile Woolf International 110 The Institute of Chartered Accountants of Pakistan


Answers

(W8) Administrative expenses and distribution costs


Admin. DIst/
Rs. in Rs. m
million
Given in the trial balance 40.00 19.80
Deduct depreciation incorrectly charged on cost (1.72) (1.15)
Add depreciation charged on revalued amount 2.10 1.40
Cost of sales 40.38 20.05
(W9) Financial charges
Rs. in
million
Balance as per trial balance 0.30
Accrued interest on debentures (Rs. 80m 12% 6/12) 4.80
Preference dividend for the year (Rs. 40m 10%) 4.00
9.10
(W10) Taxation
Deferred taxation Rs. in
million
Balance b/f 6.00
Charge for the year (balancing figure) 3.00
Balance c/f (30% Rs. 30 million temporary difference) 9.00

Tax expense Rs. in


million
Current tax 16.50
Deferred tax (see above) 3.00
19.50

Emile Woolf International 111 The Institute of Chartered Accountants of Pakistan


Financial accounting and reporting II

2.9 SHAHEEN LIMITED


Shaheen Limited
Statement of financial position
As of June 30, 2015
Assets Rs. in 000
Non-current assets
Property, plant and equipment (86,000 12,000 4,500) 69,500
Intangible assets (6,000 600) 5,400
74,900
Current assets
Stock in trade 30,000
Trade receivables (37,800 10,000) 27,800
Other receivables and prepayments (14,000 + 6,000) 20,000
Cash and bank balances 4,725
82,525
157,425
Equity and liabilities
Share capital and reserves
issued, subscribed and paid up capital 60,000
Unappropriated profit 35,372
95,372
Non-current liabilities
Long term borrowings (31,525 6,000) 25,525
Deferred taxation (5,000 1,470) 3,530
29,055
Current liabilities
Trade payables 12,000
Current portion of long term borrowings 6,000
Provision for litigation 5,000
Provision for taxation (2,000 + 9,988 2,000) 9,998
32,998
157,425
Shaheen Limited
Statement of profit or loss and other comprehensive income
As of June 30, 2015 Rs. in 000
Sales revenue 200,000
(104,708
Cost of sales (W2) )
Gross profit 95,292
Selling and distribution expenses (W2) (36,275)
Administrative expenses (W2) (30,450)
(66,725)
Financial charges (5,000)
Profit before taxation 23,567
Taxation (W3) (6,528)
Profit after taxation 17,039
Other comprehensive income net of tax -
Total comprehensive income 17,039

Emile Woolf International 112 The Institute of Chartered Accountants of Pakistan


Answers

Shaheen Limited
Statement of changes in equity 2015
As of June 30, 2015 Rs.000
Issued,
Retained
subscribed &
earnings
paid up capital
Balance July 1, 2014 60,000 32,000*
Correction of prior year error (10,000 20/120) (1,667)
Balance July 1, 2014 (restated) 60,000 30,333
Comprehensive income for the year 17,039
Dividend for the year ended June 30, 2014
(60,000*0.20) (12,000)
Balance June 30, 2015 60,000 35,372
*Retained earnings as at 01-07-09 = 20,000+ (20% of 60,000)=32,000

Workings
W1 Depreciation for the year
On building (36,000/20) 1,800
On plant and equipment (30,000 3,000)/10 2,700
Total 4,500

W2 Costs Selling and


Cost of Administrative
distribution
sales costs
costs
Opening inventory 23,000
Costs as per Trial balance 100,000 35,000 30,000
Closing inventory (30,000)
Depreciation (75%, 15%, and 10% of
Rs. 4,500) 3,375 675 450
Adjustment for goods sent on sale or
return, erroneously booked as sales
last year now returned during the year.
(10,000/1.2) 8,333
Amortization of export license
(6,000/5*0.5) 600
104,708 36,275 30,450

W3:Taxation
profit before tax 23,567
Disallowances and add backs 5,000
Taxable income 28,567
Current For the year (28,567*0.35) 9,998
For prior years (7,000 5,000) (2,000)
Deferred For the year (5,000 800)*0.35 (1,470)
6,528

Emile Woolf International 113 The Institute of Chartered Accountants of Pakistan


Financial accounting and reporting II

2.10 MOONLIGHT PAKISTAN LIMITED

(a) Moonlight Pakistan Limited


Statement of Financial Position
As at December 31, 2015
Rs. in
million
ASSETS
Non-current assets
Property, plant and equipment (W2) 3,472

Current assets
Stocks in trade 758
Trade receivables 702
Cash and bank 354
1,814
5,286
EQUITY
Issued, subscribed and paid-up capital (W3) 1,750
Share premium (420 x 2/12) 70
Retained earnings (W3) 876
2,696

Surplus on revaluation of fixed assets 240

LIABILITIES
Non-current liabilities
Long term loan 1,600
Deferred tax (22 + 80 x 35%) 50
Provision for gratuity 23
1,673
Current liabilities
Creditor and other liabilities (544 + 96) 640
Income tax payable 37
677
5,286

(b) Moonlight Pakistan Limited


Statement of profit or loss
For the year ended December 31, 2015
Rs. in
million
Sales 3,608
Cost of sales (W1) (2,149)
Gross profit 1,459
Selling expenses (W1) 252
Administrative expenses (W1) 270
522
937
Financial charges (210 + 1,600 x 12% x 6/12) 306
Profit before taxation 631
Taxation (37 + 80 x 35%) 65
Profit after taxation 566

Emile Woolf International 114 The Institute of Chartered Accountants of Pakistan


Answers

W1: Cost of sales/selling expenses/admin expenses


Cost of Selling Admin.
sales expenses expenses
Rs. in million
As per trial balance 1,784 220 250
Depreciation building (60% : 25% : 15%) (W2) 69 29 17
Depreciation plant 287 - -
Provision for gratuity (23-8) x 60%:20%:20% 9 3 3
2,149 252 270
W2: Property, plant and equipment
Land Building Plant Total
Rs. in million

Cost as at January 1, 2015 600 2,000 2,104 4,704


Accumulated depreciation - (400) (670) (1,070)

Revaluation (1,840 - (2,000 - 400 )) - 240 - 240


Current year depreciation - (287) (402)
(1,840/16) (115)

600 1,725 1,147 3,472


W3: Share Capital/Retained Earnings
Share capital Retained earnings
Rs. in million
As per trial balance 1,200 510
Bonus issue (1200 6) 200 (200)
Right issue (420 x 10/12) 350 -
Profit for the year - 566
1,750 876

2.11 FIGS PAKISTAN LIMITED


Figs Pakistan Limited
Statement of profit or loss and other comprehensive income
For the year ended 31 December 2015
2015
Rs. in
Note million
Sales 1 44,758
Cost of sales 2 (26,203)
Gross profit 18,555
Distribution costs 3 (6,431)
Administrative expenses 4 (752)
Other operating expenses 5 (399)
Other operating income 6 30
Profit from operations 11,003
Finance costs 7 (166)
Profit before tax 10,837
Taxation 8 (2,532)
Profit after tax 8,305
Other comprehensive income -
Total comprehensive income for the year 8,305

Emile Woolf International 115 The Institute of Chartered Accountants of Pakistan


Financial accounting and reporting II

Figs Pakistan Limited


Notes to the financial statements
For the year ended 31 December 2015

Rs. in
1 Sales Note million
Manufactured goods
Gross sales 56,528
Sales tax (10,201)
46,327
Imported goods
Gross sales 1,078
Sales tax (53)
1,025
Sales discounts (2,594)
44,758

2 Cost of sales
Raw material consumed (1,751 + 22,603 - 2,125) 22,229
Stores and spares consumed 180
Salaries, wages and benefits (2,367 55%) 2.1 1,302
Utilities (734 85%) 624
Depreciation and amortizations (1.287 70%) 901
Stationery and office expenses (230 25%) 58
Repairs and maintenance (315 85%) 268
25,562
Opening work in process 73
Closing work in process (125)
25,510
Opening finished goods (manufactured) 1,210
Closing finished goods (manufactured) (1,153)
25,567
Finished goods (imported)
Opening stock 44
Purchases 658
702
Closing stock (66)
636
26,203

2.1 Salaries, wages and benefits include Rs. 30 million (54 55%) and Rs. 24
million (44 55%) in respect of defined contribution plan and defined benefit
plan respectively.

Emile Woolf International 116 The Institute of Chartered Accountants of Pakistan


Answers

Rs. in
3 Distribution costs million
Advertisement and sales promotion 4,040
Outward freight and handling 1,279
Salaries, wages and benefits (2,367 30%) 3.2 710
Utilities (734 5%) 37
Depreciation and amortization (1,287 20%) 257
Stationery and office expenses (230 40%) 92
Repairs and maintenance (315 5%) 16
6,431

Salaries, wages and benefits include Rs. 16 million (54 30%) and Rs. 13
million (4430%) in respect of defined contribution plan and defined benefit plan
3.1 respectively.

Rs. in
4 Administrative expenses million
Salaries, wages and benefits (2,367 15%) 4.1 355
Utilities (734 10%) 73
Depreciation and amortization (1,287 10%) 129
Stationery and office expenses (230 35%) 80
Repairs and maintenance (315 10%) 31
Legal and professional charges 71
Auditor's remuneration 4.2 13
752

Salaries, wages and benefits include Rs. 8 million (54 15%) and Rs. 7 million
(4415%) in respect of defined contribution plan and defined benefit plan
4.1 respectively.
Rs. in
4.2 Auditor's remuneration million
Audit fees 8
Taxation services 4
Out of pocket expenses 1
13
5 Other operating expenses
Donation 5.1 34
Worker's Profit Participation Fund 257
Worker Welfare Fund 98
Loss on disposal of property, plant and equipment 10
399

5.1 Donations
Donations include Rs. 5 million given to Dates Cancer Foundation (DCF). One
of the companys directors, Mr. Peanut is a trustee of DCH.
Donations other than that mentioned above were not made to any donee in
which a director or his spouse had any interest at any time during the year.

Emile Woolf International 117 The Institute of Chartered Accountants of Pakistan


Financial accounting and reporting II

Rs. in
6 Other operating income million
Income from financial assets
Dividend income 12
Return on savings account 2
Income from non-financial assets
Scrap sales 16
30
7 Finance costs
Finance charges on short term borrowings 133
Exchange loss 22
Finance charges on lease 11
166
8 Taxation
Current - for the year 1,440
Deferred (3,120 35%) 1,092
2,532

Emile Woolf International 118 The Institute of Chartered Accountants of Pakistan


Answers

CHAPTER 3 IAS 7: STATEMENTS OF CASH FLOWS

3.1 KLEA
Statement of cash flows for the year ended 31st March 2015
Rs. in 000
Cash flows from operating activities
Profit before taxation 1,606
Adjustments for:
Depreciation (W4) 800
Finance income (50)
Interest expense 320

2,676
Increase in trade receivables (400)
Increase in inventories (1,200)
Increase in trade payables 334

Cash generated from operations 1,410
Interest paid (320)
Income taxes paid (W1) (630)

Net cash from operating activities 460
Cash flows from investing activities
Purchase of intangible assets (W2) (300)
Purchase of property, plant and equipment (W3) (1,600)
Proceeds from sale of equipment 150
Purchase of long-term investments (200)
Finance income received 50

Net cash used in investing activities (1,900)

Cash flows from financing activities


Proceeds from issue of share capital (1,000 + 278) 1,278
Payments to redeem debentures (400)
Dividends paid (400)

Net cash used in financing activities 478

Net decrease in cash and cash equivalents (962)
Cash and cash equivalents at 1 April 2014 580

Cash and cash equivalents at 31 March 2015 (32 - 414) (382)
(Note: Alternative classifications of the cash flows in accordance with IAS 7 should
receive full credit i.e. interest and dividends received as investing activities or
operating cash flows, interest and dividends paid as financing or operating cash
flows.)
Notes
(1) Analysis of cash and cash equivalents Rs. in 000
2015 2014
Cash on hand and balances with bank 32 580
Bank overdraft (414) -

Cash and cash equivalents (382) 580

Emile Woolf International 119 The Institute of Chartered Accountants of Pakistan


Financial accounting and reporting II

(2) Material non-cash transactions


During the year land was re-valued upwards by Rs.1million

Workings Rs. in 000


(W1) Taxation paid
Taxation creditor brought forward 400
Taxation expense for period 650

1,050
Taxation creditor carried forward (420)

Taxation paid in the year 630

(W2) Intangible assets


Net book value brought forward 200
Capitalised in the year (from (i)) 300

500
Amortisation charged in year (from (i)) (200)

Intangibles acquired in the year 300

(W3) Property, plant and equipment


Cost brought forward 3,000
Revaluation in year (from (ii)) 1,000
Disposals (from (iii)) (600)
Additions (balancing figure) 1,600

Cost carried forward 5,000

(W4) Depreciation and amortisation


Depreciation (150 movement + 500 on disposal) 650
Amortisation 200
Profit on disposal (W5) (50)

Charge shown in statement of profit or loss 800

Hence add back of depreciation and amortisation also takes account of the
profit on disposal of the plant and machinery.
(W5) Disposal
Cost of disposal 600
Accumulated depreciation (500)

Net book value 100
Proceeds of sale 150

Profit on sale 50

Emile Woolf International 120 The Institute of Chartered Accountants of Pakistan


Answers

3.2 STANDARD INC


Statement of cash flows for the year ended 31 December 2015
Rs. in 000 Rs. in 000
Cash flows from operating activities
Net profit before tax (W7) 64,000
Adjustments for:
Depreciation, loss on sale (W1-5) 20,000
Interest receivable (450)
Interest and premium payable 8,400

Operating profit 91,950

Increase in inventories (14,000)


Increase in receivables (1,200)
Increase in payables 14,440

Cash generated from operations 91,190
Interest paid (6,840)
Tax paid (W6) (10,500)

Net cash from operating activities 73,850
Cash flows from investing activities

Acquisition of long-term investment (4,600)


Purchase of property plant and equipment (69,000)
Receipt from sale of long-term investment 4,000
Interest received 450

Net cash used in investing activities (69,150)

Cash flows from financing activities

Proceeds from issuance of shares 70,000


Redemption of long term loan (42,000)
Dividends paid (7,500)

Net cash used in financing activities 20,500

Net increase in cash and cash equivalents 25,200

WORKINGS

(1) Plant and machinery account at cost

Rs.000 Rs.000
Balance b/d 120,000 Disposals account 8,000
Additions 39,000 Balance c/d 151,000

159,000 159,000

Emile Woolf International 121 The Institute of Chartered Accountants of Pakistan


Financial accounting and reporting II

(2) Fixtures and fittings account at cost

Rs.000 Rs.000
Balance b/d 24,000 Disposals account 5,000
Additions 10,000 Balance c/d 29,000

34,000 34,000

Fixed assets additions summary


Rs.000
Freehold property Rs.000(130,000 - 110,000) 20,000
Plant and machinery 39,000
Fixtures and fittings 10,000

69,000

(3) Plant and machinery account depreciation

Rs.000 Rs.000
Disposals account 6,000 Balance b/d 45,000
Balance c/d 54,000 Charge for year 15,000

60,000 60,000

(4) Fixtures and fittings account depreciation

Rs.000 Rs.000
Disposals account 2,000 Balance b/d 13,000
Balance c/d 15,000 Charge for year 4,000

17,000 17,000

(5) Fixed assets disposals account

Rs.000 Rs.000
Plant cost 8,000 Plant depreciation 6,000
Fittings cost 5,000 Fittings depreciation 2,000
Cash proceeds
Plant 3,000
Fittings 1,000
Depreciation underprovided
(bal fig) 1,000

13,000 13,000

Emile Woolf International 122 The Institute of Chartered Accountants of Pakistan


Answers

(6) Tax account

Rs.000 Rs.000
Cash paid (bal fig) 10,500 Balance b/f corporation tax 21,500
Balance c/f corporation tax 33,000 I&E account corporation tax 22,000

43,500 43,500

(7) Net profit before tax


Note As profit before tax is required, reconstruct the statement of profit or
loss up to this figure.
Rs. in 000
Profit before tax 64,000
Taxation Corporation tax (22,000)

42,000
Dividends (15,000)

Retained profit for year 27,000
Balance b/f 14,000

Balance c/f 41,000

(8) Cash and cash equivalents as shown in the statement of financial


position
Cash and cash equivalents consist of cash on hand and balances with banks.
Rs. in 000
2015 2014 Change
in
year
Cash at bank 11,400 200 11,200
Bank overdraft (14,000) 14,000

11,400 (13,800) 25,200

Emile Woolf International 123 The Institute of Chartered Accountants of Pakistan


Financial accounting and reporting II

3.3 FALLEN
Statement of cash flows for the year ended 31 December 2015
Cash flows from operating activities Rs. in 000
Net profit before tax 4,625
Adjustments for:
Depreciation, (W1-3) 1,472
Interest payable 152

Operating profit 6,249
Increase in deferred repairs provision 186
Increase in inventories (894)
Increase in receivables (594)
Increase in payables 324

Cash generated from operations 5,271
Interest paid (152)
Tax paid (W5) (1,775)

Net cash from operating activities 3,344

Cash flows from investing activities

Acquisition of long-term investment (198)


Purchase of property plant and equipment (3,800)
Receipt from sale of long-term investment 168

Net cash used in investing activities (3,830)

Cash flows from financing activities

Proceeds from issuance of shares (W6-7) 792


Redemption of long term loan (560)
Dividends paid (544)

Net cash used in financing activities (312)

Net increase in cash and cash equivalents (798)

WORKINGS Rs. in 000

(1) Leasehold premises (net)


Brought forward 5,700 Depreciation (to balance) 400
Additions 1,300 Carried forward 6,600

7,000 7,000

Emile Woolf International 124 The Institute of Chartered Accountants of Pakistan


Answers

(2) Plant (net)


Brought forward 3,780 Disposals 276
Additions 2,500 Depreciation (to balance) 964
Carried forward 5,040

6,280 6,280

(3) Disposals
Plant 276 Cash 168
Loss on sale (to balance) 108

276 276

(4) Dividends
Cash (to balance) 544 Brought forward 234
Carried forward 390 I&E account 700

934 934

(5) Taxation
Cash (to balance) 1,775 Brought forward
Carried forward DT 138
DT 202 CT 2,038
CT 1,730
I&E account 1,531

3,707 3,707

(6) Share capital


Brought forward 1,800
Carried forward 2,280 Cash (to balance) 480

2,280 2,280

(7) Share premium


Carried forward 2,112 Brought forward 1,800
Cash (to balance) 312

2,112 2,112

Emile Woolf International 125 The Institute of Chartered Accountants of Pakistan


Financial accounting and reporting II

(8) Long term loan


Cash (to balance) 560 Brought forward 1,800
Carried forward 1,240

1,800 1,800

(9) Analysis of the balances of cash and cash equivalents as shown in the
statement of financial position
Cash and cash equivalents consist of cash on hand and balances with banks.
Rs. in 000
2015 2014 Change in
year
Cash at bank and in hand 576 (576)
Bank overdrafts (222) (222)

(222) 576 (798)

3.4 BIN QASIM MOTORS LIMITED


Note: figures in brackets are in Rs.000
Bin Qasim Motors Limited
Statement of cash flows for the year to 30 September 2015
Rs.000 Rs.000
Cash flows from operating activities
Net profit before interest and tax (3,198 1,479) 1,719
Adjustments for:
Depreciation buildings (W1) 80
plant (W1) 276
Loss on disposal of plant (W1) 86 442
442
Amortisation of government grants (W2) (125)
Negligence claim previously provided (120)
Operating profit before working capital changes 1,916
Increase in inventories (1,046 785) (261)
Increase in accounts receivable (935 824) (111)
Decrease in accounts payable (760 644) (116)
Cash generated from operations 1,428
Interest paid (260 + 25 40) (245)
Income tax paid (W4) (368)
Dividends paid (180)
Net cash from operating activities 635

Cash flows from investing activities

Emile Woolf International 126 The Institute of Chartered Accountants of Pakistan


Answers

Bin Qasim Motors Limited


Statement of cash flows for the year to 30 September 2015
Rs.000 Rs.000
Purchase of land and buildings (W1) (50)
Purchase of plant (W1) (848)
Purchase of non-current investments (690)
Purchase of treasury bills (120 50) (70)
Proceeds of sale of plant (W1) 170
Receipt of cash on servicing contracts (W2) 175
Investment income 120
Net cash used in investing activities (1,193)
Cash flows from financing activities
Issue of ordinary shares (W3) 300
Net decrease in cash and cash equivalents (258)
Cash and cash equivalents at the beginning of the period 122
Cash and cash equivalents at the end of the period (136)
Workings
(W1) Non-current assets
Rs.000
Land and buildings cost/valuation
Balance b/f 1,800
Revaluation surplus 150
Balance c/f (2,000)
Difference cash purchase (50)

Plant cost
Balance b/f 1,220
Disposal (500)
Balance c/f (1,568)
Difference cash purchase (848)
Depreciation of non-current assets:
Building (760 680) 80
Plant (464 (432 244)) 276
The plant had a carrying value of Rs.256,000 at the date of its disposal (500
cost 244 depreciation). As there was a loss on sale of Rs.86,000 (given in
question), the sale proceeds must have been Rs.170,000 (i.e. 256 86).
(W2) Deferred income
Balances b/f current (125)
non-current (200)
(325)
Amortisation credited to cost of sales 125
Balances c/f current 100
non-current 275
375
Difference cash receipt 175

Emile Woolf International 127 The Institute of Chartered Accountants of Pakistan


Financial accounting and reporting II

(W3) Share capital and convertible loan stock


A reconciliation of share capital, share premium and the revaluation reserve
shows the shares issued for cash:
Share Share Revaluatio
capital premium n reserve
Rs.000 Rs.000 Rs.000
Opening balance (1,000) (60) (40)
Revaluation of land (150)
Bonus issue 1 for 10 (100) 100
Conversion of loan stock (see below) (100) (300)
Closing balance 1,400 460 90

Difference issued for cash 200 100 nil

The 10% convertible loan stock had a carrying value of Rs.400,000 at the date
of conversion to equity shares. This would be taken as the consideration for
the shares issued which would be 100,000 Rs.1 shares (i.e. 400,000/100
25). This would increase issued share capital by Rs.100,000 and share
premium by Rs.300,000.
(W4) Income tax
Rs.000
Tax provision b/f (367)
Deferred tax b/f (400)
Statement of profit or loss tax charge (520)
Tax provision c/f 480
Deferred tax c/f 439
Difference cash paid (368)

Emile Woolf International 128 The Institute of Chartered Accountants of Pakistan


Answers

3.5 ITTEHAD MANUFACTURING LTD


(a)
Ittehad Manufacturing Ltd
Statement of cash flows for the year to 30 September 2015
Rs.m Rs.m
Cash flows from operating activities
Net profit before interest and tax 920
Adjustments for:
Amortisation development expenditure (W1) 130
Depreciation property, plant and equipment 320
Loss on sale of plant 50
Increase in inventory (1,420 940) (480)
Increase in accounts receivable (990 680) (310)
Increase in accounts payable (875 730) 145
Decrease in deferred income (260 300) (40)
Cash generated from operations 735
Interest paid (30 (15 5 accrual adjustments)) (20)
Income tax paid (W2) (130)
Net cash from operating activities 585
Cash flows from investing activities
Purchase of property, plant and equipment (W3) (250)
Capitalised development costs (W1) (500)
Proceeds of sale of plant (W3) 20
Net cash from investing activities (730)

Cash flows from financing activities


Issue of ordinary shares (W4) 450
Issue of loan notes (300 100) 200
Dividends paid (320)
Net cash generated from financing activities 330
Net increase in cash and cash equivalents 185
Cash and cash equivalents at beginning of period (115)
Cash and cash equivalents at end of period 70
Workings
(W1) Development expenditure
Rs.m
Opening balance 100
Amount capitalised 500
Closing balance (470)
Amortisation: balancing figure 130

Emile Woolf International 129 The Institute of Chartered Accountants of Pakistan


Financial accounting and reporting II

(W2) Income tax


Rs.m Rs.m
Opening balance: tax provision 160
Opening balance: deferred tax 140
300
Tax charged to statement of profit or loss 270
Closing balance: tax provision (130)
Closing balance: deferred tax (310)
(440)
Tax paid (cash payments) 130
(W3) Property, plant and equipment
Rs.m
Opening balance 1,830
Revaluation surplus 200
Plant acquired 250
Depreciation (320)
1,960
Closing balance 1,890
Disposal at net book value balancing figure 70

Disposal of plant:
Disposal at net book value (see above) 70
Loss on sale (given in the question) (50)
Difference = Sale proceeds 20
(W4) Share capital
Rs.m
Opening balance, ordinary shares 500
Bonus issue 1 for 10 (from retained earnings) 50
550
Closing balance, ordinary shares 750
Difference: shares issued for cash (nominal value) 200
Plus increase in share premium (350 100) 250
Total cash proceeds of issue of ordinary shares 450

(b) The cash flows generated from operations were Rs.685 million and are more
than enough to pay the interest costs and taxation, but these cash flows are
not as large as the equivalent profit figure. For most companies the operating
cash flows are higher than the profit before interest and tax due to the effects
of depreciation/amortisation charges (which are not cash flows). In the case of
Ittehad Manufacturing Ltd the depreciation/amortisation effect has been more
than offset by a much higher investment in working capital of Rs.645 million.
Inventory has increased by over 50% and accounts receivable by 45%. This
may be an indication of expanding activity, but it could also be an indication of
poor inventory management policy and poor credit control, or even the
presence of some obsolete inventory or unprovided bad accounts receivable.

Emile Woolf International 130 The Institute of Chartered Accountants of Pakistan


Answers

A cause of concern is the size of the dividends, which seem high at Rs.320
million. This is a very high distribution ratio, and it seems odd that the
company is returning such large amounts to shareholders at the same time as
they are raising finance. Rs.450 million has been received from the issue of
new shares and Rs.200 million from a further issue of loan notes.
The company has invested considerably in new plant (Rs.250 million) and
even more so in development expenditure (Rs.500 million). If management
has properly applied the capitalisation criteria in IAS 38 Intangible Assets, then
this indicates that they expect good future returns from the investment in new
products or processes. The net investment in non-current assets is Rs.680
million which closely correlates to the proceeds from financing of Rs.650
million. In general it is acceptable to finance increases in the capacity of non-
current assets by raising additional finance, however operating cash flows
should finance replacement of consumed non-current assets.

3.6 WASEEM INDUSTRIES LIMITED


Waseem Industries Limited
Statement of cash flows for the year ended December 31, 2015
2015 Workings
Rs.m
Cash flows from operating activities

Profit before taxation 64 W1


Adjustments for:
Depreciation 17

Gain on sale of fixed assets (3)


Provision for gratuity 10 W2
Interest expense 16
104
Increase/decrease in working capital
Increase in stocks-in-trade (7) (55-48)
Increase in trade debts (13) (51-38)
Decrease in advance, prepayments and other
receivables 6 W3
Increase in trade and other payables 22 W4
8
Cash generated from operations 112
Gratuity paid (6)
Interest paid (18) W5
Income taxes paid (22) W6
Net cash from operating activities 66

Emile Woolf International 131 The Institute of Chartered Accountants of Pakistan


Financial accounting and reporting II

Cash flows from investing activities


Sale proceeds from sale of property, plant and
equipment 26

Purchase of property, plant and equipment (100) W7


Increase in capital work in progress (2) (20 -18)
Sale of long term investments 25 (75-100)
Net cash used in investing activities (51)

Cash flows from financing activities


Payment of long term finances (21) W8
Increase in short term finances 7 (13 - 6)
*Dividend paid (10)
Net cash used in financing activities (24)

Net increase in cash and cash equivalents (9)


Cash and cash equivalent at the beginning of the year 20
Cash and cash equivalent at the end of the year 11

W1: Profit before taxation Rs.m


Unappropriated profit closing 85
Income tax expenses for the year 2015 19
Dividend (Rs. 125 million x 8%) 10
114
Less: Unappropriated profit - opening (50)
64

W2: Provision for gratuity Rs.m


Provision for gratuity: Closing 16
Paid during the year 2015 6
22
Less: Provision for gratuity - opening 12
Provision for the year 10

W3: Advances, prepayments and other receivables Rs.m


Advances, payments and other receivables closing 37
Advance Tax - closing (10)
27

Advances, payments and other receivables opening 40


Advance Tax opening (7)
33
Decrease in advances, prepayments and other (6)
receivables

Emile Woolf International 132 The Institute of Chartered Accountants of Pakistan


Answers

W4: Trade and other payables Rs.m


Trade and other payables closing 66
Accrued mark-up closing (7)
59

Trade and other payables opening 46


Accrued mark-up opening (9)
37
Increase in trade and other payables 22

W5: Interest paid Rs.m


Accrued mark up opening 9
Expense for the year 16
25
Less: Accrued mark-up closing (7)
Interest paid during the year 18

W6: Income taxes paid Rs.m


Advances taxes closing 10
Provision for the year 19
29
Less: advance taxes - opening (7)
Income taxes paid during the year 22

W7: Fixed assets purchase Rs.m


Closing fixed assets 242
Depreciation for the year 17
Carrying amount of disposed off assets 23
302
Less: opening fixed assets (182)
Purchase of fixed assets 100

W8: Payment of long term finances Rs.m


Long term finance including current portion closing
(118 + 22) 140
Long term finance including current portion Opening
(94 + 25) (119)
Payment during the year 21

Emile Woolf International 133 The Institute of Chartered Accountants of Pakistan


Financial accounting and reporting II

3.7 JALIB INDUSTRIES LIMITED


Jalib Industries Limited
Statement of cash flow for the year ended December 31, 2015
Rs. in
million
CASH FLOW FROM OPERATING ACTIVITIES
Net profit before tax 191.40
Adjustments for:
Depreciation 27.70
Loss on sale of fixed assets 4.60
Provision for gratuity 15.50
Financial charges 10.50
Bad debt expense 1.20 Working 3
250.90
Working Capital Changes
Increase in creditors, accrued and other liabilities
([36.2 - 5] - [34.4 - 6]) 2.80
Increase in stock in trade (80.80)
Increase in trade debts (5.00) Working 4
Decrease in advances and other receivables
[(42-2.2)-(37.4-3.6)] 6.00
Cash generated from operations 173.90
Gratuity paid (4.40) Working 2
Income tax paid [3.6 + 104.6 - 2.2] (106.00)
Financial charges paid (6 + 10.5 - 5) (11.50)
Net cash from operating activities 52.00
CASH FLOW FROM INVESTING ACTIVITIES
Capital expenditure incurred (57.00) Working 1
Proceeds on sale of fixed assets 10.40
Net cash used in investing activities (46.60)
CASH FLOW FROM FINANCING ACTIVITIES
Issue of share capital 99.00 Working 5
Repayment of long term loans (120 - 98) (22.00)
Dividend paid (1.4 + 75 - 3) (73.40)
Net cash from financing activities 3.60
Net increase in cash and cash equivalents 9.00
Cash and cash equivalent at the beginning of year 3.00
Cash and cash equivalent at the end of year 12.00

WORKING 1
Rs. in
million
Capital expenditure incurred
Book value of PPE - Closing 129.40
Book value of CWIP - Closing 22.50
Add: Book value of assets sold during the year 15.00
Add: Depreciation for the year 27.70
Less: Book value of PPE - Opening (100.60)
Less: Book value of CWIP - Opening (37.00)
57.00

Emile Woolf International 134 The Institute of Chartered Accountants of Pakistan


Answers

WORKING 2
Rs. in
million
Gratuity paid during the year
Opening balance 27.50
Provision for gratuity 15.50
43.00
Less: Closing balance (38.60)
Gratuity paid during the year 4.40

WORKING 3
Bad debts expense for the year
Closing balance (28.5 0.95) - 28.5 1.50
Less: Opening balance (24.7 0.95) - 24.7 (1.30)
Add: Bad debts written off 1.00
Bad debts expense for the year 1.20

WORKING 4
Increase in trade debts
Closing balance (28.5 0.95) 30.00
Less: Opening balance (24.7 / 0.95) (26.00)
Add: Bad debts written off 1.00
5.00
WORKING 5
Issue of share capital
Closing balance of paid up capital 396.00
Closing balance of share premium 45.00
Less:
Opening balance of paid up capital (300.00)
Opening balance of share premium (12.00)
Issue of bonus shares (300 x 10%) (30.00)
99.00

Emile Woolf International 135 The Institute of Chartered Accountants of Pakistan


Financial accounting and reporting II

3.8 APOLLO INDUSTRY LIMITED


Apollo Industry Limited
Statement of cash flows for the year ended December 31, 2015
Rs. in
000
Cash used in operating activities
Profit before taxation 6,500
Adjustment for: (non cash items / separately disclosed items)
Depreciation for the year (7,000-90-1,000) 5,910
Amortization for the year (1140+50-1100) 90
Provision for staff gratuity (1,400+300-1,190) 510
Profit on sale of fixed assets (2,800-1,000) (1,800)
Mark-up on short term placement (1,000)
Operating profit before working capital changes 10,210
Increase in working capital (12,125 15,700 + 4,200 6,250) (5,625)
Cash generated from operations 4,585
Payment for staff gratuity (300)
Payment for taxation (950 + 4,660 800) (4,810)
(525)
Cash used in investing activities
Capital expenditure incurred Note 1 (13,110)
Proceeds from sale of PPE (1,200 + 1,800) 3,000
Acquisition of intangible assets (50)
Mark-up received on short term placement 1,000
Long term deposits (400-300) (100)
(9,260)
Cash used in financing activities
Issue of ordinary share capital (25,000-2,000-20,000) 3,000
Net decrease in cash and cash equivalents (6,785)
Opening balance: cash and cash equivalents 7,225
Closing balance: cash and cash equivalents 440

Note 1 Capital expenditure incurred: Rs.000


Opening book value for PPE 25,500
Opening book value for CWIP 10,000
Book value of assets sold during the
year (1,200)
Depreciation for the year (7,000-90-
1,000) (5,910)
Revaluation reserve adjustment (1,000)
Closing book value for PPE (35,000)
Closing book value for CWIP (5,500)
(13,110)

Emile Woolf International 136 The Institute of Chartered Accountants of Pakistan


Answers

3.9 MARVEL ENGINEERING LIMITED


Marvel Engineering Limited
Cash Flow Statement
For the year ended 30 June 2015

Workings 2015
Cash flows from operating activities Rs.m
Profit before taxation 88.00
Adjustment for non-cash charges and other items:
Depreciation 50.00
Impairment of plant and machinery 11.00
Financial charges 75.00
Gain on sale of fixed assets (2.00)
Gain on sale of investments (3.00)
Dividend income (30.00)
Provision for gratuity payable (55 - 50 + 6) 11.00
Working capital changes
Decrease / (increase) in current assets:
Increase in stock-in-trade (97 - 68) (29.00)
Increase in trade debts (see tutorial note) (76.00)
Other current assets (100 - 120) 20.00
Increase / (decrease) in current liabilities:
Trade and other payables ([73 - 7] - [56 - 3]) 13.00
Cash generated from operations 128.00
Financial charges paid (3 + 75 - 7) (71.00)
Income tax paid (5 + 21 + 21 - 12 - 15) (20.00)
Gratuity paid (6.00)
Net cash generated from operating activities 31.00

Cash flows from investing activities


Capital expenditure 1 (289.00)
Proceeds from sale of property, plant and equipment (5+2) 7.00
Proceeds from sale of investments (10+3) 13.00
Purchase of long term investments (130-100+10) (40.00)
Dividend received 30.00
Net cash used in investing activities (279.00)

Cash flows from financing activities


Insurance of ordinary shares 2 40.00
Proceeds from long term loan (330 - 110) 220.00
Payment of dividend (2 + (440 5%) - 4) (20.00)
Net cash from financing activities 240.00
Net decrease in cash and cash equivalents (8.00)
Cash and cash equivalent at the beginning of the year 39.00
Cash and cash equivalent at the end of the year 31.00

Emile Woolf International 137 The Institute of Chartered Accountants of Pakistan


Financial accounting and reporting II

W1: Capital expenditure Rs.m


Closing balance 633.00
Add: Depreciation for the year 50.00
Add: Impairment against plant 11.00
Add: Disposal during the year 5.00
Less: Opening balance (410.00)
289.00
W2: Issuance of ordinary shares
Closing balance of share capital 494.00
Closing balance of share premium 8.00
Less: Bonus shares issued (440 5%) (22.00)
Less: Opening balance of share capital (440.00)
40.00
Tutorial note:
The original ICAP answer did not simply adjust for the movement in trade debts but
added back the write off for bad debts (Rs. 6 million) and movement in the doubtful
debt provision (Rs. 4 million) and then adjusted for the movement in trade debt
before these write offs (Rs. 86 million).
As the trade debt contains the credit for the write off and the profit for the year
contains the debit it is easier to leave the expense in and adjust for the net
movement.
The following working was provided in the official answers.
WORKINGS (All amount in million rupees)
Provision
Trade
W1: for bad
debtors
debts
Closing balance (133 0.95) - 133 7.00 (133 0.95) 140.00
Add: Bad debts written off 6.00 6.00
Less : Opening balance (57 0.95) - 57 (3.00) (57 0.95) (60.00)
10.00 86.00

Emile Woolf International 138 The Institute of Chartered Accountants of Pakistan


Answers

CHAPTER 4 CONSOLIDATED ACCOUNTS: STATEMENTS OF FINANCIAL


POSITION BASIC APPROACH

4.1 HALL
Consolidated statement of financial position as at 31 December 2015
Rs.000
Assets
Non-current assets
Property, plant and equipment (35,000 + 20,000) 55,000
Goodwill 3,000

58,000
Current assets (16,000 + 14,000) 30,000

88,000

Equity and liabilities
Capital and reserves
Share capital 10,000
Retained earnings (W5) 16,000

26,000

Non-controlling interest (W4) 4,000


Long-term liabilities
8% Debenture loans (20,000 + 9,000) 29,000
Current liabilities (20,000 + 9,000) 29,000

88,000

WORKINGS

(1) Group structure

Hall

75%

Stand

(2) Net assets of Stand


Reporting Date of Post
date acquisition acquisition
Rs.000 Rs.000
Share capital 4,000 4,000
Retained earnings 12,000 8,000 4,000
16,000 12,000

Emile Woolf International 139 The Institute of Chartered Accountants of Pakistan


Financial accounting and reporting II

(3) Goodwill
Rs.000
Cost of shares 12,000
Less Net assets acquired (75% 12,000 (W2)) (9,000)

3,000

(4) Non-controlling interest (25% 16,000 (W2)) Rs.000


4,000

(5) Retained earnings


Rs.000
Hall Inc 13,000
Stand Inc (75% 4,000 (W2)) 3,000

16,000

4.2 HASSLE
Consolidated statement of financial position as at 31 December 2015
Rs.
Sundry net assets (207,500 + 226,600) 474,100

474,100

Equity capital 120,000


Retained earnings (W5) 123,500

243,500
Non-controlling interests (W4) 24,000
Sundry liabilities (100,000 + 106,600) 206,600

474,100

WORKINGS

(1) Group structure


Hassle

80%

Strife

Emile Woolf International 140 The Institute of Chartered Accountants of Pakistan


Answers

(2) Net assets of Strife


Reporting Date of Post
date acquisition acquisition
Rs. Rs.
Share capital 50,000 50,000
Retained earnings 70,000 50,000 20,000
120,000 100,000

(3) Goodwill Rs.


Cost 60,000
Net assets acquired (80% 100,000) (W2) (80,000)

(20,000)

(4) Non-controlling interest Rs.


20% 120,000) (W2) 24,000

(5) Retained earnings Rs.


Hassle 87,500
Strife (80% (70,000 50,000) (W2)) 16,000
Negative goodwill (W3) 20,000

123,500

4.3 HYMN
Consolidated statement of financial position as at 31 December 2015
Rs.
Assets
Non-current assets
Property, plant and equipment 170,000
Goodwill 29,000
Current assets 275,000

474,000

Equity and liabilities


Shareholders equity
Share capital 100,000
Retained earnings (W5) 178,200

278,200
Non-controlling interest (W4) 19,800
Current liabilities 176,000

474,000

Emile Woolf International 141 The Institute of Chartered Accountants of Pakistan


Financial accounting and reporting II

WORKINGS
(1) Group structure

Hymn

80%

Psalm

(2) Net assets of Psalm


Reporting Date of Post
date acquisition acquisition
Rs. Rs.
Share capital 50,000 50,000
Retained earnings 49,000 20,000 29,000
99,000 70,000

(3) Goodwill Rs.


Cost of shares 85,000
Net assets acquired
Psalm Inc (80% 70,000) (W2) (56,000)

(29,000)

(4) Non-controlling interest Rs.


20% 99,000 (W2) 19,800

(5) Retained earnings Rs.


Hymn 155,000
Psalm (80% 29,000 (W2)) 23,200

178,200

Emile Woolf International 142 The Institute of Chartered Accountants of Pakistan


Answers

4.4 HANG
Consolidated statement of financial position as at 31 December 2015
Rs.
Assets
Non-current assets
Property, plant and equipment (240 + 180) 420,000
Goodwill 26,600

Current assets (250 + 196) 446,000



892,600

Equity and liabilities


Shareholders equity
Share capital 200,000
Share premium account 25,000
Retained earnings (W5) 198,000

423,000
Non-controlling interest (W4) 87,600
Current liabilities (225 + 157) 382,000

892,600

WORKINGS
(1) Group structure

Hang

60%

Swing

(2) Net assets of Swing Inc


31 Dec 31 Dec
2015 2014
Rs. Rs.
Ordinary shares of Rs.1 each 90,000 90,000
Share premium account 49,000 49,000
Retained earnings 80,000 50,000

219,000 189,000

Emile Woolf International 143 The Institute of Chartered Accountants of Pakistan


Financial accounting and reporting II

Reporting Date of Post


date acquisition acquisition
Rs. Rs.
Share capital 90,000 90,000
Share premium 49,000 49,000
Retained earnings 80,000 50,000 30,000
219,000 189,000

(3) Goodwill Rs.


Cost 140,000
Net assets acquired (60% 189,000) (W2) (113,400)

26,600

(4) Non-controlling interest Rs.


40% 219,000 (W2) 87,600

(5) Retained earnings Rs.


Hang 180,000
Swing (60% 30,000 (W2)) 18,000

198,000

4.5 HASH
Consolidated statement of financial position as at 31 December 2015
Rs.000
Sundry net assets (207,500 + 226,600) 434,100
Goodwill (W2) 8,800

442,900

Share capital 120,000
Retained earnings (W5) 92,300

212,300
Non-controlling interests (W4) 24,000
Sundry liabilities (100,000 + 106,600) 206,600

442,900

Emile Woolf International 144 The Institute of Chartered Accountants of Pakistan


Answers

WORKINGS
(1) Group structure

Hash

80%

Stash
(2) Net assets of Stash
Reporting Date of Post
date acquisition acquisition
Rs.000 Rs.000
Share capital 50,000 50,000
Retained earnings:
At the start of the year
(70,000 24,000) 46,000
Profit for the first 9m
(24,000 9/12) 18,000
70,000 64,000 6,000
120,000 114,000

(3) Goodwill Rs.000


Cost 100,000
Net assets acquired (80% 114,000) (W2) (91,200)

8,800

(4) Non-controlling interest Rs.000
20% 120,000) (W2) 24,000

(5) Retained earnings Rs.000


Hash 87,500
Stash (80% (70,000 64,000) (W2)) 4,800

92,300

Emile Woolf International 145 The Institute of Chartered Accountants of Pakistan


Financial accounting and reporting II

CHAPTER 5 CONSOLIDATED ACCOUNTS: STATEMENTS OF FINANCIAL


POSITION COMPLICATIONS

5.1 HAIL
Consolidated statement of financial position as at 31 December 2015
Rs.000 Rs.000
Assets
Non-current assets
Property, plant and equipment 246,000
Investments (68,000 65,000) 3,000
Goodwill (W3) 6,500
Current assets
Cash at bank and in hand 39,900
Trade receivables 138,300
Inventories 92,400

526,100

Equity and liabilities
Capital and reserves
Share capital 100,000
Capital reserve (W6) 18,000
Retained earnings (W5) 210,480

328,480
Non-controlling interest (W4) 11,420

Current liabilities
Trade payables 183,000
Proposed dividend parent company 3,000
non controlling interest 200
3,200

526,100

WORKINGS
(1) Group structure
Hail

90%

Snow

Emile Woolf International 146 The Institute of Chartered Accountants of Pakistan


Answers

(2) Net assets of Snow


Reporting Date of Post
date acquisition acquisition
Rs.000 Rs.000
Share capital 50,000 50,000
Share premium account 5,000 5,000
Revaluation reserve 20,000
Retained earnings
Per question 41,200
Proposed dividend (2,000)
39,200 10,000 29,200
114,200 65,000

(3) Goodwill
Rs.000
Cost of shares 65,000
Net assets acquired (90% 65,000) (W2) (58,500)

6,500

(4) Non-controlling interest


Rs.000
10% 114,200 (W2) 11,420

(5) Retained earnings


Rs.000
Hail 185,400
Proposed dividend (3,000)
Dividend receivable from Snow 1,800
Snow (90% 29,200 (W2)) 26,280

210,480

(6) Capital reserve


Rs.000
Snow (90% 20,000 (W2)) 18,000

Emile Woolf International 147 The Institute of Chartered Accountants of Pakistan


Financial accounting and reporting II

5.2 HAIRY

Consolidated statement of financial position as at 31 December 2015


Rs.000
Assets
Non-current assets
Property, plant and equipment 180,000
Current assets
Cash at bank and in hand 15,500
Investments 3,000
Receivables 91,700
Inventory (17,000 + 11,000 800) 27,200

317,400

Equity and liabilities
Capital and reserves
Share capital 100,000
Share premium account 20,000
Capital reserve 23,000
Retained earnings (W5) 102,900

245,900
Non-controlling interest (W4) 16,500

Current liabilities 55,000



317,400

WORKINGS
(1) Group structure
Hairy

80%

Spider

Emile Woolf International 148 The Institute of Chartered Accountants of Pakistan


Answers

(2) Net assets of Spider


Reporting Date of Post
date acquisition acquisition
Rs.000 Rs.000
Share capital 60,000 60,000
Share premium account 16,000 16,000
Retained earnings
Per question 7,300
Unrealised profit (800)
6,500 2,300 4,200
82,500 78,300

(3) Goodwill Rs.000


Cost of shares 55,000
Less Net assets acquired (80% 78,300 (W2)) (62,640)

(7,640)

(4) Non-controlling interest Rs.000


Share of net assets (20% 82,500 (W2)) 16,500

(5) Retained earnings Rs.000


Hairy 91,900
Spider (80% 4,200 (W2)) 3,360
Negative goodwill (W4) 7,640

102,900

Emile Woolf International 149 The Institute of Chartered Accountants of Pakistan


Financial accounting and reporting II

5.3 HARD
Consolidated statement of financial position as at 31 December 2015

Rs.000
Assets
Non-current assets
Property, plant and equipment (225 + 175 17.5 (W6)) 382,500
Goodwill (W3) 14,000

Current assets (271 + 157) 428,000



824,500

Equity and liabilities


Shareholders equity
Share capital 100,000
Share premium account 15,000
Retained earnings (W5) 260,500

375,500
Non-controlling interest (W4) 76,000
Current liabilities 373,000

824,500

WORKINGS

(1) Group structure

Hard

60%

Soft

(2) Net assets of Soft Inc


31 Dec 31 Dec Post
2015 2014 acquisition
Rs.000 Rs.000
Share capital 100,000 100,000
Share premium account 10,000 10,000
Retained earnings 80,000 50,000 30,000
190,000 160,000

Emile Woolf International 150 The Institute of Chartered Accountants of Pakistan


Answers

(3) Goodwill Rs.000


Cost 110,000
Net assets acquired
60% 160,000 (W2) (96,000)

14,000

(4) Non-controlling interest Rs.000


40% 190,000 (W2) 76,000

(5) Retained earnings Rs.000


Hard 260,000
Less Adjustment re intra group transfer (17,500)

242,500
Soft (60% (80,000 50,000 (W2)) 18,000

260,500

(6) PURP on non current assets


IS Rs.000
Cost 50,000
Accumulated depreciation (12,500)

37,500

SHOULD BE
Cost 100,000
Accumulated depreciation (80,000)

20,000

Dr Retained earnings 17,500


Cr Non current assets 17,500

Emile Woolf International 151 The Institute of Chartered Accountants of Pakistan


Financial accounting and reporting II

5.4 HALE
(a) Consolidated statement of financial position as at 31 December 2015
Rs.000
Assets
Non-current assets
Property, plant and equipment
(152,000 + 129,600 + 28,000 (W2)) 309,600
Goodwill (W3) 61,400
Current assets
Bank (41,000 + 8,000) 49,000
Receivables (104,000 + 84,000) 188,000
Inventory (112,000 + 74,400 3,200 (W6)) 183,200

791,200

Equity and liabilities
Capital and reserves
Share capital 100,000
Retained earnings (W5) 555,200

655,200
Non-controlling interest (W3) 60,000
Current liabilities (52,000 + 24,000) 76,000

791,200

WORKINGS
(1) Group structure

Hale

128
160
= 80% ord
ordords
ords

Sowen

(2) Net assets of Sowen


Reporting Date of Post
date acquisition acquisition
Rs.000 Rs.000
Share capital 160,000 160,000
Fair value adjustment on
non-current assets 28,000 28,000
Retained earnings 112,000 (11,000) 123,000
300,000 177,000

Emile Woolf International 152 The Institute of Chartered Accountants of Pakistan


Answers

(3) Goodwill
Rs.000
Cost of shares 203,000
Less Net assets acquired (80% 177,000 (W2)) (141,600)

61,400

(4) Non-controlling interest
Share of net assets (20% 300,000 (W2)) Rs.000
60,000

(5) Retained earnings
Rs.000
Hale 460,000
PURP (W6) (3,200)
Sowen (80% 123,000 (W2)) 98,400

555,200

(6) Unrealised profits
% Rs.000
SP 125 16,000
Cost (100) (12,800)

GP 25 3,200

5.5 HELLO
Consolidated statement of financial position as at 31 December 2015
Rs.
Assets
Non-current assets
Property, plant and equipment (225 + 175 + 10 2) 408,000
Goodwill (W3) 8,000

Current assets (271 + 157) 428,000



844,000

Equity and liabilities
Shareholders equity
Called up share capital 100,000
Retained earnings (W5) 291,800

391,800
Non-controlling interest (W4) 79,200
Current liabilities 373,000

844,000

Emile Woolf International 153 The Institute of Chartered Accountants of Pakistan


Financial accounting and reporting II

WORKINGS
(1) Group structure

Hello

60%

Solong

(2) Net assets of Solong Inc


Reporting Date of Post
date acquisition acquisition
Rs. Rs.
Share capital 100,000 100,000
Retained earnings
Per the question 90,000
Less: Fair value adjustment
for depreciation (2/10 10,000) (2,000)
88.000 60,000
Fair value adjustment 10,000 10,000
198,000 170,000

(3) Goodwill Rs. Rs.


Cost 110,000
Net assets acquired
60% 170,000 (W2) (102,000)

8,000

(4) Non-controlling interest Rs.


40% 198,000 (W2) 79,200

(5) Retained earnings Rs.


Hello 275,000
Solong (60% (88,000 60,000 (W2)) 16,800

291,800

Emile Woolf International 154 The Institute of Chartered Accountants of Pakistan


Answers

5.6 HASAN LIMITED


Hasan Limited
Consolidated statement of financial position as at 31 March 2015
Rs.000 Rs.000
Assets
Non-current assets
Property, plant and equipment (W1) 4,020
Goodwill (W4) 480
Software (W1) 1,440
Investments (65 + 210) 275

6,215
Current assets
Inventories (W2) 1,274
Trade receivables (524 + 328) 852
Cash and bank (20 + 55 cash in transit) 75

2,201

Total assets 8,416


Equity and liabilities


Capital and reserves
Equity capital 2,000
Reserves
Share premium 2,000
Retained earnings (W3) 2,420

4,420

6,420
Non-controlling interest (W5) 350
Non-current liabilities
Government grants (230 + 40) 270
Current liabilities
Trade payables (475 + 472) 947
Operating overdraft 27
Income tax liability (228 + 174) 402

1,376

Total equity and liabilities 8,416


Emile Woolf International 155 The Institute of Chartered Accountants of Pakistan


Financial accounting and reporting II

Workings
(W1) Property, plant and equipment
Rs.000
Balance from question Hasan Limited 2,120
Balance from question Shakeel Limited 1,990
Fair value adjustment on acquisition (see below) (120)
Over-depreciation re fair value adjustment year to 31 March 2015 30

4,020

A fair value of the leasehold based on the present value of the future
rentals (receivable in advance) would be the next (non-discounted)
payment of the rental plus the final three years as an annuity at 10%:
Rs.000
PV of rental receipts: Rs.80,000 + (Rs.80,000 2.50) 280
Carrying value on acquisition is (400)

Fair value reduction of leasehold (120)


The depreciation of the leasehold in Shakeel Limiteds accounts would be


Rs.100,000 per annum. However in the consolidated accounts it should be
Rs.70,000 (Rs.280,000/4). This would require a reduction in depreciation
of Rs.30,000 in the consolidated accounts for the next four years.
Software:
Shakeel Consolidated Difference
Limiteds figures
accounts
Rs.000 Rs.000
Capitalised amount 2,400 2,400
Depreciation to
31 March 2014 (300) 8 year life (480) 5 year life

Value at date of 180 fair
acquisition 2,100 1,920 value adjustment
Depreciation to 180 additional
31 March 2015 (300) (480) amortisation

Carrying value
31 March 2015 1,800 1,440

(W2) Inventories
Rs.000
Amounts given in the question (719 + 560) 1,279
Unrealised profit in inventories (25 25/125) (5)

1,274

Emile Woolf International 156 The Institute of Chartered Accountants of Pakistan


Answers

(W3) Retained earnings


Rs.000
Retained profits of Shakeel Limited, 31 March 2015 1,955
Adjustments:
Excess charge for leasehold depreciation 30
Insufficient charge for Software amortisation (180)
Unrealised profit in inventory (W2) (5)

Adjusted retained profits at 31 March 2015 1,800


Retained earnings of Shakeel Limited at 1 April 2014 2,200

Shakeel Limited: loss for the year (post-acquisition loss) (400)


Rs.000
Parent company share of post-acquisition loss (90%) (360)
Hasan Limited reserves at 31 March 2015 2,900
Goodwill impairment (120)

Consolidated retained profits at 31 March 2015 2,420


(W4) Goodwill
Rs.000
At acquisition date
Shares of Shakeel Limited 1,500
Share premium of Shakeel Limited 500
Retained earnings of Shakeel Limited 2,200
Fair value adjustments:
Leasehold (W1) (120)
Software (W1) (180)

3,900

Acquired by Hasan Limited (90%) 3,510


Cost of investment 4,110

Goodwill at acquisition 600


Impairment 120

Goodwill at 31 March 2015 480


Emile Woolf International 157 The Institute of Chartered Accountants of Pakistan


Financial accounting and reporting II

(W5) Non-controlling interests


Rs.000
Share capital of Shakeel Limited 1,500
Share premium of Shakeel Limited 500
Adjusted retained earnings of Shakeel Limited, 31 March 1,800
2015 (W3)
Fair value adjustments:
Leasehold (120)
Software (180)

Total net assets at 31 March 2015 3,500


Non-controlling interests (10%) 350


(W6) Elimination of current accounts:


Rs.000
Shakeel Limiteds current account with Hasan Limited per 75
question
Deduct cash in transit regarding this balance (15)

Adjusted figure to cancel 60


(W7) Elimination of intra-group loan:


Rs.000
Investment in Hasan Limiteds books 200
Deduct repayment in transit (40)

Non-current liability in Shakeel Limiteds books 160


Emile Woolf International 158 The Institute of Chartered Accountants of Pakistan


Answers

CHAPTER 6 CONSOLIDATED ACCOUNTS: STATEMENTS OF


COMPREHENSIVE INCOME
6.1 HARRY
Consolidated statement of profit or loss for the year ended 31 December 2015
Rs.000
Revenue 1,410
Cost of sales (733)

Gross profit 677
Distribution costs (90)
Administrative expenses (100)

Operating profit 487
Investment income 9
Finance costs (22)

Profit before tax 474
Income tax expense (165)

Profit after tax 309
Non-controlling interest (W3) (15)

Profit 294

Movement on consolidated retained earnings for the


year ended 31 December 2015
Retained earnings at 1 January 2014 (W4) 127
Retained earnings for the year 294
Dividends (50)

Retained earnings at 31 December 2015 (W5) 371

WORKINGS
(1) Group structure

Harry

75%

Sally

Emile Woolf International 159 The Institute of Chartered Accountants of Pakistan


Financial accounting and reporting II

(2) Consolidated statement of profit or loss

Harry Sally Adj Consol


Rs.000 Rs.000 Rs.000 Rs.000
Revenue 1,120 390 (100) 1,410
C of S per Q (610) (220) 100
PURP (3) (733)
Distribution costs (50) (40) (90)
Administrative expenses (55) (45) (100)
Investment income (20 15) 5 4 9
Interest payable (18) (4) (22)
Tax (140) (25) (165)

PAT 60

(3) Non-controlling interest
Rs.000
25% 60,000 (W1) or as per PAT in question 15

(4) Reserves brought forward


Rs.000
Harry 100
Sally (75% (45 9)) 27

127

(5) Reserves carried forward (proof)
Rs.000
Harry 317
PURP (3)
Sally (75% (85 9)) 57

371

(6) Inter-company dividend


Rs.000
Payable by Sally 20

Receivable by Harry (75% 20) 15

Emile Woolf International 160 The Institute of Chartered Accountants of Pakistan


Answers

6.2 HORNY
Consolidated statement of profit or loss for the year ended 31 December
2015
Rs.000
Revenue 362,000
Cost of sales (169,050)

Gross profit 192,950
Operating costs (93,817)

Operating profit 99,133
Investment income 13,100
Negative goodwill 3,800

Profit before tax 116,033
Income tax (48,400)

Profit after tax 67,633
Non-controlling interest (W3) (2,996)

Profit 64,637

Movement on consolidated retained earnings for the year ended


31 December 2015
Rs.000
Retained earnings at 1 January 2015 80,200
Retained profit for the year 64,637
Dividend (20,000)

Retained earnings at 31 December 2015 124,837

WORKINGS
(1) Group structure

Horny

75% (acq 31 August 2005)

Smooth

Emile Woolf International 161 The Institute of Chartered Accountants of Pakistan


Financial accounting and reporting II

(2) Consolidation schedule


Horny Smooth Adj Consol
4
12
Rs.000 Rs.000 Rs.000 Rs.000
Revenue 304,900 65,100 (8,000) 362,000
Cost of sales (144,200) (32,850) 8,000 (169,050)

Operating costs (76,450) (17,367) (93,817)


Investment income
of H 10,500
of S (all of it) 2,600 13,100

Tax (42,900) (5,500) (48,400)



PAT 11,983

(3) Non-controlling interest @ 25%



= 2,996

(4) Consolidated retained earnings carried forward - proof


Rs.000
Horny 112,050
Simpson (11,983 2,996) 8,987
Negative goodwill 3,800

124,837

Emile Woolf International 162 The Institute of Chartered Accountants of Pakistan


Answers

6.3 HERON
Consolidated statement of financial position as at 30 June 2015
Rs.000
Assets
Non-current assets
Property, plant and equipment (31,000 + 15,000) 46,000

Current assets (23,000 + 11,000) 34,000



80,000

Equity and liabilities
Shareholders equity
Share capital 10,000
Share premium account 5,000
2
Retained earnings (20,000 + ( 18,500)) 32,333
3

47,333
1
Non-controlling interest (3 20,000) 6,667
Non-current liabilities 15,000
Current liabilities (5,000 + 6,000) 11,000

80,000

Consolidated statement of profit or loss for the year ended 30 June 2015
Rs.000
Revenue (30,000 + 25,000) 55,000
Cost of sales (9,000 + 10,000) (19,000)

Gross profit 36,000
Distribution costs (3,000 + 1,200) (4,200)
Administrative expenses (1,000 + 2,800) (3,800)
Finance costs (2,000)

Profit before tax 26,000
Income tax expense (3,000 + 3,000) (6,000)

Profit for the period
20,000
1
Non-controlling interest (3 8,000) (2,667)

Profit for the financial year attributable to the members of Heron Inc 17,333

Consolidated statement of changes in equity for the year ended 30 June 2015
(extract)
2
Retained earnings brought forward (8,000 + (3 10,500)) 15,000
Profit for the financial year attributable to the members of Heron Inc 17,333

Retained earnings carried forward 32,333

Emile Woolf International 163 The Institute of Chartered Accountants of Pakistan


Financial accounting and reporting II

6.4 HANKS
Consolidated statement of financial position as at 31 December 2015
Rs.000 Rs.000
Assets
Non-current assets
Property, plant and equipment
(32,000 + 25,000 + 20,000 + 6,000) 83,000
Goodwill 4,500

87,500
Current assets
Cash at bank and in hand (9,500 + 2,000 + 4,000) 15,500
Receivables (20,000 + 8,000 + 17,000) 45,000
Inventory (30,000 + 18,000 + 18,000 2,100) 63,900

124,400

Total assets 211,900

Equity and liabilities
Share capital 40,000
Share premium account 6,500
Retained earnings (W5) 88,300

134,800

Non-controlling interest (W4) 28,100

Current liabilities
Trade payables (23,500 + 6,000 + 17,000) 46,500
Proposed dividends to minority shareholders (2,500 2,000) 500
to Hankss shareholders 2,000
49,000

Total equity and liabilities 211,900

Consolidated statement of profit or loss for the year ended 31 December 2015
Rs.000
Revenue (W6) 310,000
Cost of sales (W6) (159,100)

Gross profit 150,900
Distribution costs (W6) (51,000)
Administrative expenses (W6) (29,500)

Profit before taxation 70,400
Tax (W6) (24,000)

Profit after taxation 46,400
Non-controlling interest (W6) (9,200)

Profit 37,200

Emile Woolf International 164 The Institute of Chartered Accountants of Pakistan


Answers

Statement of movements on reserves for the year ended 31 December 2015


Share
Share premium Retained
Capital account earnings Total
Rs.000 Rs.000 Rs.000
At 1 January 2015 40,000 6,500 53,100 (W7) 99,600
Profit for the year 37,200 37,200
Dividends (proposed) (2,000) (2,000)

At 31 December 2015 40,000 6,500 88,300 134,800

WORKINGS
(1) Group structure

Hanks

80% 60%

Streep Scott

(2) Net assets


Streep
Reporting Date of Post
date acquisition acquisition
Rs.000 Rs.000
Share capital 10,000 10,000
Retained earnings
Per question 37,000
Proposed dividend (2,500)
34,500 7,500 27,000
44,500 17,500

Scott
Reporting Date of Post
date acquisition acquisition
Rs.000 Rs.000
Share capital 15,000 15,000
Retained earnings 27,000 3,000 24,000
Revaluation reserve 6,000 6,000
48,000 24,000

Emile Woolf International 165 The Institute of Chartered Accountants of Pakistan


Financial accounting and reporting II

(3) Goodwill on Streep


Rs.000
Cost of shares 20,500
Net assets acquired (80% 17,500) (W2) (14,000)

6,500

Of which:
Written off by start of the year (6,500 5,000) 1,500
Written off by end of the year (6,500 4,500) 2,000

Recognised as impairment during the year (balancing figure) 500

Goodwill on Scott
Rs.000
Cost of shares 13,000
Net assets acquired (60% 24,000 (W2)) (14,400)

(1,400)

(4) Non-controlling interest
Rs.000
Streep (20% 44,500 (W2)) 8,900
Scott (40% 48,000 (W2)) 19,200

28,100

(5) Consolidated retained earnings c/f


Rs.000
Hanks 55,000
Dividend receivable from Streep (80% of 2,500) 2,000
Proposed dividend (2,000)
Streep (80% 27,000 (W2)) 21,600
Scott (60% 24,000 (W2)) 14,400
30
PURP ((5,200 + 3,900) 130 ) (2,100)
Goodwill impairment Streep (2,000)
Negative goodwill Scott 1,400

88,300

Emile Woolf International 166 The Institute of Chartered Accountants of Pakistan


Answers

(6) Consolidation schedule


Hanks Streep Scott Adj Consol
Rs.000 Rs.000 Rs.000 Rs.000 Rs.000
Sales revenue 125,000 117,000 82,000 (14,000) 310,000
C of S per Q (65,000) (64,000) (42,000) 14,000
PURP (W5) (2,100)
(159,100)
Distrib (21,000) (14,000) (16,000)
(51,000)
Admin (14,000) (8,000) (7,000) (500)
(29,500)
Tax (10,000) (9,000) (5,000) (24,000)

PAT 22,000 12,000
Non-controlling interest in
profit after tax @20% @40%

4,400 + 4,800 = 9,200

(7) Consolidated retained earnings b/f


Rs.000
Hanks 40,000
Share of post acquisition profits of Streep (80% (15,000 7,500)) 6,000
Share of post acquisition profits of Scott (60% (15,000 3,000)) 7,200
Goodwill impairment - Streep (1,500)
Negative goodwill credited 1,400

53,100

Emile Woolf International 167 The Institute of Chartered Accountants of Pakistan


Financial accounting and reporting II

CHAPTER 7 TANGIBLE NON-CURRENT ASSETS (IAS 16: PROPERTY,


PLANT AND EQUIPMENT AND IAS 23: BORROWING COSTS)

7.1 ROONEY
(a) Borrowing costs
IAS 23 should be applied in accounting for borrowing costs.
Borrowing costs are recognised as an expense in the period in which they are
incurred unless they are capitalised in accordance with IAS 23 which says that
borrowing costs that are directly attributable to the acquisition, construction or
production of a qualifying asset can be capitalised as part of the cost of that
asset.
A qualifying asset is an asset that necessarily takes a substantial period
of time to get ready for its intended use or sale.
Borrowing costs that are directly attributable to acquisition, construction
or production are taken to mean those borrowing costs that would have
been avoided if the expenditure on the qualifying asset had not been
made.
When an enterprise borrows specifically for the purpose of funding an asset,
the identification of the borrowing costs presents no problem as the amount
capitalised is the actual borrowing costs net of any income earned on the
temporary investment of those borrowings.
If funds are borrowed, generally, the amount of borrowing costs eligible for
capitalisation is determined by applying a capitalisation rate to the
expenditures on that asset calculated as the weighted average of the
borrowing costs applicable to general borrowings.
IAS 23 also contains rules on commencement of capitalisation, suspension of
capitalisation and cessation of capitalisation.
Amount capitalised Rs.000
Cost of manufacture 28,000
Interest capitalised (Rs.20m 5% 2 years) 2,000

30,000

(b) Accounting
Rule
IAS 16 requires that each part of an item (that has a cost that is significant in
relation to the total cost) is depreciated separately. Therefore the cost
recognised at initial recognition must be allocated to each part accordingly.

Emile Woolf International 168 The Institute of Chartered Accountants of Pakistan


Answers

Accounting
(i) 31st March 2016
Carrying Depreciation Carrying
value value
1.4.2015 31.3.2016
Rs.000 Rs.000 Rs.000
Hydraulic system 9,000 3,000 6,000
Frame 21,000 2,625 18,375

30,000 5,625 24,375

Revaluation loss
(to profit and loss) (3,375)

Fair value. 21,000

The carrying value of the assets should be written down by a factor of
21,000/24,375. This gives a carrying value for the hydraulic system (in
Rs.000) of 5,169 and for the frame 15,831.

The hydraulic plant should be depreciated over two more years and the
frame over 7 more years.

(ii) 31st March 2017


Carrying Depreciation Carrying
value charge value
1.4.2016 31.3.2017
Rs.000 Rs.000 Rs.000
Hydraulic system 5,169 2,585 2,584
Frame 15,831 2,262 13,569

21,000 4,847 16,153
Revalued amount 19,600

Total gain 3,447

To statement of profit 3,375


or loss
Other comprehensive 72
income

Fair value 19,600

The total revaluation gain is 3,447. Of this total amount, 3,375 reverses
the loss in the previous year and is therefore reported in profit and loss for
the year. The remaining 72 is reported as other comprehensive income.

(Tutorial note: Deferred tax is ignored by this question.)

Emile Woolf International 169 The Institute of Chartered Accountants of Pakistan


Financial accounting and reporting II

7.2 EHTISHAM
IAS 16 permits assets to be carried at cost or revaluation. Where the latter is
chosen, the asset must be stated at its fair value.
The original depreciation was Rs. 40,000 (Rs. 1,000,000/25 years) per annum.
On 31st March 2014 the asset is two years old. Its carrying value before revaluation
was therefore Rs.1million less accumulated depreciation of Rs.80,000 (2/25 Rs. 1
million).
Rs.
Cost/valuation 1,000,000
Accumulated depreciation (80,000)
Net book value 920,000

In order to effect the revaluation, the cost is uplifted to fair value of Rs.1.15m, the
accumulated depreciation is eliminated, and the uplift to the net book value is
credited to a revaluation surplus account.
Debit Credit
Cost/valuation 150,000
Accumulated depreciation 80,000
Revaluation surplus 230,000
The impact of the journal is as follows:
Before Adjustment After
Cost/valuation 1,000,000 150,000 1,150,000
Accumulated depreciation (80,000) 80,000 nil
Net book value 920,000 1,150,000

The asset is depreciated over its remaining useful economic life of 23 years giving a
charge of Rs. 50,000 (Rs. 1,150,000/23 years) per annum in the year to 31st March 2015.
Debit Credit
Statement of profit or loss 50,000
Accumulated depreciation 50,000
This results in a carrying value as at 31st March 2015 of:
Rs.
Cost/valuation 1,150,000
Accumulated depreciation (50,000)
Net book value 1,100,000

Transfer from revaluation surplus to retained earnings


As a result of the revaluation, the annual depreciation has increased from Rs.40,000
to Rs.50,000. This extra depreciation of Rs.10,000 is transferred from the
revaluation reserve to accumulated profits each year.
Debit Credit
Revaluation surplus 10,000
Accumulated profits 10,000

Emile Woolf International 170 The Institute of Chartered Accountants of Pakistan


Answers

By the 31st March 2015, the balance remaining on the revaluation reserve will be
Rs.220,000.
Rs.
Surplus recognised at 31 March 2014 230,000
Transfer to accumulated profits (10,000)
Net book value 220,000

The fall in property values at the year-end. The asset must be revalued downwards
to Rs.0.8million, a write-down of Rs.300,000.
Rs.220,000 of this is charged against the revaluation reserve relating to this asset,
and the remaining Rs.80,000 must be charged against profits.
The reduction of the carrying amount of the asset is achieved by removing the
accumulated depreciation and adjusting the asset account by the balance.
Debit Credit
Revaluation surplus 220,000
Statement of profit or loss 80,000
Asset at valuation 350,000
Accumulated depreciation 50,000

The impact of the journal is as follows:


Before Adjustment After
Cost/valuation 1,150,000 350,000 800,000
Accumulated depreciation (50,000) 50,000 nil
Net book value 1,100,000 800,000

This balance is depreciated over the remaining useful life of the asset (22 years).

Emile Woolf International 171 The Institute of Chartered Accountants of Pakistan


Financial accounting and reporting II

7.3 CARLY
Financial statements for the year ended 31 December 2015 (extract)
Property, plant and equipment
Land and Plant and Computer
buildings machinery equipment Total
Rs. Rs. Rs. Rs.
Cost/valuation
At 1 January 2015 1,500,000 340,500 617,800 2,458,300
Revaluation 250,000 - - 250,000
Additions (W2) - 17,550 - 17,550
Disposals - (80,000) - (80,000)

At 31 December 2015 1,750,000 278,050 617,800 2,645,850

Accumulated depreciation
At 1 January 2015 600,000 125,900 505,800 1,231,700
Charge for the year (W1) 20,000 51,191 44,800 115,991
Revaluation (620,000) - - (620,000)
Disposals - (57,000) - (57,000)

At 31 December 2015 nil 120,091 550,600 670,691

Carrying amount
At 31 December 2014 900,000 214,600 112,000 1,226,600

At 31 December 2015 1,750,000 157,959 67,200 1,975,159

Workings
(1) Depreciation charges
Buildings = (1,500,000 500,000) 2% = 20,000.
Plant and machinery:
Rs.
New machine (17,550 25% /12)
9
3,291
Existing plant (((340,500 80,000) (125,900 57,000)) 25%) 47,900

51,191

Computer equipment = 112,000 40% = Rs.44,800

(2) Cost of new machine


Rs.
Purchase price (20,000 3,000 1,000) 16,000
Delivery costs 500
Installation costs 750
Interest on loan taken out to finance the purchase 300

17,550

Emile Woolf International 172 The Institute of Chartered Accountants of Pakistan


Answers

7.4 ADJUSTMENTS LIMITED


(a) Lathe
The lathe was purchased in 2009 and was originally being written off over an
estimated useful life of twelve years. As at 1 January 2015 six of the years
have elapsed with a further six years remaining. It was decided that the
machine will now only be usable for a further four years.
IAS 16 Property, plant and equipment requires that where the original
estimate of useful life is revised, adjustments should be made in current and
future periods (not in prior periods). The unamortised cost of the asset should
be charged to revenue over the remaining useful life of the asset. The net
book value of Rs.75,000 should therefore be charged over the remaining four
years of useful life, giving an annual depreciation charge of Rs.18,750.
The revision is not a change in accounting policy, or a fundamental error but a
change in accounting estimate. It is therefore not appropriate to deal with any
excess depreciation by adjusting opening retained earnings.
(b) Grinder
The grinder was purchased in 2012 and was originally being depreciated on a
straight line basis. It has now been decided to depreciate this on the sum of
digits basis.
IAS 16 requires that depreciation methods be reviewed periodically and if
there is a significant change in the expected pattern of economic benefits, the
method should be changed. Depreciation adjustments should be made in
current and future periods. This change might be appropriate if, for instance,
usage of the machine is greater in the early years of an assets life when it is
still new and consequently it is appropriate to have a higher depreciation
charge.
If the change is implemented, the unamortised cost (the net book value) of the
asset should be written off over the remaining useful life commencing with the
period in which the change is made. The depreciation charge for the
remaining life of the asset will therefore be as follows.

Year No of digits Depreciation


Rs.
2015 7 7/28 Rs.70,000 17,500
2016 6 6/28 Rs.70,000 15,000
2017 5 12,500
2018 4 10,000
2019 3 7,500
2020 2 5,000
2021 1 2,500

1/2 7 (7 + 1) 28 Rs. 70,000

Disclosure will need to be made in the accounts of the details of the change,
including the effect on the charge in the year.

Emile Woolf International 173 The Institute of Chartered Accountants of Pakistan


Financial accounting and reporting II

(c) Leasehold land


IAS 16s allowed alternative treatment in respect of measurement of property
plant and equipment (subsequent to initial recognition), is that of revaluation.
Revaluation is made at fair value.
Where any item of property plant or equipment is revalued, the entire class to
which the asset belongs should be revalued. Revaluations must be kept up to
date. Where there are volatile movements in fair value, the revaluation should
be performed annually. Where there are no such movements, revaluations
every three to five years may be appropriate.
Accumulated depreciation at the date of revaluation is either
(i) restated proportionately with the change in the gross carrying amount so
that the carrying amount after the revaluation equals the revalued
amount (e.g. where revaluations are made to depreciated replacement
cost using indices)
(ii) eliminated against the gross carrying amount of the assets and the net
amount restated to the revalued amount of the asset (e.g. where
buildings are revalued to their market value).
IAS 16 requires that the subsequent charge for depreciation should be based
on the revalued amount. The annual depreciation will therefore be Rs.62,500,
i.e. Rs.1,500,000 divided by the 24 years of remaining life.
There will then be a difference between the revalued depreciation charge and
the historical depreciation charge.
The resulting excess depreciation may be dealt with by a movement in
reserves, i.e. by transferring from the revaluation reserve to retained earnings
a figure equal to the depreciation charged on the revaluation surplus each
year.

Emile Woolf International 174 The Institute of Chartered Accountants of Pakistan


Answers

7.5 FAM
Accounting policies
(a) Property, plant and equipment is stated at historical cost less depreciation, or
at valuation.
(b) Depreciation is provided on all assets, except land, and is calculated to write
down the cost or valuation over the estimated useful life of the asset.
The principal rates are as follows.
Buildings 2% pa straight line
Plant and machinery 20% pa straight line
Fixtures and fittings 25% pa reducing balance

Fixed asset movements Land Plant Fixtures, Payments on


and and fittings, account and
buildings machinery tools and assets in the Total
equipment course of
construction
Cost/valuation Rs.000 Rs.000 Rs.000 Rs.000 Rs.000
Cost at 1 January 2015 900 1,613 390 91 2,994
Revaluation adjustment 600 600
Additions 154 40 73 (W1) 267
Reclassifications 100 (100)
Disposals (277) (41) (318)

Cost at 31 December 2015 100 1,490 389 64 2,043
2015 valuation 1,500 1,500

Depreciation
At 1 January 2015 80 458 140 678
Revaluation adjustment (80) (80)
Provisions for year (W2) 17 298 70 385
Disposals (195) (31) (226)

At 31 December 2015 17 561 179 757

Net book value


At 31 December 2015 1,583 929 210 64 2,786

At 31 December 2014 820 1,155 250 91 2,316


Land and buildings have been revalued during the year by Messrs Jackson &
Co on the basis of an existing use value on the open market.

Emile Woolf International 175 The Institute of Chartered Accountants of Pakistan


Financial accounting and reporting II

The corresponding historical cost information is as follows.


Land and buildings
Rs.000
Cost
Brought forward 900
Reclassification 100

Carried forward 1,000

Depreciation
Brought forward 80
Provided in year 10

Carried forward 90

Net book value 910

WORKINGS
Rs.000
(1) Additions to assets under construction 53
Deposit on computer 20

73

Rs.000
600
(2) Depreciation on buildings 40 + (100 2%) 17
2% straight line depreciation is equivalent to a 50 year life.
The buildings are ten years old at valuation and therefore
have 40 years remaining.
Depreciation on plant (1,613 + 154 277) 20% 298
Depreciation on fixtures (390 + 40 41 140 + 31) 25% 70

Emile Woolf International 176 The Institute of Chartered Accountants of Pakistan


Answers

7.6 IMRAN LIMITED


(a) Specific borrowings
Rs.
Borrowing costs incurred:
13% bank loan outstanding for 10 months
(Rs. 32 million x 306/365 x 13%) 3,487,562
11% bank loan outstanding for 5 months
(Rs. 10 million x 153/365 x 11%) 461,096
Borrowing costs 3,948,658
Less: Interest that relates to suspension
13% bank loan: (Rs. 32 million x 61/365 x 13%) 695,233
11% bank loan (Rs. 10 million x 61/365 x 11%) 183,836
(879,068)
3,069,590
Less: Investment income on temporary investment of the
borrowings (500,000)
2,569,590

(b) General borrowings


Phase 1 Phase 2 Phase 3
Building cost capitalised 20,000,000 18,000,000 16,000,000
Financed out of rights issue (15,000,000)
Financed from borrowing 5,000,000 18,000,000 16,000,000

Period to the year end


March 1 to December 31 306
April 1 to December 31 275
October 1 to December 31 92
Period of suspension (61) (61)
Number of days for which
borrowing should be capitalised 245 214 92

Weighted average borrowing


rate (W3) 12.73% 12.73% 12.73%
Fraction of the year for which
the rate should be applied to
costs incurred 245/365 214/365 92/365
Capitalised borrowing 427,240 1,343,451 513,385

Total 2,284,076

Emile Woolf International 177 The Institute of Chartered Accountants of Pakistan


Financial accounting and reporting II

Workings
W1: Average borrowings Rs.m
13% bank loan outstanding for 10 months
(Rs. 32 million x 306/365 days) 26,827,397
11% bank loan outstanding for 5 months
(Rs. 10 million x 153/365 days) 4,191,781
Average outstanding for the year 31,019,178

W2: Borrowing costs incurred (or from part a) Rs.m


13% bank loan outstanding for 10 months
(Rs. 32 million x 306/365 x 13%) 3,487,562
11% bank loan outstanding for 5 months
(Rs. 10 million x 153/365 x 11%) 461,096
Borrowing costs 3,948,658

W3: Weighted average rate


Borrowing costs
/ Average outstanding for the year = 3,948,658 (W2)/31,019,178 (W1) = 12.73%

Emile Woolf International 178 The Institute of Chartered Accountants of Pakistan


Answers

7.7 HUMAYUN CHEMICALS LIMITED


(a) If review is performed on June 30, 2015

Cost of machine Rs. 10,000,000


Depreciation charged @ 20% for the year ended June 30,
2013 and June 30, 2014
(Rs. 10,000,000 Rs. 3,000,000) x 20% x 2 Rs. 2,800,000
WDV as at June 30, 2014 Rs. 7,200,000
Residual value (10% of the cost of machine) Rs. 1,000,000
Depreciable amount - on July 1, 2014 Rs. 6,200,000
Remaining useful lives 6 years
Depreciation charge for the year ended June 30, 2015 Rs. 1,033,333

If review is performed on June 30, 2014

Cost of machine Rs. 10,000,000


Depreciation for the year ended June 30, 2013
(Rs. 10,000,000 - Rs. 3,000,000) x 20% Rs. 1,400,000
WDV as at June 30, 2013 Rs. 8,600,000
Residual value (10% of the cost of machine) Rs. 1,000,000
Depreciable amount - on July 1, 2013 Rs. 7,600,000
Remaining useful lives 6 years
Depreciation charge for the year ended June 30, 2015 Rs. 1,266,667
Depreciation charged in the financial statement for the
year ended June 30, 2014 Rs. 1,400,000
Effect of change in estimate to be incorporated (Reversal)
[Rs. 1,400,000 1,266,667) (Rs. 133,333)

(b) According to IAS-16, the following factors should be considered when


estimating the useful life of a depreciable asset:
(i) Expected usage

(ii) Expected physical wear and tear

(iii) Obsolescence

(iv) Legal or other limits on the use of the assets.

Once the useful life of a depreciable asset is determined, it shall be reviewed


at least at each financial year-end.

If expectations vary from the previous estimates, then change should be


adjusted for current and future periods in accordance with the requirements of
IAS 8.

Emile Woolf International 179 The Institute of Chartered Accountants of Pakistan


Financial accounting and reporting II

7.8 FARADAY PHARMACEUTICAL LIMITED


Debit Credit
Date Particulars
Rs.000 Rs.000
01.07.2011 Building 200,000
Bank 200,000
(Record purchase of plant)
30.06.2012 Depreciation 10,000
Accumulated depreciation Building 10,000
(Record depreciation for the year 2012)
Working: Rs. 200,000 20 = Rs. 10,000
01.07.2012 Accumulated depreciation Building 10,000
Building 10,000
(Reversal of prior year depreciation)
01.07.2012 Building 40,000
Surplus on revaluation of fixed assets 40,000
(Increase in value through revaluation)
Working: Rs. 230,000 Rs. 190,000 =
Rs. 40,000
30.06.2013 Depreciation 12,105
Accumulated depreciation Building 12,105
(Record depreciation for the year 2013)
Working: Rs. 230,000 19 = Rs. 12,105
30.06.2013 Surplus on revaluation of fixed assets 2,105
Retained earnings/Profit & loss account 2,105
(transfer of surplus through retained
earning to the extent of excess
depreciation)
Working: Rs. 40,000 19 = Rs. 2,105
01.07.2013 Accumulated depreciation Building 12,105
Building 12,105
(Reversal of prior year depreciation)
01.07.2013 Surplus on revaluation of fixed assets 37,895
Revaluation expense 10,000
Building 47,895
(Decrease in value through revaluation)
Working:
Reversal of Surplus balance (Rs. 40,000
Rs. 2,105) Rs. 37,895.
Balancing figure of Rs. 10,000 charged
to Profit and Loss
Building value decline: (Rs. 230,000
Rs. 12,105) Rs. 170,000 =Rs. 47,895
30.06.2014 Depreciation 9,444
Accumulated depreciation Building 9,444
(Record depreciation for the year 2014)
Working: Rs. 170,000 18 = Rs. 9,444
01.07.2014 Accumulated depreciation Building 9,444
Building 9,444
(Reversal of prior year depreciation)

Emile Woolf International 180 The Institute of Chartered Accountants of Pakistan


Answers

Debit Credit
Date Particulars
Rs.000 Rs.000
01.07.2014 Building 19,444
Revaluation income 9,444
Surplus on revaluation of fixed assets
(balancing) 10,000
(Reversal of prior year impairment)
Working:
Revaluation income = Rs. 10,000 [ Rs.
10,000 Rs. 9,444] = Rs. 9,444
Building: [Rs. 170,000 Rs. 9,444] Rs.
180,000 =Rs. 19,444
30.06.2015 Depreciation 10,588
Accumulated depreciation Building 10,588
(Record depreciation for the year 2015)
Working: Rs. 180,000 17 = Rs. 10,588
30.06.2015 Surplus on revaluation of fixed assets 588
Retained earnings 588
(Reverse the excess depreciation)
Working: Rs. 10,000 17 = Rs. 588

Emile Woolf International 181 The Institute of Chartered Accountants of Pakistan


Financial accounting and reporting II

7.9 SPIN INDUSTRIES LIMITED


Rupees
Commitment fee 125,000
Actual borrowing costs of specific loan (W1) 2,050,000
General borrowing costs (W1) 1,175,283
Less: Investment income (W2) (137,500)
Interest costs to be capitalised 3,212,783

W1
Outstanding Outstanding
Borrowing
amount month up to Rate of
cost (Rs.)
(Rs.) Months outstanding completion interest
Specific loan
Utilised till first repayment 25,000,000 1-Sep-14 31-Jan-15 5 12% 1,250,000
Utilised after the first
repayment 20,000,000 1-Feb-15 31-May-15 4 12% 800,000
2,050,000
General Borrowings (W4)
Utilised after specific loan
nd
exhausted on 2 payment
to contractor (W3) 8,125,000 1-Dec-14 31-May-15 6 12.08% 490,750
Principal payment of 12.08%
specific loan 5,000,000 1-Feb-15 31-May-15 4 201,333
3rd payment to contractor 12,000,000 1-Feb-15 31-May-15 4 12.08% 483,200
4rd payment to contractor 9,000,000 1-Jun-15 31-May-15 0 12.08% -
1,175,283

W2: Investment income


Surplus fund available from 1-Sep-14 to 30-Nov-14 (Rs. 25m Rs.
0.125m Rs. 8m Rs. 10m) 8% 3/12 Rs.137,500

W3: Specific loan utilization


Commitment fee 125,000
Payment for obtaining permit 8,000,000
1st payment to contractor 10,000,000
2nd payment to contractor (balancing) 6,875,000
25,000,000

2nd payment to contractor (total) 15,000,000


Less: paid out of specific loan (as worked out above) 6,875,000
Paid from general borrowing 8,125,000

W4: Weighted average rate of borrowing

Weighted average Interest


amount of loan (Rs.) (Rs.)
From Bank A 25,000,000 Rs. 25,000,000 13% 9/12 2,437,500
From Bank B 20,000,000 3,000,000
45,000,000 5,437,500

Weighted average rate of borrowing (Rs. 5,437,500 / 45,000,000) 12.08%

Emile Woolf International 182 The Institute of Chartered Accountants of Pakistan


Answers

7.10 SCIENTIFIC PHARMA LIMITED


Scientific Pharma Limited
Journal entries for the year ended June 30, 2015
Debit Credit
Rs.000 Rs.000
30.06.2015 Repair and maintenance expenses 1,500
Account payable / Bank 1,500
(Repair cost of major break down of the
plant)
30.06.2015 Depreciation expense (45,000-2,000)/10.5 years 4,095
Accumulated depreciation 4,095
(Depreciation expense for the year)
30.06.2015 Revaluation surplus (10,380/10.5) 989
Retained earnings 989
(Incremental depreciation credited to retained
earnings)
30.06.2015 Impairment loss W1 5,296
Property, plant and equipment 5,296
(Impairment of plant due to break
down)
30.06.2015 Revaluation surplus 5,296
Impairment loss W1 5,296
(Impairment loss adjusted against
revaluation)

W1: Impairment loss


Recoverable amount 19,227
WDV of the plant on impairment date W2 (24,523)
Impairment loss as on 30.06.2015 (5,296)
W2: WDV of the plant on impairment date Rs.000
FOB price (US$ 800,000 at Rs. 52) 41,600
Other charges including installation cost 7,000
48,600
Accumulated depreciation
(1-1-2006 to 30-6-2010) {(48,600-2,000)/15*4.5} (13,980)
WDV as on 30-6-2010 34,620
Revaluation surplus (45,000-34,620) 10,380
Revalued amount as of July 1, 2010 45,000
Accumulated depreciation
(1-7-2010 to 30-6-2015) {(45,000-2,000)/10.5*5) (20,476)
WDV as on 30-6-2015 24,523
W3: Revaluation surplus on impairment date
Revaluation surplus W2 10,380
Transferred to retained earnings
(01.07.2010 to 30.06.2015) (10,380/10.5*5) (4,943)
Revaluation surplus balance on impairment date 5,437

Since impairment loss is less than the revaluation surplus on impairment date,
the full amount of impairment would be adjusted against the revaluation surplus.

Emile Woolf International 183 The Institute of Chartered Accountants of Pakistan


Financial accounting and reporting II

7.11 QURESHI STEEL LIMITED


Capital work in progress Factory building Rs.000
Progress invoices received from the contractor
(30,000+20,000+10,000+15,000) 75,000.00
(Rain damages paid would be chargeable to profit and loss account/
insurance claim)

Borrowing costs to be capitalised:


Loan processing charges 500.00
Interest on bank loan W1 1,841.67
Interest on running finance W2 2,730.00
Interest income from surplus loan amount W4 (395.00)
Capital work in progress June 30, 2015 79,676.67

W1: Interest on bank loan:


Rs.000
Interest amount Outstanding Interest
From To Months loan amount at 13%
01-14-2014 31-05-2015 6 25,000 1625.00
01-06-2015 30-06 -2015 1 20,000 216.67
1,841.67

W2: Interest on running finance


Rs.000
Payments from Interest
Months
Payments at 15%
Payment Invoice outstanding
Description net of Right Bank Running per
s date amount up to
deductions issue loan finance annum
30-6-10
(W3)
01-07-14 Advanced
payment 10,000 10,000 10,000 12.00 1,500
st
15-10-14 1 progress
bill 30,000 25,500 15,000 10,500 8.50 1,116
nd
15-01-15 2
progress
bill 20,000 17,000 17,000 - - -
rd
15-04-15 3
progress
bill 10,000 8,500 7,500 1,000 2.50 31
31-05-15 Loan
interest 1,625 1,625 1.00 20
31-05-15 Loan
instalment 5,000 5,000 1.00 63
15,000 *24,500 29,125 2,730

*Loan amount of Rs. 25,000,000 less processing charges of Rs. 500,000


W3: Average rate of interest for running finance facility (9,000/60,000) 15%

W4: Interest income from surplus loan amounts:


Rs.000
Interest income Interest
Surplus loan
income
amounts
From To Months at 8%
01-14-14 15-01-15 1.5 24,500 (245)
16-01-15 15-04-15 3.0 7,500 (150)
(395)

Emile Woolf International 184 The Institute of Chartered Accountants of Pakistan


Answers

7.12 GRANITE CORPORATION


Borrowing costs to be capitalised
Workings 2015 2014
Commitment fee @ 1% - 700,000
Borrowing costs on specific loan 1 6,987,500 3,033,333
Borrowing costs on running finance 3 1,381,625 -

Less: Investment income 2 (2,099,001) (1,381,334)


6,720,124 2,351,999

W1 : Actual borrowing costs on specific loan

to be capitalised
Net outstanding

Borrowing cost
amount (Rs.)
Outstanding

Outstanding

(Rs.) @ 13%
Suspension

months
month
From commencement on to June 30 70,000,000 4 0 3,033,333
Amount to be capitalised as on 30-Jun-2014 3,033,333

From June 30 to first principal repayment 70,000,000 2 0 2 1,516,667


After the 1st principal repayment 65,000,000 6 1 5 3,520,833
After the 2nd principal repayment to completion 60,000,000 3 0 3 1,950,000
Amount to be capitalised as on 30-Jun-2015 6,987,500

W2 : Investment income (All amounts in Rupees)


Available O/s amount up Used to reduce Invested in saving Total
Funds to completion running finance (14%) account @ 8% Income
Amount Income Amount Income
Rs.(70m -
25m - 0.7m) 44,300,000 4 10,000,000 466,667 34,300,000 914,667 1,381,334
Investment income 2014 1,381,334

Rs. (70m -
25m -
0.7m) 44,300,000 2 10,000,000 233,333 34,300,000 457,333 690,666
Rs.(44.3 -
5m -
4.55m) 34,750,000 5 10,000,000 583,335 24,750,000 825,000 1,408,335
Investment income 2015 2,099,001

W3 : Interest on running finance


2015
to be capitalised
Borrowing cost

(Rs.) @ 14%
No. of months
outstanding

outstanding
Suspension

months

Description Amount
Net

2nd payment to contractor (Rs. 65m - 34.75m) 30,250,000 4 1 3 1,058,750


Payment of 2nd instalment
Principal 5,000,000 3 0 3 175,000
Interest (Rs. 65m x 13% x 6/12) 4,225,000 3 0 3 147,875
3rd payment to contractor 10,000,000 0 0 0 -
49,475,000 1,381,625

Emile Woolf International 185 The Institute of Chartered Accountants of Pakistan


Financial accounting and reporting II

CHAPTER 8 IAS 38: INTANGIBLE ASSETS


8.1 FAZAL
In accordance with IAS 38, expenditure on intangible assets must be expensed
unless it meets the recognition criteria for capitalisation. These criteria require the
demonstration that future benefits will arise from the incurred costs. It would be
difficult to prove that this is the case in relation to training costs and IAS 38
specifically states that training costs should always be expensed as they are
incurred and not treated as an intangible asset.
Hence the treatment adopted by Fazal is not correct and the costs being carried
forward must be expensed to the years profits.

8.2 HENRY
Property, plant and equipment
Plant and machinery
Cost Rs.
On 1 January 2015 X
Additions 30,000

On 31 December 2015 X

Accumulated depreciation
On 1 January 2015 X
Charge for the year (30,000 9/12 5) 4,500

On 31 December 2015 X

Carrying amount
On 31 December 2014 X

On 31 December 2015 25,500

Intangible assets
Internally generated research and development expenditure
Cost Rs.
On 1 January 2015 412,500
Additions 45,000

On 31 December 2015 457,500

Accumulated amortisation
On 1 January 2015 -
Charge for the year (W) 68,750

On 31 December 2015 68,750

Carrying amount
On 31 December 2014 412,500

On 31 December 2015 388,750

Emile Woolf International 186 The Institute of Chartered Accountants of Pakistan


Answers

Working
Amortisation charge (Project A)
Rs.
Total savings (100,000 + 300,000 + 200,000) 600,000
2015 amortisation charge (100,000/600,000 412,500) 68,750

Tutorial notes
The costs in respect of Project B cannot be capitalised as there are uncertainties
surrounding the successful outcome of the project but the machine bought may be
capitalised in accordance with IAS16.
The 2015 costs in respect of Project C can be capitalised as the uncertainties have
now been resolved. However, the 2014 costs cannot be reinstated.

8.3 TOBY
Intangible assets
Goodwill Patents Brands Total
Rs. Rs. Rs. Rs.
Cost
On 1 January 2015 - - - -
Additions (W1) 10,000 20,000 50,000 80,000

On 31 December 2015 10,000 20,000 50,000 80,000

Accumulated amortisation/impairment
On 1 January 2015 - - - -
Written off/amortised during the year
(W1 and W2) 3,000 2,500 7,500 13,000

On 31 December 2015 3,000 2,500 7,500 13,000

Carrying amount
On 31 December Year 0 - - - -

On 31 December 2015 7,000 17,500 42,500 67,000

Workings
(1) Goodwill on acquisition of George
Rs.
Cost of acquisition 105,000
Minus fair value of net assets acquired (100,000 5,000) (95,000)

Goodwill 10,000
Recoverable value (7,000)

Impairment write off 3,000

(2) Amortisation of patent
20,000 8 = Rs.2,500
(3) Amortisation of brand
50,000 5 9/12 = Rs.7,500
Tutorial note
IAS38 Intangible assets prohibits the recognition of internally generated brands (3)
or internally-generated goodwill (4).

Emile Woolf International 187 The Institute of Chartered Accountants of Pakistan


Financial accounting and reporting II

8.4 BROOKLYN
1 Development expenditure
IAS 38 on intangibles requires that research and development be considered
separately:
research which must be expensed as incurred
development which must be capitalised where certain criteria are met.
It must first be clarified how much of the Rs.3 million incurred to date (10
months at Rs.300,000) is simply research and how much is development. The
development element will only be capitalised where the IAS 38 criteria are
met. The criteria are listed below together with the extent to which they appear
to be met.
The project must be believed to be technically feasible. This appears to
be so as the feasibility has been acknowledged.
There must be an intention to complete and use/sell the intangible.
Completion is scheduled for June 2016
The entity must be able to use or sell the intangible. Interest has been
expressed in purchasing the knoWhow on completion
It must be considered that the asset will generate probable future
benefits. Confirmation is required from Brooklyn as to the extent of
interest shown by the pharmaceutical companies and whether this is of a
sufficient level to generate orders and to cover the deferred costs.
Availability of adequate financial and technical resources must exist to
complete the project. The financial position of Brooklyn must be
investigated. A grant is being obtained to fund further work and the
terms of the grant, together with any conditions, must be discussed
further.
Able to identify and measure the expenditure incurred. A separate
nominal ledger account has been set up to track the expenditure.
If all of the above criteria are met, then the development element of the Rs.3m
incurred to date must be capitalised as an intangible asset. Amortisation will
not begin until commercial production commences.
2 Provision
Although the claim was made after the reporting period, IAS 10 considers this
to be an adjusting event after the reporting period. The employment of the
individual dates back to 20X2 and so the lawsuit constitutes a current
obligation for the payment of damages as a result of this past event (the
employment).
The amount and the timing are not precisely known but the likelihood of
payment of damages by Brooklyn is probable and so a provision should be
made for the estimated amount of the liability, as advised by the lawyer.
Disclosure, rather than provision, would only be appropriate if the expected
settlement was possible or remote, and the lawyers view is that a payment is
more likely than not.
It is not appropriate to calculate an expected value where there is only one
event, instead a provision should be made for the most likely outcome. The
lawyer has various views on the possible payout, but the most likely payout is
Rs.500,000 as this has a 50% probability. As settlement of the provision is not
anticipated until 2018, the provision should be discounted back at 8% to give a
liability of Rs.476,280.

Emile Woolf International 188 The Institute of Chartered Accountants of Pakistan


Answers

Provided that the payment from the insurance company is virtually certain, this
should be shown as an asset, also at its discounted value of Rs.47,628, being
10% of the provision.
In both cases the discounting should be unwound over the coming three years
through profit or loss.
3 Revaluation
IAS 16 on Property, Plant and Equipment does not impose a frequency for
updating revaluations. It simply requires a revaluation where it is believed that
the fair value of the asset has materially changed. Hence, if in the past there
have been material differences between the carrying amount and fair value at
the 5 yearly review then Brooklyn should consider having more frequent
valuations following on from this years valuation.
Revaluations should be regular and not timed simply when property prices are
at a peak. It is not acceptable for Brooklyn to defer its next revaluation while
values are low. If property prices do fall in 2016, then it may be necessary to
perform an impairment test in accordance with IAS 36 Impairment of assets.
If it is believed that an asset value has moved materially, then all assets in that
class must be revalued. Hence it is not sufficient for Brooklyn to just revalue
the London property.
IAS 16 does not require the valuation to be performed by an external party,
and so the use of the property manager to conduct the valuations is
acceptable. Notes to the financial statements will disclose that he is not
independent of the company.

8.5 ZOUQ INC


(a) (i) The depreciable amount of an intangible asset with a finite useful life
shall be allocated on a systematic basis over its useful life.
(ii) Amortization shall begin when the asset is available for use
(iii) Amortization shall cease at the earlier of the date that the asset is
classified as held for sale and the date that the asset is derecognised.
(iv) The amortization method used shall reflect the pattern in which the
asset's future economic benefits are expected to be consumed by the
entity.
(v) The amortization charge for each period shall be recognised in
statement of profit or loss.

(b) Goodwill Account


Rupees Rupees
Goodwill recognised Impairment of
01.01.2014 (W1) 270,000,000 31.12.2014 goodwill 50,000,000
31.12.2014 Balance b/d 220,000,000

270,000,000 270,000,000
01.01.2015 Balance b/d 220,000,000
31.12.2015 Balance b/d 220,000,000

220,000,000 220,000,000

Emile Woolf International 189 The Institute of Chartered Accountants of Pakistan


Financial accounting and reporting II

Brand Account
Rupees Rupees
01.01.2014 Brand
recognised 100,000,000 31.12.2014 Amortization 10,000,000
31.12.2014 Balance c/d 90,000,000

100,000,000 100,000,000

01.01.2015 Balance b/d 90,000,000 31.12.2015 Amortization 10,000,000


Impairment of
31.12.2015 Brand 13,500,000
- 31.12.2015 Balance c/d 68,000,000

90,000,000 90,000,000

W1: Value of goodwill


Rupees
Purchase price (50,000,000 x Rs. 30 x 90%) 1,350,000,000
Less: Fair value of net identifiable assets and liabilities
(Rs. 1,100,000,000 x 90%) (990,000,000)
Less: Value of brand (Rs. 100,000,000 x 90%) (90,000,000)
Goodwill recognised 270,000,000

8.6 STAR-BRIGHT PHARMACEUTICAL LIMITED


Star-Bright Pharmaceutical Limited
Statement of financial position
As at December 31, 2015
2014
2015 Restated
Rs. in million
Non-current assets
Intangible asset brand [Note 8] 274 285
Shareholders equity
Retained earnings
(W5 and 6) 2,071 1,879

Star-Bright Pharmaceutical Limited


Statement of Financial Position
As at December 31, 2015
8- Intangible assets Brand
Cost
At beginning of the year (2015: 382+24+54+38,
2014: 382+ 24+54) 498 460
Capitalised during the year 43 38
541 498
Amortization
At beginning of the year (W1 and 2) (213) (163)
*3 *4
During the year (W3 and 4) (54) (50)
(267) (213)
274 285

W1 : 382 x 50% + 24 x 30% + 54 x 20% + 38 x 10% = 213


W2 : 382 x 40% + 24 x 20% + 54 x 10% = 163
W3 : 541 x 10% = 54
W4 : 498 x 10% = 50
W5 : 1,950 + 24 + 54 + 38 + 43 [267 (382 x 60%)] = 2,071
W6 : 1,785 + 24 + 54 + 38 [213 (382 x 50%)] = 1,879

Emile Woolf International 190 The Institute of Chartered Accountants of Pakistan


Answers

8.7 RAISIN INTERNATIONAL


(a) Following are the criteria that should be used while recognizing intangible
assets from research and development work.
(i) No intangible asset arising from research shall be recognised.
(ii) An intangible arising from development shall be recognised if, and
only if , an entity can demonstrate all of the following:
the technical feasibility of completing the intangible asset so that
it will be available for use or sale.
its intention to complete the intangible asset and use or sell it.
its ability to use or sell the intangible asset.
how the intangible asset will generate probable future economic
benefits. Among other things, the entity can demonstrate the
existence of a market for the output of the intangible asset or
the intangible asset itself or, if it is to be used internally, the
usefulness of the intangible asset.
the availability of adequate technical, financial and other resources
to complete the development and to use or sell the intangible asset.
its ability to measure reliably the expenditure attributable to the
intangible asset during its development.
(b) (i) Since the product met all the criteria for the development of the
product, it should be recognised as an intangible in the statement of
financial position (SOFP) of the company. However, RI should
capitalise only the development work (i.e. Rs. 9 million) as intangible
asset. IAS-38 does not allow capitalization of cost relating to the
research work, training of staff and cost of trial run.
Since the product has a useful life of 7 years, the amortization expense
amounting to Rs. 0.32 million (Rs. 9 million 3/12 7 years) should be
recorded in the statement of profit or loss.
(ii) This purchasing of right to manufacture should be recognised as an
intangible in the SOFP because:
it is for an established product which would generate future
economic benefits.
cost of the patent can be measured reliably.

Since there is a finite life, the patent must be amortised over its useful
life. The useful life will be shorter of its actual life (i.e. 10 years) and its
legal life (i.e. 5 years. The amortization to be recorded in SOCI is Rs.
2.83 million (Rs. 17 million 10/12 5).
(iii) The acquired brand should be recognised as an intangible in the SOFP
because acquisition price is a reliable measure of its value. The
amortization to be recorded in SOCI is Rs. 0.12 million (Rs. 2 million
10 years x 7/12).
(iv) The carrying value of the intangible asset should be increased to Rs.
10 million in the SOFP. Since there is an indefinite useful life of the
intangible assets, it should not be amortised. Instead, RI should test
the intangible asset for impairment by comparing its recoverable
amount with its carrying amount.

Emile Woolf International 191 The Institute of Chartered Accountants of Pakistan


Financial accounting and reporting II

CHAPTER 9 IAS 17: LEASES

9.1 DAWOOD
The lease has been correctly classified as a finance lease as it is being leased for its
entire useful economic life which indicates that the risks and rewards of ownership
have been transferred to Dawood.
The leased asset should be capitalised as a non-current asset and depreciated over
the 5 year lease period/useful life. By 31st March 2015, the net book value of the
asset will be Rs.225,000, being the cost of Rs.250,000 less 6 months depreciation.
A finance lease creditor should be established initially for Rs.250,000. During the
year, finance costs will be added and the first payment of Rs.29,500 will be
deducted.
The finance costs on the lease of Rs.45,000 (Being total payments of (10
Rs.29,500) cash price Rs.250,000) will be spread over the lease term using the
sum-of-digits method.
n (n 1) 9 10
Sum of digits = = 45
2 2

(Note: n = number of periods of borrowing, and as the payments are in ADVANCE


not arrears, Dawood is only financing the asset of 4 1/2 years (9 six-monthly
periods)).
Therefore, the finance cost relating to the first 6 months through to 31st March 2015
is Rs.9,000 (9/45 Rs.45,000).
This will result in a movement on the finance lease creditor as follows:
6 months to Brought forward Payment Finance cost Carried forward
31 March 13 250,000 (29,500) 9,000 229,500
30 Sept 13 229,500 (29,500) 8,000 208,000
31 March 14 208,000 (29,500) 7,000 185,500
The year end liability of Rs.229,500 will be split between current liabilities Rs.51,000
(29,500 + (29,5008,000)), and the balance of Rs.178,500 as non-current liabilities.

9.2 FINLEY
Financial statements for the year ended 31 December 2015 (extracts)
Statement of financial position
Non-current assets Rs.
Property, plant and equipment (36,000 9,000) 27,000
Current liabilities
Finance lease obligations (W1) 10,000
Non-current liabilities
Finance lease obligations (W1) 17,950

Statement of profit or loss


Depreciation on leased assets ((36,000 4) 9,000
Finance lease charges (W1) 1,950

Emile Woolf International 192 The Institute of Chartered Accountants of Pakistan


Answers

Workings
(1) Finance lease obligations (boat)
Opening Lease Capital Interest Closing
Year ended balance payment outstanding at 7.5% balance
Rs. Rs. Rs. Rs. Rs.
31 December 2015 36,000 (10,000) 26,000 1,950 27,950
31 December Year 5 27,950 (10,000) 17,950 1,346 19,296

Rs.
Current (balancing figure) 10,000
Non-current 17,950

27,950

9.3 FABIAN
Financial statements for the year ended 31 December 2015 (extracts)
Statement of financial position
Non-current assets Rs.
Property, plant and equipment (126,760 31,690) 95,070

Current assets
Trade and other receivables (W1) 6,250

Current liabilities
Finance lease obligations (W2) 30,056
Non-current liabilities
Finance lease obligations (W2) 69,380
Statement of profit or loss
Operating expenses
Operating lease rentals (W1) 5,450
Depreciation on leased assets (126,760 4) 31,690

Finance costs
Finance lease charges (W2) 12,676
Tutorial note
The notes to the financial statements would disclose the fact that included in trade
and other receivables is Rs.3,750 (W1) due in more than one year.

Emile Woolf International 193 The Institute of Chartered Accountants of Pakistan


Financial accounting and reporting II

Workings
(1) Operating lease (car)
Rs.
statement of profit or loss charge = ((7,500 + (36 700)) 6/36) 5,450
=

Cash paid in 2015 (7,500 + (700 6)) 11,700
Minus charged to statement of profit or loss in 2015 (5,450)

Prepayment at end of 2015 6,250

Prepayment at end of 2015 6,250
Cash paid in 2016 (12 700) 8,400
Minus charged to statement of profit or loss in 2016 (5,450 2) (10,900)

Prepayment at end of 2016 3,750

(2) Finance lease obligations (machine)
Opening Lease Closing
Date balance Interest (10%) payment balance
Rs. Rs. Rs. Rs.
2015 126,760 12,676 (40,000) 99,436
2016 99,436 9,944 (40,000) 69,380

Rs.
Current (balancing figure) 30,056
Non-current 69,380

99,436

9.4 XYZ INC


(a) Extracts from the financial statements of XYZ Inc at 31 December 2015
Statement of financial position
(i) Tangible fixed assets held under finance leases
Plant and machinery
Rs.000
Cost
At 1 January 2015 x
Additions 4,400

At 31 December 2015 4,400

Accumulated depreciation
At 1 January 2015 x
Charge for the year 629

At 31 December 2015 629

Net book value
At 31 December 2015 3,771

At 1 January 2015 x

Emile Woolf International 194 The Institute of Chartered Accountants of Pakistan


Answers

(ii) Finance lease payables


Amounts payable:
Rs.000
Within one to five years (600 8 284) 4,516
Less future finance charges 996

3,520

Accruals
Rs.000
Finance leases (667 + 284) 951

Statement of profit or loss

Profit is stated after charging


Rs.000
Finance charges 604 (W2)
Depreciation 4,400 7 629

(b) Table

Period ended Amount Repaid Capital due 7.68% Amount due


borrowed for period interest at period end
Rs.000 Rs.000 Rs.000 Rs.000 Rs.000
30 June 2015 4,400 (600) 3,800 292 4,092
31 December 2015 4,092 (600) 3,492 268 3,760
30 June 2016 3,760 (600) 3,160 243 3,403
31 December 2016 3,403 (600) 2,803 215 3,018
30 June 2017 3,018 (600) 2,418 186 2,604
31 December 2017 2,604 (600) 2,004 154 2,158
30 June 2018 2,158 (600) 1,558 119 1,677
31 December 2018 1,677 (600) 1,077 83 1,160
30 June 2019 1,160 (600) 560 40 600
31 December 2019 600 (600)

Comparison

Period Sum of digits (W2) Actuarial (as above)


Rs.000 Rs.000
1 320 292
2 284 268
3 249 243
4 213 215
5 178 186
6 142 154
7 107 119
8 71 83
9 36 40
10

1,600 1,600

Emile Woolf International 195 The Institute of Chartered Accountants of Pakistan


Financial accounting and reporting II

WORKINGS
(1) Calculation of finance charge
Rs.000
Minimum lease payments 5 600 2 6,000
Fair value of asset (4,400)

Finance charge 1,600

(2) Allocation of finance charge

Period ended Digits Finance charge


Rs.000
30 June 2015 9 9/45 1,600 320
31 December 2015 8 8/45 1,600 284

604
30 June 2016 7 7/45 1,600 249
31 December 2016 6 6/45 1,600 213
30 June 2017 5 5/45 1,600 178
31 December 2017 4 4/45 1,600 142
30 June 2018 3 3/45 1,600 107
31 December 2018 2 2/45 1,600 71
30 June 2019 1 1/45 1,600 36
31 December 2019

n(n + 1) 9(9 + 1)
= 45 1,600
2 2

(3) Lease obligation


Period ended Amount Repaid Capital Interest Amount
borrowed due for due at
period period
end
Rs.000 Rs.000 Rs.000 Rs.000 Rs.000
30 June 2015 4,400 (600) 3,800 320 4,120
31 December 2015 4,120 (600) 3,520 284 3,804
30 June 2016 3,804 (600) 3,204 249 3,453
31 December 2016 3,453 (600) 2,853 213 3,066

Emile Woolf International 196 The Institute of Chartered Accountants of Pakistan


Answers

9.5 SNOW INC


Extracts from the financial statements of Snow Inc for year ended 31
December 2015
Statement of profit or loss
Profit is stated after charging
Rs.000
Finance charges (1,714 + 1,429 + 9,614) (W1 and 2) 12,757
Depreciation 41,667

Statement of financial position


Tangible fixed assets held under finance leases
Plant and
machinery
Cost Rs.000
At 1 January 2015
Additions (35,000 + 150,000) 185,000

At 31 December 2015 185,000

Accumulated depreciation
At 1 January 2015
Charge for year
35,000 15,000
+ 41,667
3 5

At 31 December 2015 41,667

Net book value
At 31 December 2015 143,333

At 1 January 2015

Finance lease payables

Amounts payable:
Rs.000
Within one to five years 166,000 (6,500 4 + 35,000 4)
Less future finance charges 18,243 (2,857 + 15,386 *)

147,757

* 35,000 5 = 175,000 150,000 9,614 = 15,386

Accruals
Rs.000
Finance leases 46,000 (11,000 + 35,000)

Emile Woolf International 197 The Institute of Chartered Accountants of Pakistan


Financial accounting and reporting II

WORKINGS

(1) Snowplough

(a) Calculation of finance charge


Rs.000
Deposit 2,000
MLP (6 6,500) 39,000
Fair value of asset (35,000)

Finance charge 6,000

(b) Allocation of finance charge

Period Digits Finance


ended charge
Rs.000
30.06.2015 6
621 6,000 1,714
31.12.2015 5
521 6,000 1,429
30.06.2016 4
421 6,000 1,143
31.12.2016 3
321 6,000 857
30.06.2017 2
221 6,000 571
31.12.2017 1
121 6,000 286

21 6,000

n (n + 1) 6 (7)
= = 21
2 2

(c) Period Capital Interest Amount Repayment Capital


ended O/S at start O/S at end O/S at end
Rs.000 Rs.000 Rs.000 Rs.000 Rs.000
30.6.15 33,000 1,714 34,714 (6,500) 28,214
31.12.15 28,214 1,429 29,643 (6,500) 23,143
30.6.16 23,143 1,143 24,286 (6,500) 17,786
31.12.16 17,786 857 18,643 (6,500) 12,143

(2) Snow machine

Period Amount Repayment Capital Interest Amount


ended O/s at start O/S at start at 8.36% O/S at end
Rs.000 Rs.000 Rs.000 Rs.000 Rs.000
31.12.15 150,000 (35,000) 115,000 9,614 124,614
31.12.16 124,614 (35,000) 89,614 7,492 97,106

Emile Woolf International 198 The Institute of Chartered Accountants of Pakistan


Answers

9.6 MIRACLE TEXTILE LIMITED


Miracle Textile Limited
Statement of financial position (Extracts)
As at 30 June 2015
Note 2015 2014
ASSETS Rs. Rs.
Non-current assets
Property, plant and equipment 4 16,000,000 18,000,000

LIABILITIES
Non-current liabilities
Obligation under finance lease 9 6,505,219 10,633,074

Current liabilities
Current portion of obligation under finance
lease 9 4,127,856 3,566,925

Miracle Textile Limited


Notes to the financial statements (Extracts)
As at 30 June 2015

4- Property, plant and equipment 2015 2014


Rs.000 Rs.000
Leased assets
Cost
Opening balance 20,000,000 -
Addition during the year - 20,000,000
20,000,000 20,000,000
Accumulated depreciation
Opening balance (2,000,000) -
Depreciation for the year (2,000,000) (2,000,000)
(4,000,000) (2,000,000)
Balance as at 30 June 16,000,000 18,000,000

9- Obligations under finance lease (W1)


30-Jun-2015 30-Jun-2014
Financial Financial
Minimum Minimum
charges Principal charges Principal
lease lease
for future outstanding for future outstanding
payment payment
periods periods
Rs. Rs. Rs. Rs. Rs. Rs.
Not later
than one
year 5,800,000 1,672,144 4,127,856 5,800,000 2,233,075 3,566,925
Later than
one year but
not later than
five years 7,800,000 1,294,781 6,505,219 13,600,000 2,966,926 10,633,074
Later than
five years - - - - - -
13,600,000 2,966,926 10,633,074 19,400,000 5,200,000 14,200,000

Emile Woolf International 199 The Institute of Chartered Accountants of Pakistan


Financial accounting and reporting II

9.1 The Company has entered into a finance lease agreement with a bank in
respect of a machine. The finance lease liability bears interest at the rate of
15.725879% per annum. The company has the option to purchase the
machine by paying an amount of Rs. 2 million at the end of the lease term.
The lease rentals are payable in annual instalments ending in June 2015.
There are no financial restrictions in the lease agreement.

W1: Lease Schedule


Payment Opening Principal Interest @ Closing
Instalment
date principal repayment 15.725879% principal
01.07.2013 20,000,000 5,800,000 5,800,000 - 14,200,000
01.07.2014 14,200,000 5,800,000 3,566,925 2,233,075 10,633,075
01.07.2015 10,633,075 5,800,000 4,127,856 1,672,144 6,505,219
01.07.2016 6,505,219 5,800,000 4,776,997 1,023,003 1,728,222
30.06.2017 1,728,222 2,000,000 1,728,222 271,778 -
20,000,000 5,200,000

9.7 SHOAIB LEASING LIMITED


Entries in the books of Lessor

Date Particulars Dr. Cr.


01.07.2015 Lease payments receivable (W1) 2,680,000
Machine 2,100,000
Unearned finance income (W1) 580,000

30.06.2016 Bank 860,000


Lease payments receivable 860,000

30.06.2016 Unearned finance lease 272,941


Finance income (W2) 272,941

30.06.2017 Bank 860,000


Lease payments receivable 860,000

30.06.2017 Unearned finance lease 196,640


Finance income (W2) 196,640

30.06.2018 Bank 960,000


Lease payments receivable 960,000

30.06.2018 Unearned finance lease 110,419


Finance income (W2) 110,422

W1: Total finance income Rupees


Total future lease payments (Rs. 860,000 x 3) 2,580,000
Add: Purchase bargain option 100,000
Gross investment in finance lease 2,680,000
Less: Cost of assets 2,100,000
Total finance income 580,000

Emile Woolf International 200 The Institute of Chartered Accountants of Pakistan


Answers

W2: Amortization schedule


Principal Principal
Instalment Interest Principal
Date Opening Closing
Rs. Rs. Rs. Rs. Rs.
30.06.2016 2,100,000 860,000 272,941 587,059 1,512,941.20
30.06.2017 1,512,941 860,000 196,640 663,360 849,581.19
30.06.2018 849,581 960,000 110,419* 849,581 nil
580,000 2,099,997.04

Note that there is a rounding adjustment of Rs. 3 in the last interest amount.
Shoaib Leasing Limited
Extracts from the statement of financial position as at June 30, 2016

2016
Rupees
Non-current assets Note 10
Net investment in leases 849,578

Current assets
Current portion of net Investment in leases 663,360

10 Net investment in leases


Minimum lease payments receivables (Note 10.1) 1,720,000
Add: Residual value of leased assets 100,000
Gross Investments in leases 1,820,000
Less: Unearned lease income (Rs. 580,000 - Rs. 272,941 - 3) (307,062)
Net investment in leases (Note 10.2) Note 10.2 1,512,938
Less: Current portion of net investment in leases (663,360)
849,578

10.1 Minimum lease payments


Less than one year 860,000
More than one year and less than 5 years 860,000
1,720,000

10.2 Net investment in leases


Less than one year 663,360
More than one year and less than 5 years 849,578
1,512,938

Emile Woolf International 201 The Institute of Chartered Accountants of Pakistan


Financial accounting and reporting II

9.8 NEPTUNE LIMITED


(a) Journal entries
(i) Finance Lease:
Debit Credit
Date Particulars
Rupees
1-Jan-2015 Finance lease debtors 12,000,000
Unearned finance lease
income 3,295,690
Sale 8,704,310
(Record sale of vehicles on
finance lease)
1-Jan-2015 Bank 2,000,000
Finance lease debtors 2,000,000
(Instalment received under
finance lease)
Unearned finance lease
31-Dec-2015 income 1,005,647
Finance lease income 1,005,647
(Interest income earned at
15%)

(ii) Operating lease:


1-Jan-2015 Bank 4,000,000
Unearned rental income 4,000,000
(Operating lease instalment
received in advance)
31-Dec-2015 Unearned rental income 3,803,333
Rental income
(11,410,0003)(W2) 3,803,333
(Booking of operating lease
income)
Depreciation expenses
31-Dec-2015 (15,000,0006) 2,500,000
Accumulated
depreciation on machine. 2,500,000
(Yearly depreciation on
machine)

Reason for choice of leases:

1. Lease A should be accounted for as a finance lease because the


lease term covers the entire economic life.
2. Since none of the conditions specified in IAS-17 (Leases) for
classification as a finance lease is being met, Lease B shall be
considered as an operating lease.

Emile Woolf International 202 The Institute of Chartered Accountants of Pakistan


Answers

W1 Finance lease:
Recovery
Opening Income Closing
Year Instalment of
Balance at 15% balance
Principal
Rs. Rs. Rs. Rs. Rs.
2015 8,704,310 2,000,000 1,005,647 994,354 7,709,957
2016 (A) 7,709,957 2,000,000 856,493 1,143,507 6,566,450
2017 6,566,450 2,000,000 684,967 1,315,033 5,251,417
2018 5,251,417 2,000,000 487,713 1,512,287 3,739,130
2019 3,739,130 2,000,000 260,870 1,739,130 2,000,000
2020 2,000,000 2,000,000 0 2,000,000 0
(B) 8,000,000 1,433,550 6,566,450
(A)+(B) 10,000,000 2,290,043 7,709,957

W2 Operating lease:
Rs.
Annual instalment 2015 4,000,000
2016 (4,000,000 95%) 3,800,000
2017 (3,800,000 95%) 3,610,000
11,410,000
(b) Neptune Limited
Notes to the Financial Statements
For the year ended December 31, 2015

(i) Investment in finance lease


2015
Rs.
Present value of minimum lease payments 7,709,957
Less: current maturity (1,143,507)
6,566,450

Gross Net
investment in investment
finance leases in leases
2015 2015
Rs. Rs.
Less than one year 2,000,000 1,143,507
One to five years 8,000,000 6,566,450
10,000,000 7,709,957
Less: unearned finance income (2,290,043)
Net investment in leases 7,709,957

The minimum lease payment has been discounted on interest rate of


15% per annum to arrive at their present value. Rentals are paid in
annual instalments.
(ii) Operating lease
Not later
One to
than one Total
five years
year
Rs. Rs. Rs.
Future minimum lease
payments (W2) 3,800,000 3,610,000 7,410,000

Emile Woolf International 203 The Institute of Chartered Accountants of Pakistan


Financial accounting and reporting II

9.9 QUARTZ AUTO LIMITED

(a) Entries to record the lease in books of Quartz Auto Limited


Description Debit Credit
Lease receivable (2,715,224 5) + 700,000 14,276,120
Cost of goods sold [(900,000 7) - (100,000 7
0.49718)] 5,951,974
Inventory (900,000 x 7) 6,300,000
Sales (Note 1) 9,101,974
Unearned finance income 4,826,120

Bank 2,715,224
Lease receivable 2,715,224

Unearned finance income 1,417,500


Finance income 1,417,500

Note 1 Lower of fair value i.e. 9,450,000 (Rs. 1,350,000 x 7) and


PV of minimum lease payment (2,715,227 x 3.35219 = 9,101,974)

(b) Disclosure in the financial statements

1- Net investment in lease 2015


Rs.
Lease receivable (2,715,227 x 4) 10,860,896
Unguaranteed residual amount 700,000
Gross investment in lease 11,560,896

Less: Unearned finance income (4,826,120 1,417,500) (3,408,620)


8,152,276

1.1 Details of investment in finance lease


Gross Net
investment investment
in lease in lease
Not later than one year 2,715,224 1,492,383
Later than one year but not later than five years 8,845,672 6,659,893
Later than five years - -
11,560,896 8,152,276

(W1)
Net Gross
Year Instalment
Interest Principal Investment Investment
ended at year end
in Lease in Lease
9,450,000 14,276,120
31/06/2015 2,715,224 1,417,500 1,297,724 8,152,276 11,560,896
31/06/2016 2,715,224 1,222,841 1,492,383 6,659,893 8,845,672
31/06/2017 2,715,224 998,984 1,716,240 4,943,653 6,130,448
31/06/2018 2,715,224 741,548 1,973,676 2,969,977 3,415,224
31/06/2019 2,715,224 445,247 2,269,977 700,000 700,000

Emile Woolf International 204 The Institute of Chartered Accountants of Pakistan


Answers

9.10 LODHI TEXTILE MILLS LIMITED


Particulars Debit Credit
Generator A
(i) Cash / Bank 6,000,000
Accumulated depreciation Generator 2,500,000
*1,500,00
Loss on sale/ Impairment loss 0
Property, plant and equipment - Generator 10,000,000
*(This amount comprises of impairment loss amounted to
Rs. 1 million and loss on disposal amounted to Rs. 0.5
million.)

(ii) Assets subject to finance lease - Generator 6,000,000


Liabilities against assets subject to finance lease 6,000,000

Generator B
(i) Cash / Bank 6,000,000
Accumulated depreciation Generator 6,000,000
Property, plant and equipment - Generator 12,000,000

(ii) Assets subject to finance lease - Generator 6,000,000


Liabilities against assets subject to finance lease 6,000,000

(iii) Impairment loss 1,000,000


Accumulated impairment (ASFL) - Generator 1,000,000

Generator C
(i) Cash / Bank 8,000,000
Accumulated depreciation Generator 3,000,000
10,000,00
Property, plant and equipment - Generator 0
Deferred income OR Surplus on revaluation of
fixed assets 1,000,000

(ii) Assets subject to finance lease - Generator 8,000,000


Liabilities against assets subject to finance lease 8,000,000

Emile Woolf International 205 The Institute of Chartered Accountants of Pakistan


Financial accounting and reporting II

9.11 NOMAN ENGINEERING LIMITED


Journal entries
Debit Credit
Date Description
Rs.000 Rs.000
1-Jul-2014 Bank 20,000
Accumulated depreciation (18,750-15,000) 3,750
Property, plant and equipment 18,750
Deferred gain on disposal (20,000-15,000) 5,000
(Disposal of plant under sale and finance
lease back)

1-Jul-2014 Property, plant and equipment 20,000


Long term finance lease liability 20,000
(Plant acquired under sale and lease back)

31-Dec-2014 Long term finance lease liability W.1 1,127


Interest expense W.1 1,373
Bank 2,500
(Payment of 1st. Instalment of lease
liability)

30-Jun-2015 Long term finance lease liability 1,204


Interest expense 1,296
Bank 2,500
(Payment of 2nd. Instalment of lease
liability)

30-Jun-2015 Deferred gain on disposal (5,000/6) 833


Gain on disposal 833
(Deferred gain on amortised over the life of
the plant)

30-Jun-2015 Depreciation expense (20,000/6) 3,333


Accumulated depreciation 3,333
(Depreciation for the year for plant)
Note: If there is no reasonable certainty that the lessee will obtain ownership by
the end of the lease term, the asset shall be fully depreciated over the shorter
of the lease term and its useful life.

W1: Interest
Instalment Principal
Liability against finance lease at
payments balance
13.731%
Balance 1-Jul-2014 20,000
Payments made on 31-Dec-2014 2,500 1,373 (1,127)
30-Jun-2015 2,500 1,296 (1,204)
5,000 2,669 (2,331)
Balance 30-6-2015 17,669

Emile Woolf International 206 The Institute of Chartered Accountants of Pakistan


Answers

CHAPTER 10 IAS 37: PROVISIONS CONTINGENT LIABILITIES AND


CONTINGENT ASSETS AND IAS 10: EVENTS OCCURRING AFTER THE
REPORTING DATE

10.1 BADAR

Decommissioning costs
IAS 37 Provisions, Contingent Liabilities and Contingent Assets only permits a
provision to be made if three conditions are met:
(i) The company has a present obligation, either legally or constructively, as a
result of a past event;
(ii) Probable outflow of resources is required to settle the obligation; and
(iii) A reliable estimate is available.
Although there is no legal requirement to restore the site, the company has
established a constructive obligation by setting a valid expectation in the market,
due to its published policies and past practice, from which it cannot realistically
withdraw.
It therefore appears probable that Badar will have to pay money to improve the site
and so a provision should be created for the expected amount. As the expected
payment of Rs.100,000 will not be settled for three years, the provision should be
discounted and entered at its net present value of Rs.75,131 (Rs.100,000/(1.1)3).
Over the three years, the discounting should be unwound and charged to profit or
loss as finance costs, resulting in a provision of Rs.100,000 by the end of the third
year.
The cost of the construction work has been correctly capitalised. The cost of the
future decommissioning work should be added to this asset so that the total costs of
the site can be matched to the revenue from the copper over the period of mining.
This will result in an asset of Rs.575,131 which should be depreciated over the three
year life in line with anticipated revenues.

10.2 GEORGINA
(1) Litigation for damages
Under IAS37, a provision should only be recognised when:
an entity has a present obligation as a result of a past event
it is probable that an outflow of economic benefits will be required to
settle the obligation
a reliable estimate can be made of the amount of the obligation.
Applying this to the facts given:
Georginas legal advisors have confirmed that there is a legal obligation.
This arose from the past event of the sale, on 1 September 2015 (i.e.
before the year end).
Probable is defined as more likely than not. The legal advisors have
confirmed that it is likely that the claim will succeed.
A reliable estimate of Rs.500,000 has been made.

Emile Woolf International 207 The Institute of Chartered Accountants of Pakistan


Financial accounting and reporting II

Therefore a provision of Rs.500,000 should be made.


Counter-claim
IAS37 requires that such a reimbursement should only be recognised where
receipt is virtually certain. Since the legal advisors are unsure whether this
claim will succeed no asset should be recognised in respect of this claim.

(2) Claim for unfair dismissal


In this case, the legal advisers believe that success is unlikely (i.e. possible
rather than probable). Therefore this claim meets the IAS37 definition of a
contingent liability:
a possible obligation
arising from past events
whose existence will be confirmed only by the occurrence or non-
occurrence of one or more uncertain future events.
The liability is a possible one, which will be determined by a future court case
or tribunal. It did arise from past events (the dismissal had taken place by the
year end).
This contingent liability should be disclosed in the financial statements (unless
the legal advisors believe that the possibility of success is in fact remote, and
then no disclosure is necessary).

(3) Returns
Applying the IAS37 conditions in (1) to the facts given:
Although there is no legal obligation, a constructive obligation arises
from Georginas past actions. Georgina has created an expectation in its
customers that such refunds will be given.
As at the year end, based on past experience, an outflow of economic
benefits is probable.
A reliable estimate can be made. This could be 1% 400,000 but since
the returns are now all in the actual figure of Rs.3,500 can be used.
Therefore a provision of Rs.3,500 should be made.

(4) Closure of division


Applying the above IAS37 conditions in (1) to the facts given:
A present obligation exists because at the year end there is a detailed
plan in place and the closure has been announced in the press.
An outflow of economic benefits is probable.
A reliable estimate of Rs.300,000 has been made.
However, IAS37 specifically states in respect of restructuring that any
provision should include only direct expenses, not ongoing expenses such as
staff relocation or retraining. Therefore a provision of Rs.250,000 (300,000
50,000) should be made.

Emile Woolf International 208 The Institute of Chartered Accountants of Pakistan


Answers

10.3 EARLEY INC


(a) IAS 10 (revised) Events After the Statement of financial position Date states
that assets and liabilities should be adjusted for events occurring after the
statement of financial position date that provide additional evidence relating to
conditions existing at the statement of financial position date. It specifically
includes the example of bad debts, where evidence of bankruptcy of a debtor
occurs after the year end.
In this case, Nedengy appears to have recovered part of the debt and as such
only Rs.200,000 needs to be provided. It may be argued that the receivership
has occurred as a result of events occurring after the statement of financial
position date, as a result of a change in legislation for example, but this is
unlikely.
IAS 18 Revenue states that when uncertainty arises about the collectability of
an amount already included in revenue, the amount should be recognised as
an expense.
(b) It is likely that the fall in the value of the property will fit the IAS 10 (revised)
definition of adjusting events noted in (a) above, unless, again, it can be
argued that the decline in the property market occurred after the year-end.
IAS 36 Impairment of assets and IAS 16 Property, Plant and Equipment
require that the carrying amount of property, plant and equipment should be
reviewed periodically in order to assess whether the recoverable amount has
fallen below the carrying amount. Where it has, the property, plant and
equipment should be written down to the recoverable amount, either through
the statement of profit or loss as an expense, or though other comprehensive
income to revaluation reserve in shareholders equity, but only to the extent
that the balance on the revaluation reserve relates to a previous revaluation
surplus on the same asset.
(c) IAS 2 Inventories requires that inventories be stated at the lower on cost and
net realisable value. Net realisable value is the estimated selling price in the
ordinary course of business less the estimated costs of completion and the
estimated costs necessary to make the sale.
Unless Earley was making a significant margin on the tricycles, it is likely that
the reduction in selling price of 30% will necessitate a write- down to net
realisable value, especially considering the transportation costs to Iraq which
must be included. If the Iraqi option is unlikely to proceed, it may be necessary
to write the tricycles down to scrap value.
(d) Under IAS 10, the nationalisation is likely to be regarded as a non-adjusting
event that merely requires disclosure in the financial statements. IAS 27
Consolidated Financial Statements and Accounting for Investments in
Subsidiaries, requires that an investment in a enterprise should be accounted
for as an investment (under IAS 39: Financial Instruments: Recognition and
Measurement) from the date that it ceases to fall within the definition of a
subsidiary and does not become an associate. It seems here that Earley has
neither control nor significant influence, nor even an investment as the assets
have been in fact, expropriated. The loss of the investment should be
accounted for in the year in which it occurred, but disclosed in the current
year.
If the loss of the subsidiary results in Earley no longer being a going concern,
then the event becomes an adjusting event.
(e) & (f) Both of the events described are non-adjusting event which should be
disclosed, but not adjusted for in the current year financial statements.

Emile Woolf International 209 The Institute of Chartered Accountants of Pakistan


Financial accounting and reporting II

10.4 ACCOUNTING TREATMENTS


(a) IAS 37 Provisions contingent liabilities and contingent assets states that
contingent gains should not be recognised as income in the financial
statements. The company has a debit balance already in its books which
indicates that it must be reasonably certain that at least part of the claim will
be paid. This element of the claim then is probably not a contingency at all.
The remaining part (the difference between the Rs.15,000 and the Rs.18,600)
is, and should be disclosed and not accrued.
(b) IAS 16 Property, Plant and Equipment requires that the carrying amount of
property, plant and equipment should be reviewed periodically in order to
assess whether the recoverable amount has fallen below the carrying amount.
Where it has, the property, plant and equipment should be written down to the
recoverable amount through the statement of profit or loss as an expense. In
this case this would result in the recognition of an expense of Rs.200,000.
(280,000 80,000).
It may be the case that the amounts involved are so significant as to warrant
separate disclosure in the statement of profit or loss under IAS 8 Net Profit of
Loss for the Period, Fundamental Errors and Changes in Accounting Policies.
(c) IAS 37 states that contingent liabilities should not be recognised. Though a
provision should be made for amounts where the company has an obligation
to pay them.
The question in this case is whether or there is an obligating event within the
context of IAS 37. On balance it seems inappropriate to recognise a provision
in respect of this amount but the possible liability should be disclosed as a
contingent liability.
(i) the nature of the contingency
(ii) the uncertainties surrounding the ultimate outcome
(iii) the likely effect, ie Rs.500,000 loss less likely tax relief.

(d) IAS 2 Inventories requires that inventories be stated at the lower on cost and
net realisable value. Net realisable value is the estimated selling price in the
ordinary course of business less the estimated costs of completion and the
estimated costs necessary to make the sale.
In this case, cost is Rs.1,800 and net realisable value is Rs.1,600
(e) The company should set up a provision for Rs.100,040, ie should accrue for
the 10% probable liability. It should disclose the possible liability under
contingent liabilities. The disclosure is as noted in (c) except that the financial
effect is Rs.300,120 (30% Rs.1,000,400). The balance should be ignored as
it is a remote contingent liability.
Tutorial note
In (c) above it is not appropriate to provide for 20%receivableRs.500,000, ie
Rs.100,000. This would only be appropriate where the event is recurring many
times over.
In (e) it is appropriate to use the percentages provided, as warranty work is provided
for.

Emile Woolf International 210 The Institute of Chartered Accountants of Pakistan


Answers

10.5 J-MART LIMITED


(a) Adjusting events:
Adjusting events are events that provide further evidence of conditions that
existed at the reporting date.
Examples of adjusting events include:
(i) The subsequent determination of the purchase price or of the proceeds
of sale of non-current assets purchased or sold before the year end.
(ii) The renegotiation of amounts owing by customers or the insolvency of
a customer
(iii) Amounts received or receivable in respect of insurance or the
insolvency of a customer.
(iv) The settlement after the reporting date of a court case that confirms
that the entity had a present obligation at the reporting date.
(v) The receipt of the information after the reporting date indicating that an
asset was impaired at the reporting date.
(vi) The discovery of fraud or errors that show that the financial statements
are incorrect.
Non-adjusting events:
Non-adjusting events are indicative of conditions that arose subsequent to
the reporting date.
Examples of non-adjusting events might be:
(i) Losses of non-current assets or inventories as a result of a
catastrophe such as fire or flood
(ii) Closing a significant part of the trading activities if this was not begun
before the year end
(iii) The value of an investment falls between the reporting date and the
accounts are authorised
(iv) Announcement of dividend after year end.
(b) (i) The conditions attached to the sale give rise to a constructive
obligation on the reporting date.
A provision for the sales return should be recognised for 5% of June
2015 sales. The related cost should also be reversed.
(ii) Since the law suit was already in progress at year-end and the
amount of compensation can also be estimated, it is an adjusting
event.
A provision of Rs. 400,000 should be made.
(iii) There is no obligating event at the year end either for the costs of
fitting the smoke detectors or for fines under the legislation.
No provision should be recognised in this regard.
(iv) The obligating event is the communication of decision to the
customers and employees, which gives rise to a constructive
obligation from that date, because it creates a valid expectation that
the division will be closed.
Since no communication has yet been made, no provision is required
in this regard.

Emile Woolf International 211 The Institute of Chartered Accountants of Pakistan


Financial accounting and reporting II

(v) The obligating event is the signing of the lease contract, which gives
rise to a legal obligation.
A provision is required for the unavoidable rent payments.
(vi) Since the declaration was announced after year-end, there is no past
event and no obligation at year-end and is therefore non-adjusting
event.
Details of the dividend declaration must, however, be disclosed.

10.6 AKBER CHEMICALS LIMITED


(a) The event is an accident, and since it happened before the year end, it is a
past event. However, there is no present obligation since:
(i) there is no law requiring the company to clean the canal.
(ii) there is no constructive obligation to clean the river since:
a public statement has not been made;
there is no established pattern of past practice as this was the first
time the company faced such a situation.
Although the company has decided to clean up the river and even has a
reliable estimate of the costs thereof, no liability or provision should be
recognised in the current year because:
the decision was taken after year end; and
the decision was not yet made public.
(b) It is a non-adjustable event because the event due to which the net realizable
value (NRV) of stock has fallen, arose after the reporting date.
However, if this event is material, the company should disclose the decline in
NRV in its financial statement for the year ended June 30, 2015.
(c) The company should make the provision because:
(i) the company has a present obligation because of past event
(ii) the claim of the customer is valid and is confirmed by the company's
inspection team which shows that an outflow will be required to settle
the obligation.
(iii) the amount of outflow is reliably estimated i.e. Rs. 2 million.
Since the company is certain of recovery from the vendor, it should:
(i) disclose it as a separate asset.
(ii) recognise a receivable but the same should not exceed the amount of
the related provision i.e. rs. 2.0 million.

Emile Woolf International 212 The Institute of Chartered Accountants of Pakistan


Answers

10.7 QALLAT INDUSTRIES LIMITED


(i) Provision must be made for estimated future claims by customers for goods
already sold.
The expected value i.e. Rs. 10 million ([Rs. 150m x 2%] + [Rs. 70m x 10%])
is the best estimate of the provision.
(ii) Warehouse A: It is an onerous contract. as the warehouse has been sublet
at a loss of Rs. 200,000 per month. QIT should therefore create a provision
for the onerous contract that arises on vacating the warehouse. This is
calculated as the excess of unavoidable costs of the contract over the
economic benefits to be received from it. Therefore, QIL should immediately
provide for the amount of Rs. 13.2 million. [5.5 years x 12 month x Rs.
200,000] in its financial statements i.e. for the year ended June 30, 2015.
Warehouse B: It is not an onerous contract because the warehouse has
been sublet at profit. Hence this would require no adjustment.
(iii) A provision is to be made by QIL against a contingent liability as:
(i) There is a present obligation (legal or constructive) as a result of a
past event; i.e. accident occurred on June 15, 2015.
(ii) It is probable that outflow of resources will be required to settle the
obligation; and
(iii) A reliable estimate can be made of the amount of the obligation.
The amount of provision shall be Rs. 2.0 million i.e. the most probable
amount as determined by the lawyer.
(iv) A provision of Rs. 0.4 million is required in relation to penalty for March 1 to
June 30, 2015 because at the reporting date there is a present obligation in
respect of a past event.
The reimbursement of penalty amount from the vendor shall be recognised
when and only when it is virtually certain that reimbursement will be received
if the entity settles the obligation. The reimbursement should be treated as a
separate asset in the statement of financial position. However, in profit and
loss statement, the expense relating to a provision may be netted off with the
amount recognised as recoverable, if any.

10.8 SKYLINE LIMITED


(i) Although the debt owing by the customer existed at the reporting date, the
customers inability to pay did not exist at that point. This condition only arose in
January 2016 after the fire.
Thus, this is a non-adjusting event. However, if it is material for the financial
statements, the following disclosure should be made.
Nature of the event
An estimate of its financial effect
(ii) The amount withdrawn before year end i.e. Rs. 1.5 million is an adjusting event as
although it was discovered after year end it existed at the year end. However,
since 60% has been recovered subsequently, Rs. 0.6 million would be provided.

Emile Woolf International 213 The Institute of Chartered Accountants of Pakistan


Financial accounting and reporting II

The further withdrawal of Rs. 6.0 million is a non-adjusting event as it occurred


after year end. However, if the events are considered material the following
disclosures should be made:
Nature of the event
The gross amount of contingency
The amount recovered subsequently
(iii) SL should not recognise the contingent gain until it is realised. However, if
recovery of damages is probable and material to the financial statements, SL
should disclose the following facts in the financial statements:
Brief description of the nature of the contingent asset
An estimate of the financial effect.
(iv) SL should make a provision of the expected amount i.e. Rs. 1.2 million (Rs. 1.0
million x 60% + Rs. 1.5 million x 40%) because
it is a present obligation as a result of past event;
it is probable that an outflow of resources embodying economic benefits will be
required to settle the obligations; and
a reliable estimate can be made of the amount.
In addition, SL should disclose the following in the notes to the financial
statements:
Brief nature of the contingent liability
The amount of contingency
An indication of the uncertainties relating to the amount or timing of any
outflow.

10.9 WALNUT LIMITED


(i) This is an adjusting post reporting event as it provides evidence of conditions that
existed at the end of the reporting period. The reasons for the competitors price
reduction will not have arisen overnight, but will normally have occurred over a
period of time, may be due to superior investment in technology.
An inventory write down of Rs. 2.5 million should be recognised and the amount
included as inventory on the Statement of Financial Position reduced to Rs. 12.5
million.
(ii) The provision should be recognised because the obligating event is the
communication of event to the public which creates a valid expectation that the
division will be closed.
However, the provision should only be recognised to the extent of redundancy
costs. IAS prohibits the recognition of future operating losses, staff training and
profits on sale of assets.
(iii) This is a non-adjusting event because the burglary and theft of consumable stores
occurred after reporting date. However, if the event is material, it should be
disclosed in the financial statements unless the loss is recoverable from the
insurance company.

Emile Woolf International 214 The Institute of Chartered Accountants of Pakistan


Answers

(iv) The drop in value of investment in shares is a non-adjusting event. Since the
legislation was announced after the reporting date, the event is not a past event.
However, if the amount is material, it should be disclosed in the financial
statements.
(v) This is an adjusting event as it provides evidence of conditions that existed at the
end of the reporting period. The insolvency of a debtor and the inability to pay
usually builds up over a period of time and it can therefore be assumed that it was
facing financial difficulty at year-end.
A bad debts expense of Rs. 1.5 million should be recognised in SOCI.
(vi) It is a non-adjusting event because the declaration was announced after the year-
end and there was no obligation at year end. Details of the bonus shares
declaration must, however, be disclosed.

10.10 ATTOCK TECHNOLOGIES LIMITED


(i) Since the event which caused the inventory to be sold at a loss occurred after the
year end, it is non-adjusting event. However, the effect of the event should be
disclosed in the financial statements for the year ended June 30, 2015.
(ii) It is an adjusting event in accordance with the requirement of IAS-10. The
debtors balance should be written down by 80% amount.
(iii) It is non-adjusting event as the subsequent reduction in price is due to an event,
introduction of competitive product, occurred after the reporting period.
(iv) Since this change was not enacted before the reporting date, it is a non-adjusting
event. However, a disclosure should be made for this change.
(v) Since the declaration was announced after the year-end and there was no
obligation at year-end it is a non-adjusting event. Details of the dividend
declaration must, however, be disclosed.

Emile Woolf International 215 The Institute of Chartered Accountants of Pakistan


Financial accounting and reporting II

CHAPTER 11 IAS 8: ACCOUNTING POLICIES, CHANGES IN ACCOUNTING


ESTIMATES AND ERRORS

11.1 WONDER LIMITED


2014
2015 (Restated)
Rs.m Rs.m

Wonder Limited
Extracts of Statement of financial position
For the year ended 30 June 2015
Property, plant and equipment 178.50 111.50
Retained earnings 158.65 95.05
Deferred tax liability 41.85 21.45
PPE: Year 2015: 189 - [20 - (20 10% 1.75)] + [56/4 56/7] PPE: Year 2014: 130 - 18.5(Note X)

DTL: Year 2015: [(21.45 + (45 - 27) + {(6+2) 30%}] DTL: Year 2014: 27 - 5.55 (Note X)

Wonder Limited
Extracts from the Statement of profit or loss for the year ended 30 June 2015
Profit before taxation 98.00 101.50
Taxation (34.40) (36.45)
Profit after taxation 63.60 65.05

PBT : Year 2015 : 90 + (20 10% ) + [(56/4) - (56/7)] PBT : Year 2014 : 120 - 18.5 (Note X)

Tax : Year 2015: 32 + [(6+2) 30%] Tax : Year 2014 : 42 - 5.55 (Note X)

Wonder Limited
Extracts of statement of changes in equity for the year ended 30 June 2015
Retained
earnings
Rs.m

Balance as on 1 July 2013 (108-78) 30.00


Profit for the year ended 30 June 2014 (78 - 12.95 (Note X))-
restated 65.05
Balance as at 30 June 2014 - restated 95.05
Profit for the year ended 30 June 2015 63.60

Balance as at 30-June 2015 158.65

Emile Woolf International 216 The Institute of Chartered Accountants of Pakistan


Answers

Wonder Limited
Notes to the financial statements
For the year ended 31 December 2015
X Correction of error
During the year ended 30 June 2013, the repair works was erroneously
debited to machinery account. The effect of this error is as follows:

2014
Rs.m
Effect on the statement of profit or loss
(Increase) / decrease in expenses or losses
Repairs and maintenance (20.00)
Depreciation (20 10% 9 12) 1.50
Tax expenses (30% (20-1.5)) 5.55
Decrease in profit for the year (12.95)

Effect on the statement of financial position


Increase / (decrease) in assets
Property, plant and equipment (20 1.5) (18.50)

(Increase) / decrease in liabilities


Deferred tax liability (Rs. 18.5 30%) 5.55

(Increase) / decrease in equity


Retained earnings (18.50 - 5.55) (12.95)

11.2 DUNCAN
Statement of changes in equity (extract)
Retained Retained
earnings earnings
2015 2014
Rs.000 Rs.000
Opening balance as reported 23,950 22,500
Change in accounting policy (W2) 450 400

Re-stated balance 24,400 22,900
Profit after tax for the period (W1) 4,442 3,250
Dividends paid (2,500) (1,750)

Closing balance 26,342 24,400

Workings

(1) Revised profit


2015 2014
Rs.000 Rs.000
Per question 4,712 3,200
Add back: Expenditure for the year 600 500
Minus: Depreciation (870) (450)

Revised profit 4,442 3,250

Emile Woolf International 217 The Institute of Chartered Accountants of Pakistan


Financial accounting and reporting II

(2) Prior period adjustment


The prior period adjustment is the reinstatement of the Rs.400,000 asset on 1
January 2014 and the Rs.450,000 asset at 1 January 2015. On 31 December
2015 the closing balance above of Rs.26,342,000 can be reconciled as the
original Rs.26,162,000 plus the reinstatement of the remaining asset of
Rs.180,000.

11.3 MOHANI MANUFACTURING LIMITED


Mohani Manufacturing (Private) Limited
Statement of changes in equity
For the year ended December 31, 2015
Retained
Earnings
Rs. in
million
Balance at December 31, 2013 as previously reported (Rs. 89m
Rs. 21m) 68.00
Effect of change in accounting policy (Rs. 37m - Rs. 35.5m) (1.50)
Balance at December 31, 2013 restated 66.50
Profit for the year ended December 31, 2014 - restated (W1) 39.70
Balance at December 31, 2014 restated 106.20
Profit for the year ended December 31, 2015 (W2) 8.80
Balance at December 31, 2015 115.00
Rs. in
W1: Profit for the year ended December 31, 2014 (as restated) million
Profit as previously reported 21.00
Incorrect recording of depreciation (Rs. 25 million Rs. 10 million) 15.00

Reversal of FIFO method


Opening inventory 37.00
Closing inventory (42.30)
(5.30)
Application of weighted average method
Opening inventory (35.50)
Closing inventory 44.50
9.00
39.70

W2: Adjusted profit for year ended June 30, 2015


Profit as per draft financial statements 15.00
Adjustment in Opening Inventory
FIFO 42.30
Weighted average (44.50)
(2.20)
Adjustment in Closing Inventory
FIFO (58.40)
Weighted average 54.40
(4.00)
Adjusted profit 8.80

Emile Woolf International 218 The Institute of Chartered Accountants of Pakistan


Answers

CHAPTER 12 IAS 12: INCOME TAXES

12.1 FRANCESCA
Rs. Rs.
Opening liability 1,340,600
Capital allowances during the year 50,000,000
Depreciation charged during the year (45,000,000)

5,000,000 30% 1,500,000


Interest receivable in statement of profit or


loss 50,000
Interest received in tax computation (45,000)

Receivable in statement of financial


position 5,000 30% 1,500

Interest payable in statement of profit or


loss 32,000
Interest paid in tax computation (28,000)

Payable in balance sheet 4,000 30% (1,200)


Development costs as allowable expense 500,600 30% 150,180

Revaluation 6,000,000
Carrying value (4,900,500)

Revaluation surplus 1,099,500 x 30% 329,850


Closing liability 3,320,930


Rs.
Charged to the revaluation reserve 329,850
Charged in the statement of profit or loss (balancing figure) 1,650,480

Total movement on the provision of (3,320,930 1,340,600) 1,980,330

Emile Woolf International 219 The Institute of Chartered Accountants of Pakistan


Financial accounting and reporting II

12.2 SHEP (I)


(a) Corporate income tax liability - year ended 31st December 2015
Rs.
Profit per accounts 121,000
Add Depreciation 11,000

133,000
Less tax depreciation (15,000)

Taxable profits 117,000

Tax payable @ 30% 35,100



(b) Deferred tax liability
Rs.
Carrying amount (48,000 + 12,000 = 60,000 11,000) 49,000
Tax base (48,000 + 12,000 = 60,000 15,000) 45,000

Temporary difference (4,000)

Deferred tax liability required @ 30% (1,200)

(c) Movement on the deferred tax liability
Rs.
Balance b/f
Statement of profit or loss (balancing figure) 1,200

Balance c/f 1,200

(d) Statement of profit or loss note


Rs.
Current tax expense 35,100
Deferred tax expense 1,200

Tax expense 36,300

Emile Woolf International 220 The Institute of Chartered Accountants of Pakistan


Answers

12.3 SHEP (II)


(a) Corporate income tax liability - year ended 31st December 2016
Rs.
Profit per accounts 125,000
Add Depreciation 14,000
Interest payable 500
Provision 1,200
Fine 6,000

146,700
Less tax allowance (given) (16,000)
Interest receivable (150)

Taxable profits 130,550

Tax payable @ 30% 39,165

(b) Deferred tax liability


Carrying Tax Temporary
amount base difference
Rs. Rs. Rs.
Tangible assets
Carrying amount (49bf 14) 35,000
Tax base (45bf 16) 29,000 6,000
Interest payable (25,000 x 8% x 3/12) (500) (500)
Interest receivable (4,000 x 15% x 3/12) 150 150
Provision (1,200) (1,200)

33,450 29,000 4,450

Deferred tax @30% 1,335


(c) Movement on the deferred tax liability


Rs.
Balance b/f 1,200
Statement of profit or loss (balancing figure) 135

Balance c/f 1,335

(d) Statement of profit or loss note


Rs.
Current tax expense 39,165
Deferred tax expense 135

Tax expense 39,300

Emile Woolf International 221 The Institute of Chartered Accountants of Pakistan


Financial accounting and reporting II

(e) Tax reconciliation


Rs.
Accounting profit 125,000

Accounting profit @ 30% 37,500
Tax effect of the fine (6,000 @ 30%) 1,800

Tax expense 39,300

12.4 SHEP (III)


(a) Corporate income tax liability - year ended 31st December 2017
Rs.
Profit per accounts 175,000
Add Depreciation 18,500
Interest payable (note)
Provision 2,000
Entertainment 20,000

215,500
Less tax allowance (given) (24,700)
Interest receivable (note)
Development costs (17,800)
Provision (500)

Taxable profits 172,500

Tax payable @ 30% 51,750


Note
There is no adjustment to profit for the interest paid and the interest receivable.
Consider the interest payable. The tax authority will disallow the closing accrual but
will allow last years accrual (that has been paid in this year) as a deduction. These
amounts are equal so there is no net effect.
Similar comments can be made about the interest receivable.

Emile Woolf International 222 The Institute of Chartered Accountants of Pakistan


Answers

(b) Deferred tax liability


Carrying Tax Temporary
amount base difference
Rs. Rs. Rs.
Tangible assets
Carrying amount (35bf 18.5) 16,500
Tax base (29bf 24.7) 4,300 12,200
Interest payable (500) (500)
Interest receivable 150 150
Provision (2,700) (2,700)
Development expenditure 17,800 17,800

31,250 4,300 26,950

Deferred tax @ 30% 8,085

(c) Movement on the deferred tax liability


Rs.
Balance b/f 1,335
Statement of profit or loss (balancing figure) 6,750

Balance c/f 8,085

(d) Statement of profit or loss note


Rs.
Current tax expense 51,750
Deferred tax expense 6,750

Tax expense 58,500

(e) Tax reconciliation


Rs.
Accounting profit 175,000

Accounting profit @ 30% 52,500
Tax effect of the fine (20,000 @ 30%) 6,000

Tax expense 58,500

Emile Woolf International 223 The Institute of Chartered Accountants of Pakistan


Financial accounting and reporting II

12.5 SHEP (IV)


(a) Corporate income tax liability - year ended 31st December 2017
Rs.
Taxable profits (as before) 172,500

Tax payable @ 34% 58,650

(b) Deferred tax liability


Rs.
Temporary difference (as before) 26,950

Deferred tax @ 34% 9,163

(c) Movement on the deferred tax liability


Rs.
Balance b/f 1,335
Adjustment due to change in rate 178

Opening balance restated to 34% (1,335 x 34/30) 1,513
Statement of profit or loss (balancing figure) 7,650

Balance c/f 9,163

(d) Statement of profit or loss note


Rs.
Current tax expense 58,650
Deferred tax expense relating to origination and reversal
of temporary differences 7,650
Deferred tax expense resulting from increase in tax rate 178

Tax expense 66,478

(e) Tax reconciliation


Rs.
Accounting profit 175,000

Accounting profit @ 34% 59,500
Tax effect of the fine (20,000 @ 34%) 6,800
Increase in opening deferred tax balances due to
change in rate 178

Tax expense 66,478

Emile Woolf International 224 The Institute of Chartered Accountants of Pakistan


Answers

12.6 WAQAR LIMITED


a) Computation of current period income tax liability
2015 2014
Rs.m Rs.m
Accounting profit before tax 40.00 30.00

Less: Admissible deductions


Capital Gain (10.00) (8.00)
Tax depreciation on furniture and fittings
Rs. 40.5 x 10% (4.05)
Rs. 40.5 (1-10%) x 10% (3.65)
Tax depreciation on Machinery
Rs. 90 x 10% (9.00)
Rs. 90 (1-10%) x 10% (8.10)

Add: Inadmissible deductions


Accounting depreciation on machinery 25.00 25.00
Accounting depreciation on furniture and fittings 5.00 5.00
Taxable profit 48.25 38.95
Tax rate 30% 35%
Tax payable (current tax) 14.48 13.63

b) Deferred taxation computation


Tax base Temporary Deferred tax
NBV (W1)
(W1) difference liability
Working 2 Rs.m Rs.m Rs.m Rs.m
At December
31,2013
Machinery 175.00 90.00 85.00 29.75
Furniture and fittings 40.00 40.50 (0.50) (0.18)
Deferred tax liability
at December
31,2013 (35%) 29.57

At December 31,
2014
Machinery 150.00 81.00 69.00 24.15
Furniture and fittings 35.00 36.45 (1.45) (0.51)
Deferred tax liability
at December
31,2014 (35%) 23.64

WDV as at
December 31, 2015
Machinery 125.00 72.90 52.10 15.63
Furniture and fittings 30.00 32.80 (2.80) (0.84)
Deferred tax liability
at December
31,2015 (35%) 14.79

Emile Woolf International 225 The Institute of Chartered Accountants of Pakistan


Financial accounting and reporting II

Working 1
Carrying amount and tax base of machinery NBV Tax base
Cost b/f 200.0 200.0
Accumulated depreciation b/f (25.0)
At 31 December 2013 175.0 90.0
Accounting depreciation (200/8 years) (25.0)
Tax depreciation (10% of WDV) (9.0)
At 31 December 2014 150.0 81.0
Accounting depreciation (200/8 years) (25.0)
Tax depreciation (10% of WDV) (8.1)
At 31 December 2015 125.0 72.9

Carrying amount and tax base of furniture and fittings NBV Tax base
Cost b/f 50.0 50.0
Accumulated depreciation b/f (10.0)
At 31 December 2013 40.0 40.5
Accounting depreciation (10% 50) (5.0)
Tax depreciation (10% of WDV) (4.05)
At 31 December 2014 35.0 36.45
Accounting depreciation (10% 50) (5.0)
Tax depreciation (10% of WDV) (3.65)
At 31 December 2015 30.0 32.8

c) Movement on deferred taxation account (W2) 2015 2014


At January 1 23.64 29.57
Change due to change in rate (23.64 5/35) (3.38) -
20.26
Change due to origination and reversal of temporary
differences in the period (balancing figure) (5.47) (5.93)
At December 31 14.79 23.64

d) Tax expense 2015 2014


Current tax 14.48 13.63
Deferred tax: -
Due to origination and reversal of temporary
differences in the period (3.38)
Due to change in rate (5.47) (5.93)
Tax expense 5.63 7.7

e) Tax reconciliation 2015 2014


Accounting profit 40.0 30.0
Tax rate 30% 35%
12.0 10.5
Tax effect of untaxed gain:
30% 10.0 (3.0)
35% 8.0 (2.8)
Decrease in opening deferred tax balances due to
change in rate (with rounding adjustment) (3.37)
Tax expense 5.63 7.7

Emile Woolf International 226 The Institute of Chartered Accountants of Pakistan


Answers

12.7 SHAKIR INDUSTRIES


COMPUTATION OF TAX EXPENSE
FOR THE YEAR ENDED DECEMBER 31, 2015
2015
Rs. in
million
Profit before tax 15.80
Add: Inadmissible expenses
Accounting depreciation (Rs. 1.1 million + Rs. 0.7 million) 1.80
Financial charges on finance lease 0.15
Penalty paid to SECP 0.70
Provision for gratuity 2.40
5.05

Less: Admissible expenses Rs.m


Tax depreciation 1.65
Lease payments 0.65
Payment of gratuity 1.60
Borrowing cost capitalised 2.30
6.20
Taxable profit for the year 14.65
Current tax expense @ 35% 5.13

COMPUTATION OF DEFERRED TAX EXPENSE


FOR THE YEAR ENDED DECEMBER 31, 2015
Carrying Tax Temp
amount base difference
Rs.m Rs.m Rs.m
Fixed assets Owned 16.70 13.85 2.85
Fixed assets Leased 1.80 - 1.80
Capital work in progress 2.30 - 2.30
Provision for gratuity (0.7 + 2.4 1.6) (1.50) - (1.50)
Obligation against assets subject to finance
lease (1.20) - (1.20)
Total 4.25
Deferred tax expense @ 35% 1.49

Rs. in
million
Deferred tax liability (Opening) 0.55
Deferred tax expense for the year (balancing figure) 0.94
Deferred tax liability as at December 31, 2015 (Rs. 4.25 million x
35%) 1.49

Emile Woolf International 227 The Institute of Chartered Accountants of Pakistan


Financial accounting and reporting II

12.8 MARS LIMITED


(a) Date Particulars Debit Credit
Rupees
01.07.2014 Motor Vehicle - Cost 1,600,000
Obligations under finance lease 1,600,000
Capitalisation of the lease

01.07.2014 Obligations under finance lease 480,000


Bank 480,000
First lease payment made in advance

30.06.2015 Finance charges 153,451


Accrued finance charges 153,451

Finance charge accrual for the year ended June 30, 2015
Working: (Rs. 1,600,000 480,000) 13.701% = Rs. 153,451)

30.06.2015 Depreciation 400,000


Accumulated depreciation - Motor
Vehicle 400,000

Depreciation charge for the year ended June 30, 2015


Working: Rs. 1,600,000 4 = Rs. 400,000.
Assuming that there is no reasonable certainty about transfer of
ownership at the end of lease term.

30.06.2015 Tax expense (W1) 1,492,035


Tax payable 1,492,035
Recognition of tax expense for the year ended June 30, 2015)

30.06.2015 Tax expense 22,035


Deferred tax (W2) 22,035
Recognition of deferred tax asset.

W1 Tax computation
Rs.
Accounting profit before tax 4,900,000
Add: Depreciation on leased assets 400,000
Add: Finance charges 153,451
Less: Lease payment (480,000)
Taxable profit 4,973,451
Tax @ 30% 1,492,035

Emile Woolf International 228 The Institute of Chartered Accountants of Pakistan


Answers

W2 Deferred tax computation


Carrying Tax
Difference
amount base
Taxable temporary difference
Leased assets 1,200,000 - 1,200,000
Deductible temporary difference
Obligations under finance
-
lease (1,120,000) (1,120,000)
Accrued finance charges (153,451) (153,451)
Net taxable temporary
difference (73,451)
Deferred tax @ 30% (Asset) 22,035

(b) Liabilities against assets subject to finance lease (W3)


2015
Rs.
Present value of minimum lease payments 1,120,000
Less: Current maturity shown under current liabilities (326,549)
793,451

Minimum lease payments (W3)


Not later than 1 year 480,000
Later than 1 year and not later than 5 years (480,000 2) 960,000
1,440,000
Less: future finance charges on finance lease (320,000)
1,120,000
Present value of finance lease liabilities (W3)
Not later than 1 year 326,549
Later than 1 year and not later than 5 years
(371,289 + 422,162) 793,451
1,120,000

The minimum lease payment has been discounted at an interest rate of


13.701% to arrive at their present value. Rentals are paid in annual
instalments.

W3: Repayment Schedule


Opening Principal Interest Annual Closing
Years Balance repayment 13.701% payment Balance
Rs.m Rs.m Rs.m Rs.m Rs.m
2015 1,600,000 480,000 480,000 1,120,000
2016 1,120,000 326,549 153,451 480,000 793,451
2017 793,451 371,289 108,711 480,000 422,162
2018 422,162 422,162 57,838 480,000 -
320,000

Emile Woolf International 229 The Institute of Chartered Accountants of Pakistan


Financial accounting and reporting II

12.9 BILAL ENGINEERING LIMITED


(a) Computation of current taxation
Rs.m Rs.m
Profit before tax 50.000
Add: Accounting depreciation 10.000
Financial charges on lease liability (1.00
0.3) 13.701% 0.096
Amortization of research and development
cost for the year 1.000
Less
: Tax depreciation (7.000)
Annual instalment of lease payment (0.300)
Amortization of research and development
cost (15 0.9/10) (1.350)
Current year taxable income 52.446
Tax liability for the year (52.446 35%) 18.356
Tax liability for prior periods (0.100 35%) 0.035
18.391
Deferred taxation
Accounting depreciation 10.000
Tax depreciation (7.000) 3.000

Financial charges on finance lease liability(1.00


0.3) 13.701% 0.096
Annual instalment of lease payment allowed under
tax (0.300) (0.204)

Amortization charged in accounts 1.000


Amortization cost claimed in tax (1.350) (0.350)
Excess of taxable income over accounting profit due
to time differences 2.446
Deferred tax credit at 35% (0.856)
Total tax expenses (current and deferred) 17.535
(b) Bilal Engineering Limited: Notes to the financial statements
for the year ended December 31, 2015

1.1 Relationship between tax expense and accounting


profit 2015
Rs.m
Accounting profit before tax 50.000
Tax on accounting profit at 35% 17.500
Tax on expenses disallowed (Permanent Difference) 0.035
Effective tax rate/tax charge 17.535

(c) Journal entries Debit Credit


Rs.m Rs.m
1 Income tax expenses 18.391
Provision for taxation 18.391
(Tax provision for 2015)
2 Deferred tax asset 0.856
Tax expenses deferred 0.856
(Deferred tax credit for 2015)

Emile Woolf International 230 The Institute of Chartered Accountants of Pakistan


Answers

12.10 GALAXY INTERNATIONAL


28 : TAXATION 2015 2014
Rs.m Rs.m
Current - for the year (W 1) 0.84 -
Deferred (W 2) 6.95 (0.96)
7.79 (0.96)

28.1 : Relationship between tax expense and accounting profit


Profit/(Loss) before taxation 23.50 (1.75)

Tax at the applicable rate of 35% 8.23 (0.61)


Tax effect of exempt income (1.25 x 35%) (0.44) (0.35)
7.79 (0.96)

W1 : Computation of Current Tax


(Loss) / profit before tax as per books 23.50 (1.75)
Add: Allowable income / Disallowed expenses
Accounting depreciation 15.00 15.00
Provision for gratuity 2.20 1.70
Accrued expenses - 2.00

Less: Disallowed income / Allowable expenses


Tax depreciation (6.00) (45.00)
Interest income from SIBs (Exempt) (1.25) (1.00)
Accrued expenses (2.00)
Taxable income / (loss) 31.45 (29.05)
Tax liability (@ 35% 11.01 -
Tax loss to be brought forward (29.05 x 35%) (10.17) -
Tax payable 0.84 -

W -2: Computation of Deferred Tax


Timing differences (cumulative) on account of:
Depreciation (2015: 30-51, 2014: 15-45) 21.00 30.00
Accrued expenses - (2.00)
Provision for gratuity (3.90) (1.70 )
Tax losses - (29.05)
17.10 (2.75)

Deferred tax @ 35% 5.99 (0.96)


Add: Opening deferred tax (dr.) 0.96 -
Charge/(Reversal) for the year 6.95 (0.96)

Emile Woolf International 231 The Institute of Chartered Accountants of Pakistan


Financial accounting and reporting II

12.11 APRICOT LIMITED


Taxation 2015 2014
Rs.m Rs.m
Current (W1) 20.48 10.76
Deferred (W2) (1.58) (21.35)
18.90 (10.59)

Relationship between tax expense and accounting profit 2015


Profit before taxation 60.00
Tax at the applicable rate of 35% 21.00
Less: Tax effect of exempt income (2.10)
18.90

W1: Computation of Current Tax


Profit before tax as per books 60.00 45.00
Add: Allowable income / Disallowed expenses
Accounting depreciation 10.00 9.00
Tax profit on sale of fixed assets 1.00 -
Bad debt expense 5.00 7.00

Less: Disallowed income / Allowable expenses


Tax depreciation (8.00) (7.00)
Accounting profit on sale of fixed assets (0.50) -
Capital gain (6.00) -
Bad debts written off (3.00) (4.00)
Taxable income 58.50 50.00
Tax liability (@ 35%) 20.48 17.50

2015 2014
Rs.m Rs.m
W2: Computation of Deferred Tax
Fixed assets (2014: 95-90, 2015: 82.5-80) (W2.1) 0.87 1.75
Provision for bad debts (2014: 1235%, 2015: 1435%)
[W2.2] (4.90) (4.20)
Closing balance of deferred tax (4.03) (2.45)
Less: Opening balance (2.45) (18.90)
Charge for the year (1.58) (21.35)
W2.1 Movement of Fixed Assets Accounting Tax
Opening balance 95.00 90.00
Disposal during the year (2.50) (2.00)
Depreciation for the year - 2015 (10.00) (8.00)
Closing balance 82.50 80.00

W2.2 Movement of provision for bad debts 2015 2014


Opening balance 12.00 9.00
Provision for the year 5.00 7.00
Write off during the year (3.00) (4.00)
Closing balance 14.00 12.00

Emile Woolf International 232 The Institute of Chartered Accountants of Pakistan


Answers

CHAPTER 13 RATIO ANALYSIS

13.1 WASIM
Ratios
Year 7 Year 6
Gross profit % =
Gross prof it 405 362
x 100 x 100 = 19% x 100 = 20%
Sales 2,160 1,806
Net profit % =
Net prof it 9 53
x 100 x 100 = 0.4% x 100 = 2.9%
Sales 2,160 1,806
Return on capital employed =
Prof it bef ore interest and tax
Share capital and reserv es+ Long - term debt capital

15 56
x 100 = 6% x 100 = 29%
246 190
Asset turnover =
Sales
x 100
Share capital and reserv es+ Long - term debt capital

2,160 1,806
= 8.8 times = 9.5 times
246 190
Current ratio =
Current assets 422 265
= 1.7 times = 1.8 times
Current liabilities 254 147
Quick ratio =
Current assets excluding inv entory 422 - 106 265 - 61
1.2 times 1.4 times
Current liabilities 254 147
Average time to collect =
Trade receiv ables 316 x 365 198 x 365
x 365 53 day s 40 day s
Sales 2,160 1,806
Average time to pay =
Trade pay ables 198 x 365 142 x 365
x 365 = 41 day s = 36 day s
Cost of purchases 1,755 1, 444
Inventory turnover =
Inv entory 106 x 365 61 x 365
x 365 22 day s = 15 day s
Cost of sales 1,755 1, 444

13.2 AMIR AND MO
Amir Mo
Gross profit % =
Gross prof it 90,000 490,000
x 100 x 100 = 60% x 100 = 70%
Sales 150,000 700,000

Net profit % =

Net prof it
x 100
44,895
x 100 = 30%
270,830
x 100 = 39%
Sales 150,000 700,000

Emile Woolf International 233 The Institute of Chartered Accountants of Pakistan


Financial accounting and reporting II

Return on capital employed =

Prof it bef ore interest and tax


Share capital and reserv es+ Long - term debt capital
Amir 61,500 + 500
x 100 = 28.5%
207, 395 +10,000

Mo 371,000 +12,000
x 100 = 47%
565,580 + 250,000

Asset turnover =

Sales
x 100
Share capital and reserv es+ Long - term debt capital

Amir 150,000
= 0.7 times
207, 395 +10,000

Mo 700,000
= 0.85 times
565,580 + 250,000

Amir Mo

ratio =
Current

Current assets 50,000 153,250


= 2.2 times = 1.3 times
Current liabilities 22,605 117,670

Quick ratio =


Current assets excluding inv entory 50,000 - 12,000
= 1.7 times
153,250 - 26,250
= 1.1 times
Current liabilities 22,605 117,670

Average time to collect = 37,500 105,000


x 365 = 91 day s x 365 = 55 day s
Trade receiv ables 150,000
700,000
x 365
Sales

Average time to pay =



Trade pay ables
x 365
22,605
x 365 = 137 day s
117,670
x 365 = 204 day s
Cost of purchases 60,000 210,000

Inventory turnover =
Inv entory 12,000 26,250
x 365 x 365 = 73 day s x 365 = 46 day s
Cost of sales 60,000 210,000

Emile Woolf International 234 The Institute of Chartered Accountants of Pakistan


Answers

CHAPTER 14 ETHICAL ISSUES IN FINANCIAL REPORTING

14.1 ETHICAL ISSUES


The range of comments made by Arif raises questions over his ethical behaviour
and professional standards.
A chartered accountant should be unbiased when involved in preparing and
reviewing financial information. A chartered accountant should prepare financial
statements fairly, honestly, and in accordance with relevant professional standards
and must not be influenced by considerations of the impact of reported results.
Arifs failings
Arif appears to be influenced by the need to achieve a specified level of profit. This
is not appropriate and calls his integrity into question.
In addition Arifs professional competence seems to be suspect. His comment on
not being up to date on all of the little technicalities in IFRS s suggests that he has
not maintained a level of professional competence appropriate to his professional
role.
ICAP members have a responsibility to engage in continuing professional
development in order to ensure that their technical knowledge and professional skills
are kept up to date. Arif should seek continuing professional development activities
and improve his knowledge on ethical standards. Furthermore, it might be expected
that as Waheeds superior he should set an example to Waheed and guide him in
his responsibilities. Clearly this is not happening.
As a member of ICAP Arif should be aware of the ICAP code of ethics. Arif should
know of the danger of self-interest threats and intimidation threats to himself and to
others. His attempt to influence the outcome of a fellow professional by applying
such a threat to that individual is very unprofessional.
Waheeds ethical issues
Waheed faces a self-interest threat, in that there is the possibility of a bonus
provided the earnings per share figure remains the same as last year. Arif has also
suggested that she can influence the Boards decision over employing him as a
replacement finance director another self-interest threat to Waheed. Both of these
threats must be ignored.
Arifs comments imply that his application of professional responsibility is lacking.
This may extend into the way in which the current financial statements have been
prepared. Waheed must be very careful (as always) to carry out the review with all
due care.
Waheed should first discuss his recommendations with Arif and remind his of his
professional responsibilities to ensure that the accounting standards are correctly
followed. If the financial statements are found to contain errors or incorrect
accounting treatment then they must be amended. If Arif refuses to amend the draft
financial statements if necessary Waheed should discuss the matter with other
board members (including non- executives and the audit committee, if possible).
Further action might include consulting with ICAP.

Emile Woolf International 235 The Institute of Chartered Accountants of Pakistan


Financial accounting and reporting II

14.2 SINDH INDUSTRIES LTD


(a) Financial reporting issues
Revenue
IAS 18 Revenue sets out the rules to be followed in recognising revenue.
The fact that the customer cannot cancel the contract is not relevant to the
recognition of revenue. Revenue from providing a service is recognised
according to the stage of completion subject to satisfying criteria set out in IAS
18. In the absence of other information the revenue in this contract should be
recognised over the life of the contract as time progresses. As the contract
was only signed just before the year end, none of the revenue can be
recognised in 2015.
The credit for the amount received should be recognised as a liability. This
represents the obligation that the company has to provide the service over the
next two years.
The fact that the customer cannot cancel the contract is not relevant to the
recognition of revenue. If Sindh Industries failed to provide the service they
would be sued for restitution. Therefore the revenue can only be recognised
as the service is provided.
New factory
Borrowing costs directly attributable to construction of an asset which
necessarily takes a substantial period to get ready for its intended use should
be capitalised as part of the cost of that asset under IAS 23 Borrowing Costs.
IAS 23 states that the capitalisation of borrowing costs should commence
when three conditions are all met for the first time: borrowing costs are being
incurred, expenditure is being incurred and activities to prepare the asset are
being undertaken. Although borrowing costs were incurred throughout the
year and expenditure was incurred from 1 February 2015 (the date the land
was purchased), construction only started on 1 June 2015. Therefore this is
the date on which capitalisation commences.
Capitalisation ceases when substantially all of the activities required to make
the asset ready for use/sale have been completed, that is on 30 September
2015. (The actual date on which the factory was brought into use is irrelevant.)
Therefore the period of capitalisation should be four months.
Where construction is financed from general borrowings, the calculation of the
amount to be capitalised should be based on the weighted average cost of
borrowings. This is:
(Rs.1,000,000 9.75%) + (Rs.1,750,000 10%) + (Rs.2,500,000 8%)/
(Rs.1,000,000 + Rs.1,750,000 + Rs.2,500,000) = 9%
Therefore the amount capitalised should be 9% Rs.4.5 million (land Rs.1.8
million plus construction costs Rs.2.7 million) 4/12 = Rs.135,000. The total
cost of the factory should be measured at Rs.4,635,000 (Rs.1.8 million plus
Rs.2.7 million, plus Rs.135,000). The amount that has been recognised in the
statement of financial position should be reduced by Rs.315,000 (Rs.450,000
Rs.135,000). Finance costs recognised in profit or loss should be increased
by Rs.315,000.
Land should not be depreciated because it has an indefinite life. Under IAS 16
Property, Plant and Equipment depreciation charges should start when the
asset becomes available for use, from 1 October 2015 in this case.

Emile Woolf International 236 The Institute of Chartered Accountants of Pakistan


Answers

Depreciation of Rs.35,000 ((Rs.2.7 million, plus (Rs.135,000 2.7/4.5) 20)


3/12) should be recognised in profit or loss for the year ended 31 December
2015 and the carrying amount of the asset reduced by the same amount to
Rs.4.6 million.
Useful life of the blast furnace
Depreciation of the blast furnace has been based on an estimated useful life
of 20 years. This is at variance with a report by a qualified expert. The asset
valuation specialist treats the furnace as being made up of two components,
the main structure and the lining, which must be replaced at regular five yearly
intervals over the life of the asset. This is the approach required by IAS 16.
The uncertainties inherent in business mean that many items in financial
statements cannot be measured with certainty, but estimates should always
be made using the most up to date and reliable information. Where estimates
have been prepared by professionals with relevant qualifications, then it is
nearly always most appropriate to use those estimates. Therefore in
accordance with the valuers report the main structure of the furnace should
be depreciated over 15 years from 1 January 2015 and the lining should be
depreciated over five years from that date.
The reassessment of the estimated lives of assets is a change in accounting
estimate, rather than a change in accounting policy (IAS 8 Accounting
Policies, Changes in Accounting Estimates and Errors). Changes in
accounting estimate should be dealt with on a prospective basis. This is
achieved by including the effect of the change in profit or loss in current and
future periods. The additional depreciation should be calculated as:
Rs.000

Revised depreciation: main structure 140


((Rs.3.5m Rs.1.4m)/15 years)
lining (Rs.1.4m/5 years) 280

420

Current depreciation (Rs.3.5m/20 years) (175)

Additional depreciation 245

IAS 8 requires the disclosure of the nature and amount of the effect of the
change in the estimate of useful lives on the profit for the year.

(b) Revised financial statements


Statement of profit or loss extract for the year ended 31 December 2015
Borrowing Blast
Draft Revenue costs furnace Revised
Rs.000 Rs.000 Rs.000 Rs.000 Rs.000
Profit before tax 2,500 (1,000) (315)+ (35) (245) 905

Emile Woolf International 237 The Institute of Chartered Accountants of Pakistan


Financial accounting and reporting II

Statement of financial position at 31 December 2015


Borrowing Blast
Draft Revenue costs furnace Revised
Rs.000 Rs.000 Rs.000 Rs.000 Rs.000
Non-current assets
Property, plant and
equipment 12,000 (315) + (35) (245) 11,405
Current assets 3,500 3,500
Total assets 15,500 14,905
Share capital 2,000 2,000
Retained earnings 6,000 (1,000) (315) + (35) (245) 4,405
Equity 8,000 6,405
Non-current liabilities 5,000 500 5,500
Current liabilities 2,500 500 3,000
Total equity and liabilities 15,500 15,905
(c) Ethical issues
It is noticeable that all the adjustments required reduce profit. This and the
background to the previous finance directors resignation suggest serious
problems.
It is not clear who actually prepared the draft financial statements. If they were
prepared by more junior staff in the absence of a finance director, some of the
adjustments (for example, the calculation of borrowing costs to be capitalised)
could be the result of genuine errors or lack of accounting knowledge.
However, it seems reasonably clear that the managing director has attempted
to influence the treatment of the revenue and the estimated useful life of at
least one significant non-current asset. (Note: the directors have reviewed the
useful lives of several items of plant and machinery and it is possible that
other assets besides the furnace are being depreciated over unrealistically
long periods.)
It seems almost certain that the previous finance director resigned as a result
of pressure from the managing director (and possibly from other members of
the Board) to present the financial statements in a favourable light. The
directors intend to seek a stock market listing in the near future. Therefore
they have clear motives for manipulating the profit figure and also (perhaps)
for making controversial decisions before the financial statements come under
much greater scrutiny as a result of the listing. The job title of financial
controller is also significant. It suggests that the role has been downgraded
and that the person holding it has less authority than the rest of the Board.
Possible courses of action:
Discuss with the managing director the financial reporting standards that
apply to the transactions and explain the implications of non-compliance.
If the managing director is himself a member of a professional body then
it might be worth pointing out to him that he himself is bound by an
ethical code.
Advise him that as a Chartered Accountant you are bound by the ICAP
code of ethics, and that you would not be prepared to compromise your
views of the figures he has prepared for career advancement.
Consider speaking to the other directors (or audit committee if there is
one) and seeking their support.
If all of these actions produce a negative response then it would be
appropriate to consult the ICAP ethical handbook and/or the Institute.
If all else fails then consider seeking alternative employment.

Emile Woolf International 238 The Institute of Chartered Accountants of Pakistan


Head Oce-Karachi: Chartered Accountants Avenue, Clifton, Karachi-75600
Phone: (92-21) 99251636-39, UAN: 111-000-422, Fax: (92-21) 99251626, e-mail: info@icap.org.pk
Regional Oce-Lahore: 155-156, West Wood Colony, Thokar Niaz Baig, Raiwind Road, Lahore
Phone: (92-42) 37515910-12, UAN: 111-000-422, e-mail: lahore@icap.org.pk
Islamabad Oce: Sector G-10/4, Mauve Area, Islamabad
UAN: 111-000-422, Fax: (92-51) 9106095, e-mail: islamabad@icap.org.pk
Faisalabad Oce: 36-Z, Commerical Center, Near Mujahid, Hospital Madina Town, Faisalabad
Phone: (92-41) 8531028, Fax: (92-41) 8503227, e-mail: faisalabad@icap.org.pk
Multan Oce: 3rd Floor, Parklane Tower, Ocers Colony, Near Eid Gaah Chowk, Khanewal Road, Multan.
Phone: (92-61) 6510511-6510611, Fax: (92-61) 6510411, e-mail: multan@icap.org.pk
Peshawar Oce: House No. 30, Old Jamrud Road, University Town, Peshawar
Phone: (92-91) 5851648, Fax: (92-91) 5851649, e-mail: peshawar@icap.org.pk
Gujranwala Oce: 2nd Floor, Gujranwala Business Center, Opp. Chamber of Commerce, Main G.T. Road, Gujranwala.
Phone: (92-55) 3252710, e-mail: gujranwala@icap.org.pk
Sukkur Oce: Admin Block Sukkur IBA, Airport Road, Sukkur
Phone: (92-71) 5806109, e-mail: sukkur@icap.org.pk
Quetta Oce: Civic Business Center, Hali Road, Quetta Cantt
Phone: (92-81) 2865533, e-mail: quetta@icap.org.pk
Mirpur AJK Oce: Basic Health Unit (BHU) Building Sector D, New City Mirpur, Azad Jammu and Kashmir
e-mail: mirpur@icap.org.pk

2015

FINANCIAL ACCOUNTING
AND REPORTING II
QUESTION BANK

You might also like